Vous êtes sur la page 1sur 176

/* ---------- cquest --------

some of the c frequently asked questions (collected from express computer mag.)
*******************************************************************************
1. what is the error in the following sequence of program.

int i1;
switch(i1)
{
printf("the value of i1 is :");
case 1: printf("%d",i1);
break;
case 2: printf("%d",i1);
break;
default : printf("invalid entry");
}

2. what is an error in the following sequence of a program.

int i1;
switch(i1)
{
case 1: goto lure;
break;
case 2: printf("this is second choice");
break;
default: printf("this is default choice");
}
void fun(void)
{
lure: printf("this is unconditional jump");
}

3. what is an error in the following sequence of a program.

int i;
switch(i)
{
case 1: printf("this is first choice");
break;
case j: printf("this is second choice");
break;
case 1+2+4: printf("this is the third and last choice");
break;
}

4. what is an error in the following sequence of a program.

int i;
switch(i)
{
default: printf("this is default value");
break;
case 1: printf("this is first choice");
break;
case 2: printf("this is the second choice");
}
1. will the following be used as an identifier?
a. sum_of_credits b. initial tree c. final_#
d. while e. section_6 f. bingo-square
g. 2_4_87

2. are the identifiers name and name are same?

3. is it right to type # of #define other than in first column?

4. does c require expressions to be enclosed in parenthesis for while loop?

5. will the preprocessor terminates with semicolon ?

6. what is the return value of scanf statement?

7. what will happen, if there are two statements (without grouping) in if


condition and an else is there for that if.

8. what will be the output of this program.

int no_fish;
no_fish=1;
if (no_fish==1)
printf("the water was to warm\n");
else ;
printf("the wates were all fished out\n");

9. is parenthesis required for conditional expression in if condition?

cv paper:
*****************************************************************

1-18 general (i) data sufficiency


(ii) analytical
(iii) mathematics
19-45 c&unix

1. |x-a|=a-x ans: (c) x<=a

2. there is six letter word vganda . how many ways you can arrange the
letters in the word in such a way that both the a's are together.
ans : 120 (5x4!)

3. if two cards are taken one after another without replacing from
a pack of 52 cards what is the probability for the two cards be
queen. ans : (4/52)*(3/51) (1/17)*(1/13)

4. 51 x 53 x ... x 59 ; symbols ! - factorial


^ - power of 2
(a) 99!/49! (b) (c) (d) (99! x 25!)/(2^24 x 49! x 51!)

5. the ratio fo boys to girls is 6:4. 60% of the boys and 40% of girls
take lunch in the canteen. what % of class takes lunch in canteen.
ans : 52% (60/100)*60 + (40/100)*40
data sufficiency : a) only statement a is sufficent , b is not
b) only statemnet b
c) both are necessary
d) both are not sufficient.

6. x is an integer. is x dvisible by 5?
a) 2x is divisible by 5.
b) 10x is divisible by 5.

ans : a)

7. (a) anna is the tallest girl


(b) anna is taller than all boys.
(q) . is anna the tallest in the class

ans : c

8. maths question
9, 10 analytical

zulus always speak truth and hutus always speak lies. there are
three persons a,b&c. a met b and says " i am a zulu or i am hutu".
we don't know what exactly he said. then b meets c and says to c
that " a is a zulu ". then c replied " no, a is a hutu ".

9. how many zulus are there ? ans 2( check)


10) who must be a zulu ? ans b (check)

11,12.13,14.
-----------

a father f has 5 sons, p,q,r,s,t. not necessarly in this order.


two are of same age. the eldest and youngest cannot be twins. t is elder
to r and younger to q and s has three older brothers

q) who are the twins? s,t


q) who is the oldest and youngest? q, (s&t)
q)
q)

15,16,17,18
----------

there are 7 people who take a test among which m is the worst, r is
disqualified, p and s obtain same marks, t scores less than s and q scores
less than p, n scores higher than every one.
ans : n p s t q r m (may be, just check) or n s p t q r m

c & unix
--------

19. what does chmod 654 stand for.

ans : _rw_r_xr__
20. which of following is used for back-up files?
(a) compress (b) tar (c) make (d) all the above ans : b
21 what does find command do ? ans : search a file
22. what does " calloc" do?
ans : a memory allocation and initialising to zero.
23 what does exit() do?
ans : come out of executing programme.
24. what is the value of 'i'?
i=strlen("blue")+strlen("people")/strlen("red")-strlen("green")
ans : 1
25. i=2
printf("%old %old %old %old ",i, i++,i--,i++);
ans : check the answer.
26. using pointer, changing a to b and b to a is swapping the function
using two address and one temperory variable. a,b are address, t is
temporary variable. how function look like?
ans : swap(int *, int *, int )
27. in 'o' how are the arguments passed?
ans : by value.
28. find the prototype of sine function.
ans : extern double sin(double)
29. scope of a global variable which is declared as static?
ans : file
30. ascii problem
i=..
ans : 6
31 .
32. what is the o/p
printf(" hello \o is the world ");
ans : hello is the world.
33. clarifying the concept addresses used over array ; ie changing
the address of a base element produces what error?
34. child process -- fork
child shell -- sh
35. answer are lex 7 yacc & man read these things in unix
36. what is
int *p(char (*s)[])
ans : p is a function which is returning a pointer to integer
which takes arguments as pointer to array of characters.
************************************************************

---------------------------

wipro paper(system software)


july-1997
------------

part --a
------------------------------------------------------
1) abcd+abcd+abcd+abcd
then the simplified function is
( capital letters are copliments of corresponding letters
a=compliment of a)

[a] a [b] ab [c] abc [d] a(bc)* [e] mone


(bc)*=compliment of bc

ans: e
-------------------------------------
2) a 12 address lines maps to the memory of

[a] 1k bytes [b] 0.5k bytes [c] 2k bytes [d] none

ans: b

----------------------------------------
3) in a processor these are 120 instructions . bits needed to impliment
this instructions
[a] 6 [b] 7 [c] 10 [d] none

ans: b

-----------------------------------------
4) in 8085 microprocessor ready signal does.which of the following
is incorrect statements
[a]it is input to the microprocessor
[b] it sequences the instructions

ans : b
----------------------------------------

5) return address will be returned by function to


[a] pushes to the stack by call
ans : a
------------------------------------------
6)
n=7623
{
temp=n/10;
result=temp*10+ result;
n=n/10
}

ans : 3267
----------------------------------------------
7) if a>b then
f=f(g);
else b>c then
f=g(g);
in this , for 75% times a>b and 25% times b>c then,is 10000 instructions
are there ,then the ratio of f to g
[a] 7500:2500 [b] 7500:625 [c] 7500:625 if a=b=c else
7500:2500
--------------------------------------------------
8) in a compiler there is 36 bit for a word and to store a character 8bits are
needed. in this to store
a character two words are appended .then for storing a k characters string,
how many words are needed.
[a] 2k/9 [b] (2k+8)/9 [c] (k+8)/9 [d] 2*(k+8)/9 [e] none

ans: a
---------------------------------------------------------
9) c program code

int zap(int n)
{
if(n<=1)then zap=1;
else zap=zap(n-3)+zap(n-1);
}
then the call zap(6) gives the values of zap
[a] 8 [b] 9 [c] 6 [d] 12 [e] 15

ans: b
---------------------------------------------------------------

part-b
-------
1) virtual memory size depends on
[a] address lines [b] data bus
[c] disc space [d] a & c [e] none

ans : a
-----------------------------------------------
2) critical section is
[a]
[b] statements which are accessing shared resourses
ans : b
-------------------------------------------------

3) load a
mul a
store t1
load b
mul b
store t2
mul t2
add t1

then the content in accumulator is

ans : a**2+b**4
---------------------------------------------------
4) question (3) in old paper
5) q(4) in old paper
6) question (7) in old paper
7) q(9) in old paper
------------------------------

hughes,delhi:
-------

> > (a)aptitude :25 qns, 20 minutes


> >
> > 1. 2 x 4 analytical gre type qns
> > 2. 2-3 reasoning qns (gre type)
> > 3. probability of getting a sum of 7 when two dices are thrown together
> > 4. rest quantitative questions
> >
> > (b) technical: 50 qns, 45 minutes
> >
> > 1. 3 qns on operating systems. i qn on dijkestra algorithm
> >
> > 2. using which pin it's possible to address 16 bit addresses even though
there
> > are only 8 address bits in 8085? ans: ale
> > 3. voltage gain for an amplifier is 100 while it is operating at 10 volts.
> > what is the o/p voltage wen i/p is 1 volt
> > 4. quality factor indicates a0 quality of inductor b) quality of capacitor
> > c) both
> > 5. qns related to bridges, routers and generators, which osi layer they
> > corresspond to. (refer to stevens 4th chapter)
> > 6.opamp's i/p ciurrent, o/p current and cmrr is given, what is the voltage
> > gain
> > 7. 2-3 qns on scope of static variables in c. qn to view o/p odf a c static
> > var
> > 8. qn to print a value of a pointer
> > 9.resistance increases with temperature in a) metal b) semiconductor
> > 10. a qn to find the physical address from a given virtual address, virtual
> > to physical address table was provided
> > 11. 16 bit mantissa and 8 bit exponent can present what maximum value?
> > 12. 4 bit window size in sliding window protocol, how many acknowledements
can be held?
> > 13. security functionality is provided by which layer of osi
> > 14. frequency spectrums for am, fm and pm (figure given, u'veto tell which
> > kind of modulation it belongs to)
> > 15. among am and fm which is better and why?
> > 16.last stage of ttl nand gate is called: ans: totem pole amplifier
> > 17. sr to jk flip flop conversion. ans: s=jq', r=kq
> > 18. lsb of a shift register is connected to its msb, what is formed: ans:
> > ring counter
> > 19. 2-3 qns based on demorgan's laws (identiies: (a+b)' = a'b', etc)
> > 20. 2 qns on logic gates (o/p of logic gates)
> > 21. diff in iret and ret statements of 8086
> > 22. how many address bytes are required to address an array of memory chips
> > (4 * 6), each chip having 4 memory bits and 8k registers.
> > 23. diff. in memory mapped and i/p o/p mapped input/output (refer a book on
microprocessor)
> > 24. qn on pipeline architecture
> > 25 qn on lapb protocol
> >
> >
>

the quetions may not be in order

q 1-5:

1) 6 persons a,b,c,d,e,f are playing a game of cards . a's father and mother ,
uncle were in the gang. there were two women. b is the mother of a gets more
points then her husband. d got more points then e but less then f. niece of
e got the lowest points. father of a agot more points then f. but would not win
the game.

i). who won the game?


a)a b) b c) f d) d ans: b

ii) who got lowest points ?

a) a b) c c) e d) b ans: a
iii). who is the husband of b ?
a) f b) e c) d d) c ans : c

iv) b was one of the lady. who was other lady?


a) c b) d c) e d) a ans: a

v) who stood second in the game?


a) a b) b c) c d) d ans: c

note: check answers.

6). a is the widow of b. b & c were the only children of e. c is unmarried and
is a doctor. d is the grand daughter of e and studies science. how is a related to
d.

a)aunt b) daughter c) sister d) sister_in_law ans: b

follow the following for q. 7-8.


a) only conclusion i follows definately.
b) ,, ii ,,
c) both i & ii follow
d) neither i nor ii follows.
e) either i or ii follows.

7). statements:
it is some times said that corruption is due to proverty and gross diparities.
it is an uncalled for slurr on the poor. because the poor in india are among the
most unsight persons.

conclusions:
i. upsight persons can't be corrupt persons.
ii. disparities of income make the people corrupt.

ans: c

8) statements:
the indian economy has suffered considerble stresses and strains during the
lostthree years. the general picture is one of stagnations.
conclusions:
i. stresses and strains in economy lead to stagnation .
ii. indian economy is stagnant.

ans: e

q. 9-15 follow this answers.


a) only inference i follows.
b) only inference ii follows.
c) both i & ii follow.
d) neither i nor ii follows.

9). no hawk is a sparrow.


all hawks are hens.
inferences:
i. no hen is a sparrow.
ii. no sparrow is a hen.
ans: c

10). all graduates are chairs.


all chairs are tables.
inferences:
i. all graduates are tables.
ii. all tables are graduates.
ans: a

11) some birds are elephnts.


some elephants are white.
inferences:
i. some birds are white
ii. some white are birds
ans: d

12) all fans are vans.


none van is pan.
i. no pan is fan.
ii.no fan is pan.
ans: d

13). all boats are coats.


no coat is shirt.
i. all boats are shirts
ii. all shitrs are boats.
ans: a

14). no pen is hen.


some hens are healthy.
i. no pen is healthy.
ii. no healthy thing is pen.
ans: d

15) all hopes are doops.


all fears are hairs.

i. all hopes are fears.


all doops are hairs.
ans: d

below we are giving q'es whatever remember to us. it is not clear and order.

1). poem, song, rommance, dirty,couplet


ans: rommance

2). historical, ancient, olden ,inheritant.


ans: inheritant.

3) widow ..........., ans: daughter.

4) for one coding problem ( ie he will give some numbering to alphbets )


ans:
1) priqov
2) mofzb
6). synonyms:
i) placeble ii) emblish iii) incence iv) depricate v) palret

7) oddmen out:

i) bench, chair, winch,thrown,pedastal ans: pedestal


ii) book, volume, saga......
iii) 3,4,20,38 ....... ans: 20
iv) 3,6,52.5,157.5 ....ans: 52.5

8). one age problem father and son 10 years ago........ans: 14,38.

>>from naidu@indy.iitm.ernet.in fri oct 3 17:19:49 1997


received: from [144.16.241.170] by violet.iitm.ernet.in (aix 3.2/ucb 5.64/4.03)
id aa23610; fri, 3 oct 1997 17:19:49 gmt
received: by indy (940816.sgi.8.6.9/940406.sgi)
id raa02593; fri, 3 oct 1997 17:15:32 +0530
date: fri, 3 oct 1997 17:15:32 +0530
from: naidu@indy.iitm.ernet.in (p.v.t. naidu)
message-id: <199710031145.raa02593@indy>
to: mepg9618@violet.iitm.ernet.in
subject: deshaw!
cc: oepg9627@violet.iitm.ernet.in
status: ro

de-shaw software,hyderabad.
---------------------------
section-a

you should write the programs for the following problems in c.

1. swapping two variables x,y without using a temporary variable.


2. a program has been given asking to find the output
ans. it is algorithm for finding g.c.d
3. write a program for reversing the given string.
4. the integers from 1 to n are stored in an array in a random
fashion. but one integer is missing. write a program to find the
missing integer.
ans. idea. the sum of n natural numbers is = n(n+1)/2.
if we subtract the above sum from the sum of all the
numbers in the array , the result is nothing but the
missing number.
5. some bit type of questions has been given on pointers asking to
to find whether it is correct from syntax point of view. and if
it is correct explain what it will do.(around 15 bits).

section-b

answer as many as possible.

6.& 7. two programs on pre-processor directions has been given asking


to find the output.
8. simple question given on pointer
a) kernighan & richie page no. 122
b) schaum series page no. 323
try to understand the pointer problems given here he asked to
explain
in the interview if answer it in exam.
9. write a c program to find whether a stack is progressing in forward
or reverse direction.
10. write a c program that reverses the linked list.

guidelines:
the interview is very tough he is asking to do the problems in
front of him. he is also giving some new problems to solve before
him.
you should have a very good funda of c language.

-----------------------------------------------------------------------
p . v . thatayya naidu
m.s ( cs) hostel : 91-044-2351326
research scholar (pp: #341 naidu)

341, brahmaputra hostel infotech lab : 91-044-2351365


iit madras - 600036 ext:3529/3530
india
email : naidu@indy.iitm.ernet.in
naidu%bronto@iitm.ernet.in
-----------------------------------------------------------------------

this is the paper in iit kharagpur

deshaw 97
part 1 time:20 min
* 20 questions , each question carries 2 marks

1. a solid ice of 11 x 8 x2 inches is made into rod of dia 4 inch. what


is the lenge of rod?

ans: 3.5 inch

2. there were 750 people when the first song was sung. after each
song 50 people are leaving the hall. howmany songs are sung to make
them zero?

ans:16

3. a person is climbing of 60 mts . for every minute he is climbing 6 mts


and slipping 4 mts . after howmany minutes he may reach the top?

ans: (60-6)/2 +1 :28

4. howmany zeros are there in the product of the integer from 1to 100?

ans: 24( not given)

1 to 10 -2 zeros
21 to 30 -3 zeros : because 25 = 5*5
22 *5
24 *5

5. a can do work in 2 hours b can do a work in 3 hours what is the


shortest time tyey can finish the work?
ans: 1hour 12 min.

6..salary is increased by 1200 ,tax is decreased from 12% to 10% but paying
same amount as tax . what is the previsious salary?

ans:6000

7. the least no. which is when devided by 4,6,7 leaves a remainder of 2 ?

ans: 86

8. a man driving the car at twice the speed of auto oneday he was driven
car for 10 min. and car is failed. he left the car and took auto to goto
the office . he spent 30 min. in the auto. what will be the time take by
car to go office?

ans:25 min

9. a report hs 20 wheets, each of 55 lines and each su;ch a line consists


of 65 characters. if the report has to be retyped with each sheet having 65
lines and each line have 75 characters, the percentage of reduction of
no of sheets is closest is to?

ans: 25%

10. out of 100 families in neighbour hood , 55 own radio, 75 own t.v
and 25 own vcr. only 10 families have allof these, and each vcr owner
has tv . if 25 families have the radio only, the no. of families have
only tv are?

ans: 30

aptitude: ( questions 16 t019)

kya kya is an island in the south pacifi . the inhabitants of kya kya
always answer any question with two sentences, one or which is always
true and other is always false.

1. you are walking on the road and come to a fork. you ask ,the inhabitants
ram.laxman, and lila as" which road will take me to the vilage?

ram says: i never speak to strangers. iam new to this place


laxman says: iam married to.lila. take the left road
lila says: iam married to ram. he is not new to this place

ans: left road take yu to the village

2. you find that your boat is stollen. u questioned three inhabitants ot


islands and their replies are

john : i didnot do it. mathew didnot do it


mathew : i didnot do it. krishna didnot do it
krishna: i did not do it; i donot know who did it

ans: mathew stolen the boat

3. you want to speak to the chief of village , u ask three fellows amar
bobby, charles and bobby is wearing red shirt

amar : iam not bobby`s son ; the chief wears red shirt
bobby : iam amars father ; charles is the chief
charles : the chief is one among us; iam the chief

ans: bobby is the chief

4. there is only opne polot in the village(island). you interviewed threem man
koisk ,lorry and mishra
u also notice that koisk is wearing cap.

m says : larry is father in the pilot .larry is not the priests son
koisk : iam the priest on theis island only prists can wear the caps
larry : iam the priest son . koisk is not the prest

ans : koisk is the pilot

this paper has given in iit delhi , so go through this it will give
rough idea;

1. typedef struct{
char *;
nodeptr next;
} * nodeptr;
what does nodeptr stand for?

2. supposing thaty each integer occupies 4 bytes and each charactrer


1 byte , what is the output of the following programme?

#include<stdio.h>
main()
{
int a[] ={ 1,2,3,4,5,6,7};
char c[] = {' a','x','h','o','k'};
printf("%d\t %d ", (&a[3]-&a[0]),(&c[3]-&c[0]));
}
ans : 3 3

3. what is the output of the program?

#include<stdio.h>
main()
{
struct s1 {int i; };
struct s2 {int i; };
struct s1 st1;
struct s2 st2;
st1.i =5;
st2 = st1;
printf(" %d " , st2.i);
}

ans: nothing (error)


expl: diff struct variables should not assigned using "=" operator.

4.what is the output of the program?

#include<stdio.h>
main()
{
int i,j;
int mat[3][3] ={1,2,3,4,5,6,7,8,9};
for (i=2;i>=0;i--)
for ( j=2;j>=0;j--)
printf("%d" , *(*(mat+j)+i));
}

ans : 9 6 3 8 5 2 7 4 1

5.

fun(n);
}
int fun( int n)
{
int i;
for(i=0;i<=n;i++)
fun(n-i);
printf(" well done");

}
howmany times is the printf statement executed for n=10?

ans: zero
expl: befire reaching to printf statement it will goes to infinite loop.

6.what is the output of the program?

main()
{
struct emp{
char emp[];
int empno;
float sal;
};
struct emp member = { "tiger"};
printf(" %d %f", member.empno,member.sal);
ans: error. in struct variable emp[], we have to give array size.
if array size given
ans is 0, 0.00

7. output of the program?

# define infiniteloop while(1)


main()
{
infiniteloop;
printf("done");
}

ans: none
expl: infiniteloop in main ends with ";" . so loop will not reach end;
and the done also will not print.

8. output of the program?

main()
{
int a=2, b=3;
printf(" %d ", a+++b);
}

ans:5
expl: here it evaluates as a++ + b.

9. output of the program?

#define prn(a) printf("%d",a)


#define print(a,b,c) prn(a), prn(b), prn(c)
#define max(a,b) (a<b)? b:a

main()
{
int x=1, y=2;
print(max(x++,y),x,y);
print(max(x++,y),x,y);
}

ans: 3 4 2

10. which of the following is the correct declaration for the function main() ?

ans: main( int , char *[])

11. if ptr is defined as

int *ptr[][100];
which of the following correctly allocates memory for ptr?

ans: ptr = (int *)(malloc(100* sizeof(int));

few more programs here but not available:


************

1. name of the company


infosys technologies ltd.
2. date of int/selection
30th and 31st aug '95
3. panel details
the panel consisted of 3 chota guys, one gm, and one veeps(hrd). the
vp(hrd) was quite painful during the written test and exceedingly
strict. gm was called kris, and had some fundaes
4. area of work
area of work is 100% appli s/w. has 4 groups, mainframes (50% of
revenue), as/400, open systems & client server, and osdc (off-shore
devpt group). usually cs, ee and ece guys are placed in open sys, and
osdc.
5. pay (gross) (take home)
8,925/- pm, gross for mtech/ms. 20% of basic increment every april.
6. perks and benefits
a big office, 15k sq ft, non a/c, about 15 kms from b'lore proper. daily
bus trips from 13 diff places in b'lore. subsidised lunch and breaker. 4
coffee/tea free per day. no advances or loans for house, motor vehicle.
stock offer after 1 year or so. high chances of going abroad for brief
stints of 3-4 months.
7. work atmosphere, and facilities
atmosphere is cool, hajaar iit guys there, and avg age is 23-25. good
news for girls and bad for guys, only about 10-12% of staff are kudis,
the rest are chokras. an at-486 66 mhz for everyone, with facility to
use it as a ws, or a node. e-mail group ids for everyone, and if you
are on osdc, internet ids.
8. selection procedures
two tests, one mathematical ability, another analytical. short-listed
junta for gd (only for pgs). then personal interview (some tech ques
thrown in).
9. qualities students should have
prepare well for tests. speak in the gd, dont be too agressive, try
to give some new thots when it is your turn. personal int is cool.
10. some statistics.
about 500 guys took written test. 30 pgs short-listed for gd & int.
22 selected. 128 btechs selected.
11. some comments about the company
junta company, i.e. takes guys in large nos. and truck-loads. mainly
ugs, and a few pgs. 7th in s/w export revenue-wise. good co. for mtechs.
ms with no interest in appli prog, don't apply.

************

1. name of the company


tisl - tata information systems limited, blr.
2. date of int/selection
date of interview 14/09/95 - interview time 9.30 pm
date of selection 15/09/95
3. panel details
two panels. three guys, two technical and one hr. one technical
for each panel, and the hr guy going between the two.
4. area of work
mostly ibm projects, and some system areas.
5. pay (gross) (take home)
gross 1.38 lakhs for pgs, and guys with cg 9-9.5 get a 5% hike on
basic, amounting to a approx. amount of rs. 400/-pm, and 10% hike
on basic for guys with cg 9.5 and above.
6. perks and benefits
for junta from outside blr, airfare, or first a/c train fare, and
two weeks stay in a hotel for self spouse and children. 10 months hra
(approx. rs.23000) as interest free loan for paying advance.rs.50000
vehicle loan at 5% interest. subsidized lunch. medical, and other
insurance policies.
7. work atmosphere, and facilities
presently promised that each one will get a separate cabin, and a
pc and a terminal attached to the mainframes. e-mail facility will be
provided for most junta.
8. selection procedures
one written test and one interview.
written test
has 3 sections, finding the next element in a series of alphabets,
finding next figure in a series of figures, and finally a quantitative
ability test. this test needs more speed than anything else as you have
to answer something like 30 questions in 10 minutes etc.
interview
first question tell me about yourself, and depending on your answer the
rest of the interview proceeds.

9. qualities students should have, i.e. cgpa, talking skills etc.


for written test cgpa should be above 7.5 for btech and mtech cs junta.
but talking skills are important, mainly be confident, and remember
that they need you as much as you need them if not more.

10. some statistics i.e. how many attended int/test, and how many got thru.
for written, there were hajjar junta. so can't specify the no.
for interview
total, 30 in first set and 10 in second set.from cs, 7 btechs, 2 mtechs
and one ms.
total junta who got thro' 26. from cs, 6 btechs, and 1 mtech.

**********

1. name of the company


bharat electronics ltd ,central &research labs
2. date of selection
22nd september
3. panel details
there were 4 persons in that panel.each of them asked questions on
different subjects.
4. area of work
mostly , for defense and embedded s/w
5. pay
i do not know clearly. i think around rs 7200 to rs 7500 without pf .
6. perks & befits
it is a public sector. may not give any perks.they told that they give
vehicle allowance , canteen subsidy and such things.
7. work atmosphere & facilities
it is good it seems. i heard that the campus is also good.
8. selection procedure
only interview. all the panel members asked questions on different
subjects.usually , thet ask os, networks,s/w and the course i am doing
(dc)now and project. they will ask whether we really want to join in
it. there is 3 years bond. 1 year training period.
9. qualities we should have
i do not know clearly about this. they are not so stringent on cgpa. if
we can impress them in the interview for the questions they asked in
subjects and if we can give the guarantee to join in that company , it
is o.k. .also, they prefer ece degree people with computers postgraduate
degree.
10. statistics
around 20 or 25 people attended. they selected 6 students.(2 m.tech
1 m.tech(e.c.e) 1 m.s(e.c.e) 2 ph.d

***********

1. name of the company


oracle software (i) ltd.
2. date of int/selection
19 th sept.
3. panel details
two project managers - mr. g.ramesh & mr.ravishankar(expert in sco
unix)
4. area of work
they want expertise in unix internals preferably.
5. pay (gross) (take home)
1.3 lacs gross. take home not known.
6. perks and benefits
the best perks among all the software co's.
some of the perks are
- bike loan (interest free)
- visa card.
- anz grindlays atm account.
- club membership.
- house advance
- flexi timings
- catered lunch daily.
- dinner allowance of rs. 50 if you work in the night. etc.
7. work atmosphere, and facilities
don't know much. but they mentioned there's one workstation per
person.
8. selection procedures
intreview only.
9. qualities students should have, i.e. cgpa, talking skills etc.
a good cgpa.(not necessarily very high ).
10. some statistics i.e. how many attended int/test, and how many got thru.
6 mtechs,5 btechs and 2 ms's attended,of which 2 mtechs,2 ms's and
1 btech got selected.

************

1. name of the company


2. date of int/selection
3. panel details
4. area of work
5. pay (gross) (take home)
6. perks and benefits
7. work atmosphere, and facilities
8. selection procedures
9. qualities students should have
10. some statistics.

************

1. name of the company


wipro systems, bangalore.
2. date of int/selection
1st & 2nd september, 1995.
3. panel details
2 guys on the panel.
4. area of work
they are interested in application programming, mainly. they have
groups which work on os and networking also. lots of interest in
mainframe cobol, and other tools. c++ and windows work also going
on.
5. pay (gross) (take home)
about 11k gross.
6. perks and benefits
hajaar perks like house rent advance, vehicle loan, free grub,
medical, lta. medical bills can include ray ban(!).
7. work atmosphere, and facilities
good ambience, and office is on mg road, bangalore. only thing is
uniform, ties, shoes etc. if not on mainframes, you have a ws to
yourself, and if you are on mainframes, only a node for you.
8. selection procedures
only interview. but a short-list initially is put out based on
cgpa. during 94 they had the cut-off at 8.3. the interview is
peaceful and all cs and ee guys selected.
9. qualities students should have
good and decent dress, cgpa above cut-off and goos commn skills.
10. some statistics.
about 12 pgs attended the interview and 10 got thru.

easi -96
------------------

now i am sending easi-tech-96 paper


first priority for --f e m & c++
you should prepare one fem project work
and mention it in your biodata as your present work.then
they will definitely call for interview even you perform
worstly in written test.
one more thing is this year they have big ford project.
so priority is there for mechanicals.
i will send remaining papers by post.
oooookkkkkk!
easi-tech(96)
write the output for following programs.
1) printf("%d%x\n",ox2,12);
2) int a=10;
int b=20;
a=a^b;
b=a^b;
a=a^b;
printf("%d%d\n",a,b);
3)enum
{ellipse,
triangle,
rectangle,
square=100,
circle=5
}
printf{"%d%d%d%d\n",triangle-rectangle,square*circle-rectangle};
4) define the following...
a) pointer to a integer
b) pointer to a char
c) function pointer returning a pointer integer.
5) void a(void);
main()
{
a();
}
void a(void)
{
char a="hellow";
char *b="hellow";
char c[10]="hellow";
printf("%s%s%s\n",a,b,c);
printf("%d%d%d\n",sizeof(a),sizeof(b),sizeof(c)));
}
6) int a=15;
int b=16;
printf("%d %d \n",a&b,a/b); (bitwise operators)
7) int a[5],*p;
for(p=a;p<&a[5];p++)
{
*p=p-a;
printf("%d\n",*p);
}
8) sscanf("xyz abc abc 345" "% *[a-z a-z]lf",&a);
printf("lf",a);
9) not clear
fprintf(- - - - -
10) main()
{
int i=10;
printf("%d",i);
{
int i=20;
printf("%d",i);
}
printf("%d",i);
}
11)struct class
{
int i;
float a;
string[12];
}
sizeof(class)=?
12) int *p;
i=10;
p=i;
printf("%d",*p);
13) fact(5)
int n;
fact(n)
{
sum=n*fact(n-1);
}
next finite element analysys questions
1)
name the equation and explain the terms
a) m/i=f/y=e/r.
b) mx..+cx.+kx=f(t) ans: sdof mass-spring-dashpot equn.
2)
what are the difft.types of finite elements.
3)
what is the size of stiffness matrix for a 4 noded shell element in 3-d
space.
4)
how many nodes are there for a beam element ? why?
5)
write stiffness matrix of truss element.

6)what are plane stress and plane strain elements

7)what is ment by stress smothening


8)what is gauss point
9)what are co and c1 problems

[ok 13q in c and 9q in fem]


this is original full width paper
ssssssssuuuu
yours,
murthy.

1) you should learn pointers(they may ask in i.w)


2)****should learn c++(this is very very imp,based
on this only i got job.one more thing is
class,inheratenci&polymorphism is sufficient)
3)you should prepare one fem project and tell them
that as your own work(this will increase chance
to 99%)
easi-tech
written test pattern
--------------------
1] c test 10q 20minuits
2]f e m test 19q 20minuits
3]aptitude test 15q 20minuits
-----------------------------
f e m test
----------
1)who used the term finite element for the first time?
a)
b)
c) clough
2)derive the jacobien |j| for beam element with strain
energy?
(ans:promlem is not correct please do not attempt)
3)for an element sigma ni=1,which type of element it is?
a)
b)natural co-ordinate
(ans 100% correct)
4)timoshenko beam element theory to consider -- - - -?
a)
b)
c)shear deformation
5)shear locking - - - - ?
(ans is very stiff 'k')
6)membrane locking
(ans is arch element)
7)ex(epsiolan x)=du/dx,ey=dv/dy,r(x,y)=?(gama(x,y)=?)
(ans is du/dy+dv/dx )
8)k=integral b(transpose)*d*b for large deformation
which matrix will get effected?
(ans is d matrix )100%correct
9)for plane strain f(ex,ey,ez,r(x,y) ) - - -
(ans is ez=0)
10)serendipity element is
(ans 8 noded element)
(the element which is having nodes only on boundary is
called serendipity element)
11)if the rotation of element and the displacement about
n-a is same then the order of continuity
(ans is c1)
12)frontal theory is applied for
(please refer any fem book)
13)mindlins theory is applied for
c) this is the answer(both co&c1problems)
14)x=sigmani*xi,u=sigmani*ui which type of element
(refer book)
15)beam subjected to udl find the moments at the
2 nodes
16)integral b(transpose)*sigma(here sigma means stress)*dv
represents?
(ans is internal load vector)
17)integral et(epsiolan transpose)*sigma*dv
p=strain displacement vector
q=stress-strain deformation
find [k]
(ans [k]=[p]t(p transpose)*[q]*[p] )
18)
19)

remember order may not be correct

aptitude test
1)33 1/3 of 101 + 296 is
(ans 1200) check
2)0.625= ? (ans 27/40)
3)one ship goes along the stream direction 28 km and in opposite
direction 13 km in 5 hrs for each direction.what is the velocity
of stream?
(ans 1.5 kmph)
4)cubic root of 3375=?
(ans 15)
5)2020201-565656=?
(ans 1454545)
6)chairs problem
5 chairs=9 tables,12 tables = 7 stools likethat- - -
(ans is 80rs)
7)one clock ringes 7 o'clock in 7 sec.in how many seconds it will
ring 10 o'clock.
(ans 10.5 sec)
8)one watch is showing 30 past 3 .what is the angle between minutes &
hours hand?
(ans 75 degrees)
9)the average of 4 consecutive even numbers is 27. what is the largest
number?
(ans 30)
10) 25 stations ,24 stations are inbetween- - - - -
how many tickets should be required.
(ans 25*24=600)puzzles to puzzle you "s.devi"prob 24
11)one ball was dropped from 8ft height and every time it goes half
of the height. how much distance it will travell before coming to
rest.
(ans 24 approximately)
12)two trains are travelline at equilateral .train a is travelling
in the direction of earths spin.other train b is travelling in
opposite direction of earths spin.which trains wheels will wear
first?and why?
(ans train b .because of less centrifugal force.)

ok hari 3 more ques are there.i hope you can solve easily.
now i have only 3 c ques. i am sendins them in immediate
mail. any whey i will try to send remaining 7 c ques
before 20th.

c questions:what is the out put for following programms


1)main()
{
char a[2];
*a[0]=7;
*a[1]=5;
printf("%d",&a[1]-a)
ans:
ans may be 1.(illegal initialization)
2)
#include<stdio.h>
main(){
char a[]="hellow";
char *b="hellow";
char c[5]="hellow";
printf("%s %s %s ",a,b,c);
printf(" ",sizeof(a),sizeof(b),sizeof(c));
}
(ans is hellow,hellow,hellow
6,2,5 )
3)
#include<stdio.h>
main()

float value=10.00;
printf("%g %0.2g %0.4g %f",value,value,value,value)
}
(ans is 10,10,10,10.000000)
4)
#include<stdio.h>
void function1;
int i-value=100;
main()
{
i-value=50;
function1;
printf("i-value in the function=",i-value);
printf("i-value after the function=",i-value);
}
printf("i-value at the end of main=",i-value);
functioni()
i-value=25;

this is rough idea of the program


ans are
1)i-value in the function=25;
2)i-value after the function=50;
3)i-value at the end of the main=100;

5)
main()
{
funct(int n);
{
switch(n)
case1:
m=2;
break;
case2:
m=5;
break;
case3:
m=7;
break;
default:
m=0;
}
this is rough idea:
(ans:out put is m=0)

remaining 5 programs some what large and complicated.any whey


i will send remaining programs before 20th.
i am sending mainly c paper and some questions.rao
also will send somethig.there are 15 c q's all are discriptive.

1)fallacy
f()
{
int a;
void c;f2(&c,&a);
2)a=0;
b=(a=0)?2:3;
a) what will be the value of b? why
b) if in 1st stmt a=0 is replaced by -1, b=?
c) if in second stmt a=0 is replaced by -1, b=?
3)char *a[2]
int const *p;
int *const p;
struct new { int a;int b; *var[5] (struct new)
4)f()
{
int a=2;
f1(a++);
}
f1(int c)
{
printf("%d", c);
}
c=?
5)f1()
{
f(3);}
f(int t)
{
switch(t);
{
case 2: c=3;
case 3: c=4;
case 4: c=5;
case 5: c=6;
default: c=0;}
value of c?
6)fallacy
int *f1()
{
int a=5;
return &a;
}
f()
int *b=f1()
int c=*b;
}
7)a)function returning an int pointer
b)function ptr returning an int ptr
c)function ptr returning an array of integers
d)array of function ptr returning an array of integers
(see scham series book)
8)fallacy
int a;
short b;
b=a;
9)define function ?explain about arguments?
10)c passes by value or by reference?
11)post processed code for
abc=1;
b=abc1; (1 or 2 blank lines are given)
strcpy(s,"abc");
z=abc;
12)difference between my-strcpy and strcpy ?check
13)f()
{
int *b;
*b=2;
}
14)function which gives a pointer to a binary trees const an integer value
at each code, return function of all the nodes in binary tree.(study)check
15)calling refernce draw the diagram of function stack illustrating the
variables in the -----then were pushed on the stack at the point when
function f2 has been introduced
type def struct
{ double x,double y} point;
main( int argc, char *arg[3])
{double a;
int b,c;
f1(a,b);}
f1(double x, int y)
{
point p;
stack int n;
f2(p,x,y)}
f2(point p, double angle)
{ int i,j,k,int max)
}
_____________________________________________________________
1)least no. when divide by [7 gives remainder 6,6gives 5,5 gives 4 and
soon ans;419
2)what compilation do (ans source code to obj)
3)artficial language is provided which of the language (lisp) check
4)241 change its equivalent octal ?
5)for cube and sphere 3 views are similarly draw one such figure?
6)write a program to exchange two variaables without temp
7)fortran cannot have value by reference
8)4,6,8,__
9)success is to failure, joy is to
10)meaning of jolly?
11)opposite to essential?
12)"raw" means
13)to be good "wrestler " one should have?
14)"command" opposite?
15)genuine opposite?
16)two proverbs are goven
17)sum of two consecutive nos is 55, larger one is?
18)a person goes 4/5 of his usual speed reaches 10min lateto his
destinaton, time taken?
19)80% pass in english, 70%pass in maths , 10%fail in both , 144 pass in
both . how many all appeared to the test?
20)to get a parabola if you cut a section of?
21)bird is flying 120km/hr b/w b to r. two trians at b to r at 60 kmph
the distance trvelled by the bird before it is killed.ans.120
22)meaning of inert
if any are there rao will send you. prepare well
for the interview. mostly on graphics , geometry .prepare questions like
(for interview)prove some of the angles in a triangle are 180.angle in a
half circle is 90.how will you measure hight of building when you are at
the top of the building and if you have stone with you.
best of luck
sekhar

gssl

itest : general intelligence

ii test : technical test

iii test : c-language

i test

some of the questions what i have iam sending.

1. 5,5,,13,13,21,21 ans: 29

2.0,7,26,63,124, ans: 215 i.e.n**3-1 rule following

3. 1,3,5,7, ans: 9 '1' is not a prime number

4. if a person walks at 4/5th of his usual spee he reaches 40min late. if


he walks athis usual speed how much time does he travels.

ans:160min or 2hr 40min

5. two trains a&b start at opposite points 120km at 60kmph. a fly


starting along with train a at 120kmph reaches b then returns back to
touch and continue. by the time two trains meet howmuch distance the fly
would have travelled?

ans : by 1hour both trains meet, so the distance travel by fly in 1hr is
120km.
6.in a class 80% have passed english,70% passed hindi 10% didnot passed
either. if 144 students passed both. what is the total strength of the class.

ans: 240

. find the least number when divided by 7 gives the reminder 6, when
divided by 6 gives reminder 5, when divided by 5 gives reminder 4 and so
on....
ans: 419

8. if a man stands in front of sun what is the first letter of the


direction which is left to him:

ans: north(n)
9.a square is to circle what is cube to

ans: sphere

10. synonyms

i) joy = gay

ii) inert = inactive


11.one wordswill begiven find oddman out: ans:sickle like that
a) sow b) cut c) d) sickel

12. if i bought a cycle before 2days of my birthday and i broke it after


3 days of my birthday the day i broke is mar2, 1956? answer following
logical questions?
i) when is his birthday?
ans: april,28 (due to leap year i.e.1956) but do not keep answer
blindly we just think before choosing answer iam just giving ideaof question

13.
monday
aug25, 96

mr a,
you forgott your umbrella during the party on last friday.i
expected you to collect it on your visit on wednesday,i plan to leave on
this friday.

questions: when a missed umbrella?


when a is supposed to collect it?

when k leaves?

hint: these dates i.e. aug 25 is not exactly given in test, but iam
giving feel of that question.if you solve this question you can solve it
easily in exam.

14.what is my father's sons son to my son?

ans: cousin brother

15.on cutting which solid parabola would be generated ans: cone

16. eulers formula: ans: f+v-e =2;

f= faces;v= vertices;e = number of edges

17. newton rapson method is to find ans:to find the root of f(x) = 0;

18. how many tangents can be dran within three circles if they donot lie
within each other
ans : 12 but this answer is not there i kept 8 as answer

19. in language the fortran which is true. a) fortran uses call by value

20. when a program is compiled what it produces


ans:source code to object code

21. in the following venn diagram shaded region is represented by some


question like that i can't draw figure here thats why iam sendinganswer only

ans: (b-a)' i.e. (b-a) whole dash

22. xy-x+2y = 6 equation is shifted to form equation xy=c what is c?

ans : 4

23.when x is real what is the least value of (x**2-6*x+5)/(x**2+2*x+1)

ans:-1/3

24.when an object like cube or sphere is seen along x,y,z,axes we get the
same.apart from these suggest another which has similar characteristics?

ans:you have to draw the figure ans: is triangular prism you draw
triangular prism.

25. when an object is seen from front side we can see two concentric
squares and topview also without any hidden lines. draw the side view.

ans: i know ans but i canot draw.

26. convert 251 decimal to base 8(i.e. octal)?

ans: similar question but for this question answer is 373

27. how much information can be solved in 1 byte of a ibm pc compatible?

ans: 256

28.what is the language used for artificial intelligence ans: lisp

29. swap two variables without using temporary variable

ans: a= a+b;
b= a-b;
a= a-b:

30.which is not the operating system ?ans: bios

31. what is the optimum number of operations for 2*(x**3)+3*(x**2)+5*x+5?

ans: three multiplications, three additions.

32. a=> bmeans


ans: if a is true b has to be true

33.if a is not invertible and ba = i is not possible

ans: determinant is zero.

34, what is free body diagram


ans: used for analysing fem.

35. a die is thrown twice what is the probability that you get same number

ans:11/36.

dear murthy! yaa! you are luck fellow. i got hcl full paper. i am mailing it.
the quetions are in order. so you no need to prepare answers and mugging that
that quetions. just you mug that answers which are in order or you write all
these answers on the hand compactly while you are going to exam.

paper model:
section i: computer awareness(i.e general things about computer) q.15
-ve
marks: 1/4

section ii: simple c- language q. 15 & -ve marks: -1/4

section iii: on pointers & structures


and c++,java( only 1 on this) q.10 each quetion ->2 marks
-ve
marks: -1
section iv: analytical q.20 each quetion -> 2 marks.
-ve
marks: -1/4

murthy from each section i am giving one are to quetions also because for
checking whether the same paper or not.
and for doubtful answers also i am writing quetions but not writing answers
for these quetions.

section-i

1). piggy backing is a technique for


a) flow control b) sequence c) acknowledgement d) retransmition

ans: c piggy backing

2). the layer in the ost model handles terminal emulation

a) session b) application c) presentation d) transport

ans: b application

3) ans: a odd numbers of errors

4)q. in signed magnitude notation what is the minimum value that can be
represented with 8 bits

a) -128 b) -255 c) -127 d) 0

5) c 20

6) a 120

7) b synchronise the access

8) a system call

9) b the operating system

10) a 177333

11) d used as a network layer protocall in network and windows system

12) b has to be unique in the sub network

13)q. there is an employer table with key feilds as employer no. data in every
n'th row are needed for a simple following queries will get required results.

a) select a employe no. from employe a , where exists from employe b


where a employe no. >= b employe having (count(*) mod n)=0
b) select employe no. from employe a, employe b where
a employe no. >= b employ no. grouply employe no. having (count(*) mod n=0 )
c) both a& b
d)none of the above

14)q. type duplicates of a row in a table customer with non uniform key feild
customer no. you can see

a) delete from costomer where customer no. exists


( select distinct customer no. from customer having count )
b) delete customer a where customer no. in
(select customer b where custermer no. equal to b custemor no. ) and a rowid >
b rowid
c) delete customer a where custermor no. in
( select customer no. from customer a, customer b )
d) none of the above

15) c volatile modifier


---------- section i over with 15 quetions -------------------

section-ii

section ii is not covered completly but it is very very easy. you can do it
very easely.

1) ans: recursion

2) long int size


a) 4 bytes b) 2 bytes c) compiler dependent d) 8 bytes
ans: compiler dependent
note: order of a,b,c,d are doubt but answer is correct.

3) x=2,y=6,z=6
x=y==z;
printf(%d",x) ?

4) if(x>2)?3:4

5)

6)

7) ans: c 6 ( quetion on enum )

8) ----
---
---
--
14) c : class a,b and c can have member functions with same name.

15) ans: d none of the above

section-iii

1) ans: b it does not work when rp is the last element in the linked list

2) ans: a always

3) ans: b 13

4) ans: b 16

5) ans: d 55,55
6) ans: c 5,10,10,3

7) ---

8) ans:d 4

9) ans: c 5

10)ans: c semicolon missing

section-iv

following are not in order:

2. m > d > y ans: (a)

6. 10 in 4 seconds,
? in 6 minutes = 10x6x60/4 = 900 ans: (a)

7. a=2, b=4, c=5


(a+b)/c - c/(a+b) = 11/30 (ans).

8. 100(100000000+100000000)/10000 = 2x1000000 (ans).

9. what does the hexanumber e78 in radix 7.


(a) 12455 (b) 14153 (c) 14256 (d) 13541 (e) 131112 ans: (d)

10. q is not equal to zero and k = (q x n - s)/2 find n?


(a) (2 x k + s)/q (b) (2 x s x k)/q (c) (2 x k - s)/q
(d) (2 x k + s x q)/q (e) (k + s)/q

(from gre book page no:411)


data:
a causes b or c, but not both
f occurs only if b occurs
d occurs if b or c occurs
e occurs only if c occurs
j occurs only if e or f occurs
d causes g,h or both
h occurs if e occurs
g occurs if f occurs

note: check following answers.

11. if a occurs which of the following must occurs

i. f & g
ii. e and h
iii. d
(a) i only (b) ii only (c) iii only (d) i,ii, & iii
(e) i & ii (or) ii & iii but not both ans: (e)

12. if b occurs which must occur

(a) d (b) d and g (c) g and h (d) f and g (e) j ans: (a)

13. if j occurs which must have occured

(a) e (b) either b or c (c) both e & f (d) b (e) both b & c ans: (b)

14. which may occurs as a result of cause not mentioned

(1) d (2) a (3) f

(a) 1 only (b) 2 only (c) 1 & 2 (d) 2 & 3 (e) 1,2,3 ans: (c)

15. e occurs which one cannot occurs

(a) a (b) f (c) d (d) c (e) j ans: (b)

11 to 15:- ----------- e , a , b , c , b ---------------

below are in order:

16. to 20. answers:

e
a
c
a
e

--------------- over ----------------

best of luck

in written test in each section you have to get minimum marks i.e you have to
pass in each section. there will be questions from c, c++, java. about 10
questions in c++ in the written test.

interview: i will send interview quetions tomorrow early morning.


ok
yours lovingly
srihari.k

three. i have answers for q/a paper. for q/c you


should try for answers.

infosys 1997. time 1hr.


paper code q/a. marks 50.
--------------------------------------------------------------
1)at 6'o clock clock ticks 6 times. the time between first and
last ticks was 30sec. how much time it takes at 12'o clock.

ans. 66 sec. 2 marks.

2)three friends divided some bullets equally. after all of them


shot 4 bullets the total no.of remaining bullets is equal to that of
one has after division. find the original number divided.

ans. x x x
x-4 x-4 x-4
3x-12 = x
x= 6
ans is 18 2 marks

3)a ship went on a voyage after 180 miles a plane statrted with 10 times
speed that of the ship. find the distance when they meet from
starting point.

ans. 180 + (x/10) = x


x = 20
ans is 180+20=200miles. 2 marks

4) fill the empty slots.


three football teams are there. given below the list of maches.

played won lost draw goals for goals against


a 2 2 *0 *0 *7 1
b 2 *0 *1 1 2 4
c 2 *0 *1 *1 3 7

the slots with stars are answers. 4 marks


bc drew with 2-2
a won on b by 2-0
a won on c by 5-1
( you have to fill the blanks at appropriate star symbols.)`
5) there are 3 societies a,b,c. a lent tractors to b and c as many
as they had. after some time b gave as many tractors to a and c
as many as they have. after sometime c did the same thing.
at the end of this transaction each one of them had 24.
find the tractors each orginally had.

ans a had 39, b had 21, c had 12, 4 marks


6) there n stations on a railroad. after adding x stations 46 additional
tickets have to be printed. find n and x.

ans. let n(n-1) = t;


(n+x)(n+x-1) = t+46;
trail and error method x=2 and n=11 4 marks

7)given that april 1 is tuesday. a,b,c are 3 persons told that their
farewell party was on
a - may 8, thursday
b - may 10,tuesday
c - june 8, friday
out of a,b,c one is only correct one of the regarding month,day,date.
other told two correct and the third person told all wrong.what is
correct date,month,day. 5 marks
(ans may be may 10 sunday. check once again)
8)there are 4 parties. df,gs,dl(depositloss),ew ran for a contest.
anup,sujit,john made the following statements regarding results.
anup said either df or ew will definitely win
sujit said he is confident that df will not win
john said he is confident that neither ew nor dl will win
the result has come. only one of the above three has made a correct
statement. who has made the correct statement and who has won
the contest. 5 marks.
(ans dl )
9)five people a,b,c,d,e are related to each other. four of them make
one true statement each as follows.
i) b is my father's brother. (ans. d said this)
ii)e is my mother-in-law. ( b )
iii)c is my son-in-law's brother. ( e )
iv)a is my brother's wife. ( c )
who said each statement.
10 marks.

10) all members of d are also members of a


all '' e '' d
all '' c '' both a and b
not all '' a are members of d
not all '' d '' e
some questions on these conditions.(5questions 5 marks)
11)
boys are allowed to watch football at c.v.raman auditorium subjected to
conditions.
i)the boy over age 16 can wear overcoat
ii)no boy over age 15 can wear cap
iii)to watch the football either he has to wear overcoat or cap
or both
iv) a boy with an umberella or above 16 or both cannot wear sweater.
v) boys must either not watch football or wear sweater.

what is the appearence of the boy who is watching football.


try to solve this question................

ok!!!!!
infosys paper 1997.
paper code q/c.
------------------------------------------------------------------------

1) be * be = acb
a,b,c,e are non zero numbers find b,e.
ans) b=1 e=9
2) a,b,c,d,e are having numerical values. there are some conditions
given
a) a=c <===> b!=e
b) difference between a and c as same as difference between c and b
as same as difference between a and d
c) c<a and c>d
then find a,b,c,d,e
3) there are six cards in which it has two king cards. all cards are
turned down and two cards are opened
a) what is the possobility to get at least one king.
b) what is the possibility to get two kings.
4) a person went to a shop and asked for change for 1.15paise.
but he said that he could not only give change for one rupee.
but also for 50p,25p,10p and 5p. what were the coins he had
ans) 1-->50 4--->10p 1--->25p
5) there are 3 nurses and they work altogether only once in a week.
no nurse is called to work for 3 consecutive days.
nurse 1 is off on tueseday,thursday and sunday.
nurse 2 is off on saturday.
nurse 3 is off on thursday,sunday.
no two nurses are off more than once a week.
find the day on which all the 3 nurses were on work.
6) there are 5 persons a,b,c,d,e and each is wearing a block or white
cap on his head. a person can see the caps of the remaining 4 but can't
see his own cap. a person wearing white says true and who wears block
says false.
i) a says i see 3 whites and 1 block
ii) b says i see 4 blocks
iii) e says i see 4 whites
iiii) c says i see 3 blocks and 1 white.
now find the caps weared by a,b,c,d and e
7) there are two women, kavitha and shamili and two males shyam, aravind
who are musicians. out of these four one is a pianist, one flutist,
violinist and drummer.
i) across aravind beats pianist
ii) across shyam is not a flutist
iii) kavitha's left is a pianist
iiii) shamili's left is not a drummer
v) flutist and drummer are married.
8) 1/3 ed of the contents of a container evaporated on the 1 st day.
3/4 th of the remaining contents of the container evaporated the
second day. what part of the contents of the container are left at
the end of the second day.
9) a man covered 28 steps in 30 seconds but he decided to move fast and
covered 34 steps in 18 seconds. how many steps are there on the
escalator when stationary.
10) all fair skinned, rich, handsome, muscular, lean and employed are
tall men
1) all lean men are muscular.
2) no fairskinned person who is not rich is handsome.
3) some muscular men are handsome.
4) all handsome are fairskinned.
5) no person who is neither fair skinned nor muscular is enplyed.
6) we unable to recall this condition and question also incomplete.
------------------------------------------------------------------------

/* ------------info -------------*/
/*
george summers puzzle books are suggested.
-----------------------------
infosys technologies limited.
-----------------------------
question paper
--------------
part 1.
------
(1) 9 cards are there. u have to arrange them in a 3*3 matrix.
cards are of 4 colors.they are red,yellow,blue,green.
conditions for arrangement: one red card must be in first row
or second row.2 green cards should be in 3rd column.yellow
cards must be in the 3 corners only. two blue cards must be in
the 2nd row. atleast one green card in each row.
solution:
yello red gren
blu blu gren
yello gren yello
2. 4 cards are placed on a table, each card has two colors. u
don't know the color of the back side of eachcard.4 persons a
b c and d are sitting on the table before the cards. they can
see red, green red and blue .out of the 4 poeple 2 always lie.
they see the color on the reverse side and give the following
comment
a: yello/green
b: neither blue/nor green
c: blue/yello
d: blue/ yello
find out the color on the other side of the 4 cards.

3.red and brown tribes [from barrons gre] conditions to


get married with each other.

4. venn diagram regarding rich, muscular, soft-skinned,


employed, etc.,( refer barrons gre guide)

------------------------------------------------------------------------
part 2.
-------
1. sakuntala devi's puzzle book : puzzles to puzzle you.
problem no: 3. ( brothers and sisters)
a family i know has several children. each boy in this
family has as many sisters as brothers but each girl has
twice as many brothers as sisters. how many brothers
and sisters are there?
ans: 4 boys and 3 girls.
2. no. of animals is 11 more than the no. of birds. if the
no. of birds were the no. of animals and no. of animals
were the no. of birds( ie., interchanging no.s of animals
and birds.), the total no. of legs get reduced by one fifth
(1/5). how many no. of birds and animals were there?
ans: birds:11,animals:22
3. in a soap company a soap is manufactured with 11 parts.
for making one soap you will get 1 part as scrap. at the
end of the day u have 251 such scraps. from that how many
soaps can be manufactured? ans: 22 + 2+ 1 = 25.
4. 2 * * |
3 * * | no. 7 does not occur in this
---------------- |
5 * * | multiplication.
* 4 * |
* * 3 | find the product.
---------------- |
* * * * * |
---------------- |
--------------------------------------------------
ans 2 8 1
3 2 2
-----
5 6 2
5 6 2 0
8 4 3 0 0
---------
9 0 4 8 2
---------
5. there is a 5digit no. 3 pairs of sum is eleven each.
last digit is 3 times the first one.
3 rd digit is 3 less than the second.
4 th digit is 4 more than the second one.
find the digit.
ans : 25296.
6. there are five thieves, each loot a bakery one after the
other such that the first one takes 1/2 of the total no.
of the breads plus 1/2 of a bread. similarly 2nd, 3rd,4th
and 5fth also did the same. after the fifth one no. of
breads remained are 3. initially how many breads were there?
ans : 31.
7.escalator problem of sakuntala devi 'puzzles to puzzle'book.
problem no: problem 27( down the escalator)
ans : the no of steps in the stair way : 46.
8.harbour line and main line problem of sakuntala devi puzzle
book. ans : 4/5.
( more puzzles book)
9.there are some chicken in a poultry. they are fed with corn
one sack of corn will come for 9 days.the farmer decides to
sell some chicken and wanted to hold 12 chicken with him.
he cuts the feed by 10% and sack of corn comes for 30 days.
so initially how many chicken are there?
10.two people x & y walk on the wall of a godown in opposite
direction. they meet at a point on one side and then go
ahead. x after walking for some time, walks in opposite
direction for 15 mtrs.then again he turns back and walks
in the original direction. what distance did y walk before
they met again, if x walks 11 mtrs by the time y walks
8 mtrs.
11.problem from sakuntala devi 'puzzles to puzzle u'.
problem no: 23( walking back to happiness.)
the walking time : 55 mins.
with this the paper has been completed. for the lack time
i have'nt typed those probs, which are in sakuntala devi
puzzle books. i wish all of you the best.
send me the mail immediately after the written test will
be over. thank you.
-sarma. b. y.

interview practice
-------------------------------------

practicing for the interview means practicing several behaviors


- not just answering questions. you must dress well, watch
your body language and posture, practice your manners and eye
contact as well as practice answering questions correctly,
smoothly, and with confidence. the practice questions below, in
one form or another, account for a large percentage of
interview questions. with each question, you are given a
series of choices as to how you might answer the question.
when you select an answer, you will learn whether your answer is
correct-and why. answering these questions will help you polish
your interviewing techniques. the questions and answers in
this excercise are generic and, in many cases, must be
tailored to your individual situation. still, the logic behind
the answer remains essentially the same.

why are you the best person for the job? i 've held a lot of
positions like this one, and that experience will help me
here." "because i am good at what i do."

"our discussion here leads me to believe this is a good place to


work."

"you need someone who can produce results, and my background and
experience are proof of my ability. for example..."

if asked a point blank question such as: are you creative? are
you analytical? can you work under pressure? etc. what is
the best way to answer?

answer yes or no.

answer yes and give a specific example.

answer yes and give an explanation.

3 tell me about yourself.

1.outline personal data, hobbies, and interests.

2.give an overview of your personality and work habits.

3.give three specific examples of your personality traits and


accomplishments.
4. what was your cgpa? why is it low?

1.my cgpa is -----. basically because i had a lot of fun in college.

2.my gpa is ----- because i held a full time job while in


college, working my way through school. it is not relfective
of my ability to do the job.

3.my gpa is ------ which i don't think is too low, at least


compared to my friends.

5. what are you looking for in a position?

1."i'm looking for an opportunity to apply my skills and


contribute to the growth of the company while helping create
some advancement opportunities for myself."

2."i'm looking for an organization that will appreciate my


contributions and reward my efforts."

3."i'm looking for a position that will allow me to make


enough money to support my lifestyle. i am a hard worker and
will give a concerted effort to earn the money i need."

6. what do you know about our organization?

1."i've done a little homework and here is what i know about


your organization...(cite examples)"

2."everything i've seen and heard makes me want to be a part of


this organization. i understand your industry is ________ and
your primary customer is __________. a particularly exciting
part of your business appears to be _________ ."

3."i know enough to know this is an exciting place to work. it


appears to be fit for my career goals."

7. what is a weakness?

1. i get really frustrated with my co-workers when they whine


about everything.

2. i don't "toot my own horn". i like to go in and get the job


done and i don't dwell on who gets credit.

3. i don't think i have a weakness.

8. how would you characterize your work style?

1. i am a very driven person. i make a list of what needs to be


done and then tackle the items until i've completed the job.

2. i show up whenever i'm scheduled to work.

3.i haven't held a job yet so i don't know what my work style will be.

9. what are your strengths?


1."i am good at giving constructive criticism to my coworkers.
this honesty is something i'm very proud of and have found
essential to having open working relationships."

2."i consider myself to be very consistent. i have proven myself


to be someone who can be counted upon to do what is
expected."

3."i would have to chose between two skills. i am very proud of


my determination and ability to get things done. at the same
time, i am very proud of my analytical abilities and problem
solving skills. these skills combine to give me a unique
ability to solve problems and then implement the solutions."

10. how would co-workers describe you?

1."they perceive me as a leader. the people who have worked with


me learned great deal and accomplished in many cases more than
they thought possible."

2."my employees would tell you they got direction when they
needed and the room to work when it was appropriate. i believe
a measure of a good manager is how much he is able to get done
through others."

3."they perceive me as someone who cared about them personally


and had high expectations. i get a great deal of satisfaction
from helping others do their best. my former employees would
highlight three of my priorities which are to build loyalty
and a team environment, obtain results and develop people."

11. what did you like and dislike about college?

1. i didn't like the tremendous ammount of homework some


professors assigned us. i liked those professors who
realized, i had a life outside of class.

2.i liked the opportunity to be involved on campus. it was a


small campus that allowed students to take responsibility,
organize events and be a part of the planning team. i can't
really think of anything i disliked, college was a wonderful
experience.

3.i liked having wednesdays off. i think the thing i disliked


most was living in the dorms.

12. are you willing to relocate?

1.no.

2.i'm open to opportunities within the company so if that


involves relocation i would consider it.

3.i'd move, but i know your headquarters is in alaska and that's


too cold for me.

13. what are your short term goals?


1."short term, i just want a job."

2."bills are beginning to pile up. in the short run i need to


find work so i can keep up with my obligations."

3."short-term, i'd like to find a position that is a good fit


and where i can contribute to a company's bottom line. the
position we are here to discuss today would appear to be such an
opportunity. could you tell me more about it?"

14. where do you want to be in 5 years?

1.i haven't really thought that far ahead but i think i'd want your job.

2."if selected i would hope to meet my goals and take advantage


of opportunitites to learn so i will be considered for other
positions within the company. i hope to build my career with a
company such as this one."

3."long term, i hope to start my own business."

15. describe a situation you've encountered with a difficult


customer.

1.in my last job i dealt with customers at a retail store. one


customer wanted to return an item she had obviously bought
several years ago, which was against store policy. i talked to
her calmly and explained our policies and encouraged her to shop
and i would exchange the item however i could not give a refund.

2. i rarely encounter a difficult customer because of my


excellent customer service.

3. i had a customer once who claimed her steak was not cooked
correctly....i picked up a knife and cut it open and explained
that's the way it's supposed to be.

16. before we go any further, what kind of money do you need to


make?

1.i feel i am worth at least $30,000."

2."my salary requirements are negotiable. your firm has a


reputation of compensating employees fairly and i trust you
would do the same in my case. i am very interested in finding
the right opportunity and will be open to any fair offer when i
do so."

3."money is not very important to me. i need to be able to pay


the bills but the work environment is far more important to
me."

face to face interview preparation

so you have a job interview, now what?

you have sent out hundreds of resumes and finally a human


resource person calls to schedule a time for you to come in for
a face to face interview. this is a great company, and you
think you are perfect for the job. now, you must demonstrate
that to the hiring authority you are perfect for the position.

here are some pointers to keep in mind when going in for the
interview.

prepare - do not go in attempting to "wing" it. keep in mind


there are lots of competitors for the job, and you will only
have this opportunity to impress the hiring authority.research
and know the company, their history, products, services and
reputation.

prepare answers to questions that will likely come up, like


"what is your biggest weakness?" "what is your greatest
failure?" "describe your best and worse boss?" "tell me
something about yourself?" "why did you leave your last job?"
"how can you contribute to this company in this job?"

there will likely be a longer list of questions, each a


potential landmine that can eliminate you from the competition.
keep your answers short and to the point. do not ramble.
always be honest and avoid negative statements. wear your best
suit - the first impressions of a candidate are lasting. it is
better to be over-dressed than under-dressed. your appearance
will demonstrate that you are taking the initiative. go out of
your way to make a good impression. even if you never wear a
suit to work, even if it is a production job, the interviewer
will be pleased by your effort to impress.

be on time - it is better for you to be early and wait for your


appointment time than to keep the interviewer waiting and
wondering. if you miss your appointment time you may not get
another. if a problem or emergency arises, and they do, call
immediately and explain and offer to come in at another time,
whenever is convenient for the interviewer.

body language - sit up, look attentive, do not cross your arms
or legs make eye contact. interviewers rely on body language to
determine your level of interest.

present a positive attitude - companies seek pleasant and


positive people.

bring extra copies of your resume - you may interview with a


team of people, do not make them share one resume.

avoid smoking and drinking alcohol before and during an


interview. if going to lunch with an interviewer, decline
alcholic beverages even if the interviewer orders a cocktail
with their meal. if you are a smoker, abstain before and during
the interview. smoking can offend many people, even smoking
before an interview can leave an odor which may be offensive to
the interviewer.

do not lead with money or benefit questions in the initial


interview - if your main motivation is money it will probably
turn off most hiring authorities. convince them you can do the
job and want the job before discussing money and benefits.
money, benefits and vacation will come up and can be discussed
should the company wish to make you an offer.

sell yourself - if you don't no one else will. use facts and
figures to quantify your experience and ability. tell them the
things that will benefit the company if they hire you.

show interest in the job - if you act cool they may not realize
that you want the job. companies want someone who is interested
and enthusiastic about the job and the company. don't make the
hiring authority guess if you want the job.

do not close any doors - no matter what you see or hear during
the interview don't overreact. obstacles may come up regarding
money, relocation or other issues that can be resolved later if
you leave the issue open.

getting an offer - if an offer is made to you during an


interview never, reject it outright. ask for time to think it
over to consider money and the specific details of the offer.
always respond positively. if you let them know you want the
job it will be a lot easier to negotiate items like salary
later.

when the interview is over - ask if there are any reservations


about your ability to do the job. deal with these now. ask
what is the next step in the interview process. if you like
the job and want an offer now is the time to tell them. do not
assume they will know if you want the job or not.

follow-up - be sure to send a note letting them know you


appreciate the opportunity to interview and reiterating your
interest in the position. keep the note brief, and to the
point.

1.in my last job i dealt with customers at a retail store. one


customer wanted to return an item she had obviously bought
several years ago,

2."if selected i would hope to meet my goals and take advantage


of opportunitites to learn so i will be considered for other
positions within the company. i hope to build my career with a
company such as this one."

3."long term, i hope to start my own business."

15. describe a situation you've encountered with a difficult


customer.

interview -tips
-------------------------------
1)you should do some rough work immediately
adjacent to the problem(aptitede)
2)you should write the explanationary notes
adjacent to the c program.

interview aim:
1)project should be on fem.
2)you have to explain project clearly
3)don't tell "iam expert in fem or in c"
other wise they will ask more typical ques.
4)you tell "i know c++ & c . i am learning c++
simultaneously with c."
q1)tell about your work (means project work)
q2)tell about your family
ans: sir,i came from a family which gives higher
priority to education.we are two.because of
my parents active guidance we are pursuing
higher studies.i sister studied msc and she
got married.
i studied b.tech in n.u
10 in- - - -
inter in - - - -
q3)why you are coming to soft ware?
ans:well sir,
what i have learned all these days definitely
increased my technical skills.but a the same time also
increased my analytical,prouble shooting& problem solving
skills.
what ever may be the technical backgrount ,above
skills are necessary for s-w.
rapid grouth,frequent updating&attractive compensation pay
packages.
q4)what is your ambition?
ans)sir , i would like to become a person who is someone in
soft ware.
q5)where do you like to see after 5 years?
ans:sir
one year from now i would like to become a good soft ware
expert.
two years from now i would like to become a person who
handles team work(i.e project leader)
five years from now i have to handle morethan one project
at a time.
q6)why this college?
sir, from the very first begining i have had special
carving(interest) to pursue(get) my higher studies
from any reputed institution.i hope i have succeeded.
q7)your +points
ans:i am a team person (you sould mension as captain of cricket)
i can handle problems as they came
q8)your -points
sir i am a bit over confident(or emotional)and i believes
people in first insident.
q9)if you can't selected to this job ,what can you do?
ans:
sir i am confident that i should definitely get job in your
organization.i that worst case will happened i will try
to rectify my errors and once again i will try for your
organization.
even then if i failed , i will search for another
organization which is suitable for my potential.
q10)why this company?
ans:
sir iam looking in the company for the following
things.
1)which is a gaint in s-w.
2)which is well reputed organization.
3)which is having noninert enveronment
i hope i will find all the above things in your organization.
ooooooooook!
these are most frequent interview questions.
you should prepare well and leave the result to god.

sssssssssseu!
yours,
murthy.

> we do not remember the questions exactly, here we just tried to cover
> the major areas.
>
> paper consists of two sections : 1. analytical (20 marks)
> 2. c skills (20 marks)total time 45 min
>
> analytical:
> -------------
>
> 1. given a cube, with different colors on its faces, and then is cut
> into 64 pieces, and the questions relate to the colors of different
> colored small cubes.
>
> 2. a few ladies and gents sit around table in some given order and 4
> questions are about their seating arrangement with some restrictions.
>
> 3. draw a venn diagram for 3 items : white, flowers, cloth
>
> ans : draw 3 circles each intersecting the other , with white kept in the
> middle.
>
> 4. a problem related to seating arrangement of 4 people ( 2 ladies and
> 2 gents) with some restrictions
>
> 5. problem related to milk with water added to it for three times
>
> ans: 20.5 litres
>
> 6. problem related to diagrams . five diagrams were given and asked to
> find the sixth one.
>
> c-skills:
> --------
>
> 1. max number of nodes in a binary tree with height 3 is 20 : ans: false
>
> 2. 10,20,30,40,50,60 : give the order when put in a queue and in a stack
>
> ans : queue : 10,20,30,40,50,60
> stack : 60,50,40,30,20,10
>
> 3. debugging is the process of finding
>
> ans : logical and runtime errors
>
> 4. trace the error:
>
> void main(){
> int &a;
> /* some other stuff here */
> }
> ans: syntax error
>
> 5. a problem with a function named 'myvalue' will be given and asked to
> find the value of main() for an argument of 150,
>
> ans : 150
>
> 6. some problems related to 'for' loops.
>
> 7. problem on conditional inclusion.
>
> 8. problem related to depth first and breadth first search (dsa subject)
>
> 9. study the syntax of malloc function
>
> 10. locate the error
>
> int arr (20);
> ans: syntax error.
>
>
> the above are only the guide lines, may not be the exact paper.
>
> wishing you all the best

********************************************************************************
there are two sections in intergraph.

1.aptitude 20 nos
2.'c' programming

1.non-verbal questions

________________________________________________________
| *| | | | | | |
| | | | | | | |
|_______|_______|_______|_______|_______|_______|__________|
| | | | | | | |
| | | | | | | |
|_______|_______|_______|_______|_______|_______|____________|

like this

2.venn diagram

draw venn diagram for flowers,white,clothes

(complete question is not there)

3.analytical

abcdef -->boys
pqrst -->girls

ad
st
br
cq
4.

a.a will with d


b.s will with t
c.b will with r
d.c and q wants to be in the same team.

5. matricide:mother::homicide:?
a)human b)children c)father d)none

8.rs.3 costs 5 oranges.then sold each orange at 1 rupee each.then sold each
orange at 1 rupee each.for rs.36 profit what is the no. of oranges
required?

ans.90

9.the wages of the worker was increased from rs.18 to rs.21 per week.the
average working hours is 7 hrs/day instead of 8 hrs/day.how much fast wokers
should do work?

10.a sales person visits m,n,p,q,r and s

s can be varied according to the question.

ans. s mnpqr

13.a cube is colored on the opposite faces blue,black and yellow.it is


then cut into 36 cubes.out of which 32 were small cubes and 4 where big
cubes.

a)how many cubes were there,colored on the face


b)how many cubes were colored on three faces.
16.a cask contain 40 liters milk.8 liters we taken out and filled with
water.the same process is contained second&third time.how much amount of
milk will be there?

'c' language
------------

1). main()
{
int a[100];
int i;
x= 0
y= 100
---------
-----------
for(i=1;i<=4;i++)

return;

2). union(one questions) ans:a

3). debugging

ans: logical&routine error

technical test (mascot)


--------- ----

total 6 sections.
1)computer fundamentals
2)algorithms
3)hardware
4)software
5)general awareness
6)languages

* all together 60 questions with each section containing 10 questions.


* all are multiple choice questions

1. one nibble = ?
2. hexadecimal and octal representation of 1024 ?
3. what a compiler does ?
4. dos ? (whether graphical interface or character interface and two more
choices are given)
5. cpu - abbrevation
6. fortran used as ( ans : scientific language)
7. ascii character set ? (total number)
8. windows nt- what nt stands for?
9. which is not an input device ? (keyboard, disk, mouse,lightpen)
10. which is not a pointing device ?( mouse,joy stick,lightpen none)
11. microsoft chief ?
12. 4gl ? (fortran,sql,ada, ....)
13. father of computers?
14. which of them is object oriented language.?
15. power pc is the product of ? (microsoft,ibm,apple,motorola,intel)
(a combination of any correct 3 companies)
16. latest processor used ? (pentium, power pc, pentium pro,none)
17. ms word is ?
18. first super computer built in india. (ans : param)
19. which of the following companies donot manufacture chips?
(microsoft, motorola, intel, hp)
20. lan - abbrevation
21. wan - abbrevation
22. modem is ? (related to electrical hardware.- like modulator &demodulator)
23. fdd - abbrevation ( floppy disk drive)
24. bit - abbrevation
25. information is ? (message, data,processed data,none)
26. which is not networking ?(internet,ethernet,arcnet,none)
27. one gigabyte =?( 2 power30 , 2 power 20 ,2 power 10 none)
28. which of the following is not rdbms ? (sybase, sql,acess,none)_
29. oracle is ? (ans : relationasl data base management system)
30. in oracle, table means ( ans : collection of records)
31. dma - abbrevation (direct memory access,discrete memory access,
disk memoryaccess)
32. what is meant bystatic variable ?
33. what is meant by queue? {refer any data structures text}
34. what is meant by stack? {refer any data structures text}
35. the processor used in first ibm pc? (8086,8088,zig4,intel)
36. difference between 80286 and 80287
37. in bubble sort , no. of comparisons required ?(ans : formula : n*(n-1)/2)
38. no. of comparisons of an item in 100 items by binary comparison?
( 10,25,50 100)
39. crt - (cathode ray tube)
40. no. of entryvalues are there in ideally in a subroutine.
41. binary tree?
42. flow in one direction ? (single linked list,double linked
list,.......,....)
43. electron screen size ? (here, 2 lines of algorithm is given.
name the algoirithm)
44. which is not storage device.? (printer,cd rom,disk,none)
45. a question regarding memory ? ( least used memory,recently unused
memory,..)
46. iso - (international standard organisation)
47. html - (hyper text makeup language)
48. flow chart for factorial n? (ans :choice a)
49. what is meant by recursion ?

50. struct(s)
{
int a;
long b;
}
union (u)
{int a;
long b;
}
print sizeof(s)and sizeof(u) if sizeof(int)=4 and sizeof(long)=4

51.switch (i)
i=1;
case 1
i++;
case 2
++i;
break; ( ans : 1,2,3,none)
case 3
--i;
output of i after executing the program

52. char s;
char s[6]= " hello";
printf("%s ",s[6]);

output of the above program ? (0, ascii 0, i,unpredictable)

53. unsigned char c;


for ( c=0;c!=256;c++2)
printf("%d",c);

no. of times the loop is executed ? (127,128,256,infinitely)

54. int i;
i=0;
repeat
i=i+1; <====== pascal program
print i;
until(i<10)
end
no. of times the loop is executed?

55. convert (int a,var ,int b;int c)


{
a=10;
b=4-;
c=120;
}
convert (inta,b,c)
{ <====== pascal program
a=1;
b=4;
c=12;
}
convert (a,b,c) ? (10,40,120
10,40,12,..........,............)

56. procedure a
begin

--------

end <====== pascal program


procedure b no. of errors in the program ?(1,2,3,none)
begin

-----------

end

57. int i;
i=2;
i++;
if(i=4)
{
printf(i=4);
}
else
{
printf(i=3);
}
output of the program ? (4,3,unpredictable,none)

58. what is fat?.

file allocation table

quantitative apptitude test :


---------------------------

total 44 questions are there.


refer r.s. aggarwal

1. how many degrees hours hand rotate in 10 minutes?


2. 1/(10 power 18) - 1/(10 power 20) = ?
3. 0 <x < 1: which is greater ? (1/(x square),1/x,x,x square)
4. c=a/b ; a-1=c what is the relation between a&b
5. n = ((8.42)**2)*(95. ....)/((0.982)*(64.....)**2)
if n contains the first three digits as 988,which of the following is the
nearest answer (9.88,98.8,988,none)
6. the sum of 7 consecutive odd integers with 27 as the fourth number.
7. (66666666666666)**2 + 888888888888=?
8. 2486724...... express it in product of 3 numbers.(the given number
is not exactly correct. choices are given. very easy)
9. radius of sphere is increased by 50%. by how much percentage is surface
area is increased.
10. in which of the following , 2 as a common factor , can be eliminated.

ans : log(x square)/log(y square) = log(x)/log(y)

antonyms :
11. auspicisious
12. rehalishly
13. & 14. - sentence correction
15, 16 , 17 - spelling mistakes
18, 19 - two questions

6 letters are assigned certain numbers.


in each qusetion , 4 combination of digits are given . of which , which gives
a meaningful word.
20 & 21. - 4 sentences will be given in each question. arrange them in
logical order .
22. 5 straight lines are cut in a circle. a question regarding this.

reasoning :

a) if statement a alone is required


b) if statement b alone is required
c) if bothe statements a & b are required
d) neither

23. does mr. mathew give tution ?


a) mathew is a teacher
b) in school, the teacher is not expected to give tutions.

24. there is no power cut in advanced country


if t is a city, is there power cut in that city?
a) z is a advanced country
b) t ia a capital city of country z

25. tanzanians are in east africa. tanzanizns are good in either education ,
dance or music
a) tanzanians are good at education
b) east africa are good at dance & music.

remaining questions 26 - 29 are of the same type.

30. on onw day, 14 children are admitted in a school by their mothers.


2 are sisters, 3 are brothers,2 are brother and sister and 2 are twins.
the rest are singles. how many mothers came? (5 , 7, 9, 14)

31. problem cisterns & pipes.

it will take 8hrs to fill a cistern. but due to leak at the bottom, it
take 10 hrs to full it. in how many hrs, th eful tank will be emptied
because of leak.? (18,8,40,10)

32. question regarding men & work

a does alone a work in 4 days


b does alone a work in 8 days
c does alone a work in 10 days
a & b together ------
c & b together ------

33. in 3.5 kg rod , there is 74% silver. if it is alloyed with a 0.5 kg rod,
the % of silver goes p to 84%. the percent of silver in 0.5 kg rod?

34. two chords of lengths 2l1 and 2l2 are drawn in a circle. their lengths
are inversely proportional to the straight distance joining the centre.
find the radius of circle.

35. a kg of tea costs rs 49.50 . but the supplier gives 10 gms less for
every kg he sold. what is the actual cost.

36 - 40

a question on transportation table. 5 questions were given.

-------------------------------------------------------
a b c d e

-------------------------------------------------------
a x 12 8 20 6
-------------------------------------------------------
b 12 x 12 5 9
-------------------------------------------------------
c 20 8 x 4 7
-------------------------------------------------------
d 3 15 6 x 10
-------------------------------------------------------
e 12 5 8 3 x
-------------------------------------------------------

a,b,c,d,e denote the stages.


x denote the start of the stage.
the bus goes from a to e and e to a woith back stops at b,c, & d
for each the charge is rs.0.70
the numbers in the table are how many passengers are there in the bus
upto that stage.( the numbers given the table are not correct)

36. total no. of passengers in onward journey

37. total amount in the conductors bag just before the bus reaches
the stage c

38. how many rs. 1.40 tickets are issued to passengers in backward journey.

39. if the bus breaks down between the stages c & d , the amount refunded to
passengers.

40. if the ticket costs rs.1.50, how much is the profit in backward journey.

remaining 4 questions may be fron english.

---------------------------------------------------------

718, 1047, 1243


570 ------> infosys
1173 -----> moscot

hello junta!,
i am sending some papers received from iit kharag . it consists
novel, verifone, oracle etc etc....
with best wishes,

novel
******************************************

part 1

1). a beggr collects cigarette stubs and makes one ful cigarette
with every 7 stubs. once he gets 49 stubs . how many cigarettes
can he smoke totally.
ans. 8
2). a soldiar looses his way in a thick jungle at random walks
from his camp but mathematically in an interestingg fashion.
first he walks one mile east then half mile to north. then 1/4
mile to west, then 1/8 mile to south and so on making a loop.
finally hoe far he is from his camp and in which direction.
ans: in north and south directions
1/2 - 1/8 + 1/32 - 1/128 + 1/512 - and so on
= 1/2/((1-(-1/4))
similarly in east and west directions
1- 1/4 + 1/16 - 1/64 + 1/256 - and so on
= 1/(( 1- ( - 1/4))
add both the answers
3). hoe 1000000000 can be written as a product of two factors
neither of them containing zeros
ans 2 power 9 x 5 ppower 9 ( check the answer )
4). conversation between two mathematcians:
first : i have three childern. thew pproduct of their ages is 36
. if you sum their ages . it is exactly same as my neighbour's
door number on my left. the sacond mathematiciaan verfies the
door number and says that the not sufficient . then the first
says " o.k one more clue is that my youngest is the youngest"
immmediately the second mathematician answers . can you aanswer
the questoion asked by the first mathematician?
what are the childeren ages? ans 2 and 3 and 6
5). light glows for every 13 seconds . how many times did it
between 1:57:58 and 3:20:47 am
ans : 383 + 1 = 384

6). 500 men are arranged in an array of 10 rows and 50 columns .


all tallest among each row aare asked to fall out . and the
shortest among them is a. similarly after resuming that to their
originaal podsitions that the shorteest among each column are
asked to fall out. and the longest among them is b . now who is
taller among a and b ?
ans a
7). a person spending out 1/3 for cloths , 1/5 of the remsaining
for food and 1/4 of the remaining for travelles is left with
rs 100/- . how he had in the begining ?
ans rs 250/-
8). there are six boxes containing 5 , 7 , 14 , 16 , 18 , 29
balls of either red or blue in colour. some boxes contain only
red balls and others contain only blue . one sales man sold one
box out of them and then he says " i have the same number of red
balls left out as that of blue ". which box is the one he solds
out ?
ans : total no of balls = 89 and (89-29 /2 = 60/2 = 30
and also 14 + 16 = 5 + 7 + 18 = 30
9). a chain is broken into three pieces of equal lenths
conttaining 3 links each. it is taken to a backsmith to join into
a single continuous one . how many links are to tobe opened to
make it ?
ans : 2.
10). grass in lawn grows equally thickand in a uniform rate. it
takes 24 days for 70 cows and 60 for 30 cows . how many cows can
eat away the same in 96 days.?
ans : 18 or 19
11). there is a certain four digit number whose fourth digit is
twise the first digit.
third digit is three more than second digit.
sum of the first and fourth digits twise the third number.
what was that number ?
ans : 2034 and 4368

if you qualify in the first part then you have to appear for
the second i.e the following part.

part 2.

1. from a vessel on the first day, 1/3rd of the liquid


evaporates. on the second day 3/4th of the remaining liquid
evaporates. what fraction of the volume is present at the end of
the ii day.
2. an orange galss has orange juice. and white glass has apple
juice. bothe equal volume 50ml of the orange juice is taken and
poured into the apple juice. 50ml from the white glass is poured
into the orange glass. of the two quantities, the amount of
apple juice in the orange glass and the amount of orange juice in
the white glass, which one is greater and by how much?
3. there is a 4 inch cube painted on all sides. this is cut
into no of 1 inch cubes. what is the no of cubes which have no
pointed sides.
4. sam and mala have a conversation. sam says i am vertainly not
over 40. mala says i am 38 and you are atleast 5 years older
than me. now sam says you are atleast 39. all the sattements by
the two are false. how hold are they realy.
5. ram singh goes to his office in the city, every day from his
suburbun house. his driver mangaram drops him at the railway
station in the morning and picks him up in the evening. every
evening ram singh reaches the station at 5 o'clock. mangaram
also reaches at the same time. one day ramsingh started early
from his office and came to the station at 4 o'clock. not
wanting to wait for the car he starts walking home. mangaram
starts at normal time, picks him up on the way and takes him back
house, half an hour early. how much time did ram singh walk.
6. in a railway station, there are tow trains going. one in the
harbour line and one in the main line, each having a frequency of
10 minutes. the main line service starts at 5 o'clock. the
harbour line starts at 5.02a.m. a man goes to the station every
day to catch the first train. what is the probability of man
catchinhg the first train
7. some people went for vaction. unfortunately it rained for 13
days when they were there. but whenever it rained in the
morning, they had clean afternood and vice versa. in all they
enjoyed 11 morning and 12 afternoons. how many days did they
stay there totally
8. exalator problem repeat
9. a survey was taken among 100 people to firn their preference
of watching t.v. programmes. there are 3 channels. given no of

people who watch


at least channel 1
" " 2
" " 3
no channels at all
atleast channels 1and 3
" " 1 and 2
" " 2 and 3
find the no of people who watched all three.
10. albert and fernandes they have two leg swimming race. both
start from opposite and of the pool. on the first leg, the boys
pass each other at 18 mt from the deep end of the pool. during
the ii leg they pass at 10 mt from the shallow end of the pool.
both go at const speed. but one of them is faster. each boy
rests for 4 sec to see at the end of the i leg. what is the
length of the pool.
11. t h i s each alphabet stands for one
i s digit, what is the maximum value t
-------------- can take
x f x x
x x u x
--------------
x x n x x
--------------

1. an escalator is descending at constant speed. a walks down


and takes 50 steps to reach the bottom. b runs down and takes 90
steps in the same time as a takes 10 steps. how many steps are
visible when the escalator is not operating.
2. evvery day a cyclist meets a train at a particular crossing.
the road is straignt before the crossing and both are travelling
in the same direction. cyclist travels with a speed of 10 kmph.
one day the cyclist comes late by 25 min. and meets the train 5km
before the crossing. what is the seppd of the train.
3. five persons muckerjee, misra, iyer, patil and sharma, all
take then first or middle names in the full names. there are 4
persons having i or middle name of kumar, 3 persons with mohan, 2
persons withdev and 1 anil.
--either mukherjee and patil have a i or middle name of dev or
misra and iyer have their i or middle name of dev
--of mukherkjee and misre, either both of them have a first or
middle name of mohan or neither have a first or middle name of
mohan
--either iyer of sharma has a i or middle name of kumar hut not
both.
who has the i or middle name of anil
4. reading conprehension
5. a bird keeper has got ppigeon, m mynas and s sparrows. the
keeper goes for lunch leaving his assistant to watch the birds.
a. suppose p=10, m=5, s=8 when the bird keeper comes back, the
assistant informs the x birds have escaped. the bird keeper
exclaims oh no! all my sparrows are gone. how many birds flew
away.
b. when the bird keeper come back, the assistand told him that
x birds have escaped. the keeper realised that atleast2 sparrows
have escaped. what is minimum no of birds that can escape.
6. select from the five alternatives a,b,c,d,e
at the end of each question ,two conditions will be given.
the choices are to filled at follows.
a. if a definete conclusion can be drawn from condition 1
b. if a definete conclusion can be drawn from condition 2
c. if a definete conclusion can be drawn from condition 1 and 2
d. if a definete conclusion can be drawn from condition 1 or 2
e. no conclusion can be drawn using both conditions
1. person 1 says n<5
person says n>5
person 3 says 3n>20
person 4 says 3n>10
person 5 says n<8
what is value of n
a) 1. no of persons who speak false being less than no of persons
who tells the truth.
2. person 2 is telling the truth.
b) 1. no of persong telling the truth is greater than no of
persons telling lies
2. person 5 is telling the truth.
7. there are n coins on a table. there are two players a&b.
you can take 1 or 2 coins at a time. the person who takes the
last coin is the loser. a always starts first
--1. if n=7
a) a can always win by taking two coins in his first chanse
b) b can win only if a takes two coins in his first chance.
c) b can always win by proper play
d) none of the above
--2. a can win by proper play if n is equal to
a) 13 b) 37 c) 22 d) 34 e) 48 ans. e.
--3. b can win by proper play if n is equal to
a) 25 b)26 c) 32 d) 41 e) none
--4. if n<4, can a win by proper play always

8. two turns have vertain pecular charcteristics. one of them


always lies on monday, wednesday, friday. \the other always lies
on tuesdays, thursdays and saturdays. on the other days they tel
the truth. you are given a conversation.
person a-- today is sunday my name is anil
person b-- today is tuesday, my name is bill

answers for selected questions


2. equal 1. 150
3. 8 2. 60 kmph
4. 37(m),41(s) 3. mukherjee
5. 45 min. 8. today is tuesday
6. 0.8
7. 18
11. t max value = 4

3. anil kumar mukherjee


kumar misra dev
mohan iyer dev
kumar patil mohan
mohan sharma kumar

verifone interview.
****************************:-
here, we had three single man panels. they are seeing the subjects
taken and asking questions in that mainly. they didn't go deep into
any subject and they were just asking overview. when they catch any
new word they are asking about it.

these are some of the questions asked to us.(not to a single person.)

what is a finite automata.


what is a turing machine.
how many processors are there in a pentium microprocessor. in sparc.
difference between risc and cisc.
is risc always fast.

what is a real time system.


name some real time os
what are the characteristics of real time os.
is dos a real time os.
what is a kernel,shell.

what is binary search, traversal, hashing etc.


given a scenario what is the suitable data structure.
write a code to count the no. of 1's in a binary rep. of a number.
memory taken for char *, int * etc.
char *cp; int *ip; cp++,ip++ - what is the result.
compare the no. of bytes in unix and dos for long char short int.
how to make programs portable on unix and dos under such circumstances.
in c++, what is a constructor, destructor etc.
what is friend etc.

what is waterfall model, prototype model etc.


what is testing. what is unit testing, integration testing etc.

what is indexing in databases?


what is atomicity?
can recursive pgms be written in c++, write a recursive pgm to
calculate factorial in c++.
what is best data structure to store the processes info in a real time
operating system?

verifone ------> paper:-


*********************************
verifone test questions :

there are two parts :

1. aptitute test : 15 minutes, 20 questions

some questions are:


(not in order)
1. a question (first one)on addition of fraction of inches
a. was the answer

2. there were 36 chairs. how many ways can they be placed such
that all rows have equal no. of chairs and atleast three chairs
are there in each row and there are atleast three rows.
5 ways.

3. there are 27 balls, of which 1 is heavier. given a balance


how many times you need to weigh to find out the odd ball.
3 weighs.

4. product of three consecutive nos. 210. what is the sum


of two least numbers
11.

5. if the area of the sqaure is increased by 69 % how


much the length of the side will increase?
30%
6. if the sum of five consecutive nos. 35? how many prime nos
are there :
2 primes.

7. if the length of the rectangle is reduced by 20% and breath


is increased by 20 % what is the net change ?
4 % decrease
8. a question on sets.
there are some 20 basketball players & 30 football players,
and 25 cricket players. 1 of them plays all the three games.
8 of them plays atleast two games. they are 50 altogether.
how many of them plays none of the games.
9. a question on directions.
b is 20 miles east of a. d is 30 miles east of c. e is 10 miles
north of d. c is 20 miles north of b. how far e is from a?
some 3 questions on reasoning like,
10. if you say that giving stock options to employees increases the
productivity of the company, which of the following sentences
support it.
a) giving stock options increases the morale of the employees
..
..
etc.,
11. gamblers comes to the amusement parks. there are some amusement
parks in each city. there are some gamblers in each city. so
what can you infer.
a) amusement park always have gamblers.
..
..
etc.,

2. technical questions.

i. electrical & electronics : 15 questions


1.a circuit with nand gates. (ans. may be xor)

2.cmrr. relates to (options not in order)


voltage follower
non invering amplifier
inverting amplifier
integrator

3. given a circuit , give the ouput.


(ans. may be triangular wave.)

4. o/p of an assembly code.


mulitply by 11.

5. how to handle asynchronous events.


a) polling
b) interrrupt
etc.

ii)data structures, algo., & complexity theory : 5 questions


iii) os : 5 questions
iv) networks and hardware: 5 questions
v) databases and misc.: 5 questions
vi) c pgm. : 5 questions
some more 10 questions.

1. if w is a sequence of strings without a and w' is its reversal


then waw' is accepted by:

context free grammmars

2. whether all recusive pgm can be writtten iteratively?


yes.
3. what data structes you will use if you want to go to first record from
the last and vice versa?
doubly linked circular list

4. given 10000 nos. and 48mb memory. what is the complexity of the
efficient sorting algo.? (the algo. is not mentioned)
o(n)

5. given a c code and ask what it does?


code was something similar to bubble sort and that
particular code does the sorting in desending order and the complexity
is o(n^2)(which is the next question).

6.
a code like this is given.
a. for(i=0;i<num;i++)
b. for(i=num;i>0;i--)
assuming no code optimization and assume that the microprocessor
has flags etc. which one is correct.

b will execute faster.

7. if there are too many page faults what is the problem?


8. to ensure one pgm. doesnt corrupt other pgm. in a multi-pgm enviornment
what you should do?

9. which one you will use to implement critical section?


binary semaphore

10. which one is not needed for multi-processing. enviornment?


options are: virtual memory,security,time sharing,none of the
above.

11. which one is not done by data link layer ?


bit stuffing, lrc,crc,parity check

12. which one is not related to data link layer?

13. which one is not suitable for client-server application?


tcp/ip,message passing,rpc,none of the above.

14. what is sql.


procedural relational db query language.

15. indexing in databases give you


options were like 1.efficient deleting and inserting
2.efficient deleting.
etc.
16. int a=1,b=2,c=3;
printf("%d,%d",a,b,c);
what is the output?
17. scope of static variable ............
in a file.

18. for(i=0; i<=10;i++,printf("%d",i)); +- (+- is there in the questions)

20. real time os should have


a)fast context switch
b)virtual memory etc.

21. messages are transferred in some e71 code, where after 7 bits of data,
1 bit of stopping data is to be transferred. what should be done.
options were like
a) send directly
b) send after encoding
etc.
22. there are three processes a, b, c. a sends data to b. b removes the
header stores it and sends the data to c. c returns it to b. b receives the
message, identifies the message and adds the header that was stored and
sends to a.b receives the messages from c such that atmost 'm' messages
b
are pending.
identify the best data structure.

23. a question in compiler about the heap and stack allocation of memory.

24. struct
{
char a[3];
int b;
}x;
char *cp;

a) size of x is 7.
b
b)
c)
d) cp takes the size of a pointer.
(d ) is the ans.

--------------------------------------------------------------------------------
over
--------------------------------------------------------------------------------

*******************************************************
oracle
*******************************************************

section 2:

1. what is sparese matrices?. give (at least) two methods for implemetation
rather than two dimentional array.
2.what are cheap locks/latches?.
3.what is two phase locking?. name two locks.
4. what are volatile variables in c?. what is their significance ?.
5. will these two work in same manner
#define intp int *
typedef int * inpp;
6. what are binary trees?. what is its use?.
7.

section 3 :

a). write header file containing functions used, etc (c),


problem is to maitain a queue. user has to give size and type of queue.
this problem is like this i don't remember exactly.
b). c++
1. what is polymorphism?
2. what is inheritence?.
3. mention four object oriented programming languages?>
4. mention basic concepts of oop.
5. what are messages in oop?.
6. what is garbase collection?.
7.what is object?.
8. what is a class?.
section 4:
1. expand the following:
a.sei b. iso
2. what are different levels of sei?.
3. what is significance of iso?>
4. expand the following:
a. www
b. http
c. html
d. tcp/ip
5. what is black box testing?.
6. explain the following:
1. white box testing
2. white box testing
3. boundary testing
4 stress
5. negative
6. system
7. unit
8.module
9.destructive

--------------------------------------------------------------------------------

*********************************************
infosys
*********************************************

1) there are two balls touching each other circumferencically.


the radius of the big ball is 4 times the diameter of the small
ball.the outer small ball rotates in anticlockwise direction
circumferencically over the bigger one at the rate of 16 rev/sec.
the bigger wheel also rotates anticlockwise at nrev/sec. what is
'n' for the horizontal line from the centre of small wheel always
is horizontal.

2) 1 2 3 4
+ 3 4 5 5
----------
4 6 8 9
- 2 3 4 5
----------
2 3 4 4
+ 1 2 5 4
------------
3 6 9 8

q) strike off any digit from each number in seven rows (need not
be at same place) and combine the same operations with 3 digit numbers
to get the same addition. after this strike off another digit from all
and add all the no.s to get the same 2 digit no. perform the same
process again with 1 digit no.s. give the ' no.s in 7 rows at
each stage.

3) there is a safe with a 5 digit no. the 4th digit is 4 greater than
second digit, while 3rd digit is 3 less than 2nd digit. the 1st digit
is thrice the last digit. there are 3 pairs whose sum is 11. find
the number. ans) 65292.

4) there are 2 guards bal and pal walking on the side of a wall of a
wearhouse(12m x 11m) in opposite directions. they meet at a point and
bal says to pal " see you again in the other side". after a few moments
of walking bal decides to go back for a smoke but he changes his
direction again to his previous one after 10 minutes of walking in
the other(opposite) direction remembering that pal will be waiting
for to meet.if bal and pal walk 8 and 11 feet respectively, how
much distance they would have travelled before meeting again.

5) xxx)xxxxx(xxx
3xx
-------
xxx
x3x
------
xxx
3xx
------

q) find the 5 digit no.


hint: 5 is used atleast once in the calculation.
6) afly is there 1 feet below the ceiling right across a wall length
is 30m at equal distance from both the ends. there is a spider 1 feet
above floor right across the long wall eqidistant from both the ends.
if the width of the room is 12m and 12m, what distance is to be
travelled by the spider to catch the fly? if it takes the shortest
path.

7) ramesh sit around a round table with some other men. he has one
rupee more than his right person and this person in turn has 1 rupee
more than the person to his right and so on, ramesh decided to give
1 rupee to his right & he in turn 2 rupees to his right and 3 rupees
to his right & so on. this process went on till a person has
'no money' to give to his right. at this time he has 4 times the
money to his right person. how many men are there along with ramesh
and what is the money with poorest fellow.

8)question related to probabilities of removing the red ball from a


basket,given that two balls are removed from the basket and the other
ball is red. the basket contains blue,red,yellow balls.

9)venkat has 1boy&2daughters.the product of these children age is 72.


the sum of their ages give the door numberof venkat.boy is elder of
three.can you tell the ages of all the three.

analytical
----------
1)l:says all of my other 4 friends have money
m:says that p said that exact one has money
n:says that l said that precisely two have money
o:says that m said that 3 of others have money.
p:land n said that they have money.
all are liers.who has money&who doesn't have?

2)a hotel has two,the east wing and the west wing.some east wing rooms
but not all have an ocean view(ov).all ww have a harbour view(hv).the
charge for all rooms is identical, except as follows
* extra charge for all hv rooms on or above the 3rd floor
* extra charge for all ov rooms except those without balcony
* extra charge for some hv rooms on the first two floor&some ew rooms
without ov but having kitchen facilities. (gre modrl test 3-question
1j-22)
3)post man has a data of name surname door no.pet name of 4 families.
but only one is correct for each family.there are a set of statements
&questions.
4)4 couples have a party.depending on the set of statements,find who
insulted whom and who is the host of the party.
5)5 women given some of their heights(tall,medium,short)hair( long,
plainted),stards(black or brown), sari,2 medium,2-short.tall->no
sari.plainted->medium.answer the combinations.

1) a person has to go both northwards&southwards in search of a job.


he decides to go by the first train he encounters.there are trains for
every 15 min both southwards and northwards.first train towards south
is at 6:00 a.m. and that towards north is at 6:10 .if the person arrives
at any random time,what is the probability that he gets into a train
towards north.
2) a person has his own coach&whenever he goes to railway station he
takes his coach.one day he was supposed to reach the railway station
at 5 o'clock.but he finished his work early and reached at 3 o'clock.
then he rung up his residence and asked to send the coach immediately.
he came to know that the coach has left just now to tje railway station.
he thought that the coach has left just now to the railway station.he
thought that he should not waste his time and started moving towards
his residence at the speed of 3mi/hr.on the way,he gets the coach and
reaches home at 6 o'clock.how far is his residence from railway
station.

3)radha,geeta&revathi went for a picnic.after a few days they forgot the


date,day and month on which they went to picnic.radha said that it was
onthursday,may 8 and geeta said that it was thursday may 10.revathi
said friday jun 8.now one of them told all things wrongly,others one
thing wrong and the last two things wrongly.if april 1st is tuesday
what is the right day,date and month?

-------------------------------------------------------------------------

*******************************************
siemens info
*******************************************

this paper consists 6 parts. all are multiple choice q's


1)general
2)c/unix
3)c++/motif
4)database
5)x-windows
6)ms-windows
we have written q's not acc. to each part.total 50. q's. time is
sufficient.
if u have basic idea about all of the u can easily answer the paper.
paper
------
1)which of following operator can't be overloaded.
a)== b)++ c)?! d)<=
2)#include<iostream.h>
main()
{
printf("hello world");
}
the program prints hello world without changing main() the o/p should
be
intialisation
hello world
desruct
the changes should be
a)iostream operator<<(iostream os, char*s)
os<<'intialisation'<<(hello world)<<destruct
b) c) d)none of the above
3)cdpath shell variable is in(c-shell)
a) b) c) d)
4) term stickily bit is related to a)kernel
b)undeletable file
c) d)none
5)semaphore variable is different from ordinary variable by
6)swap(int x,y)
{
int temp;
temp=x;
x=y;
y=temp;
}
main()
{
int x=2;y=3;
swap(x,y);
}
after calling swap ,what are yhe values x&y?
7) static variable will be visible in
a)fn. in which they are defined
b)module " " " "
c)all the program
d)none
8)unix system is
a)multi processing
b)multi processing ,multiuser
c)multi processing ,multiuser,multitasking
d)multiuser,multitasking
9)x.25 protocol encapsulates the follwing layers
a)network
b)datalink
c)physical
d)all of the above
e)none of the above
10)tcp/ip can work on
a)ethernet
b)tokenring
c)a&b
d)none
11)a node has the ip address 138.50.10.7 and 138.50.10.9.but it is
transmitting data from node1 to node2only. the reason may be
a)a node cannot have more than one address
b)class a should have second octet different
c)classb " " " " "
d)a,b,c
12) the osi layer from bottom to top
13)for an application which exceeds 64k the memory model should be
a)medium
b)huge
c)large
d)none
14)the condition required for dead lock in unix sustem is
15)set-user-id is related to (in unix)
16) bourne shell has
a)history record
b)
c)
d)
17)wrong statement about c++
a)code removably
b)encapsulation of data and code
c)program easy maintenance
d)program runs faster
18)struct base {int a,b;
base();
int virtual function1();
}
struct derv1:base{
int b,c,d;
derv1()
int virtual function1();
}
struct derv2 : base
{int a,e;
}
base::base()
{
a=2;b=3;
}
derv1::derv1(){
b=5;
c=10;d=11;}
base::function1()
{return(100);
}
derv1::function1()
{
return(200);
}
main()
base ba;
derv1 d1,d2;
printf("%d %d",d1.a,d1.b)
o/p is
a)a=2;b=3;
b)a=3; b=2;
c)a=5; b=10;
d)none
19) for the above program answer the following q's
main()
base da;
derv1 d1;
derv2 d2;
printf("%d %d %d",da.function1(),d1.function1(),d2.function1());
o/p is
a)100,200,200;
b)200,100,200;
c)200,200,100;
d)none
20)struct {
int x;
int y;
}abc;
you can not access x by the following
1)abc-->x;
2)abc[0]-->x;
abc.x;
(abc)-->x;
a)1,2,3
b)2&3
c)1&2
d)1,3,4
21) automatic variables are destroyed after fn. ends because
a)stored in swap
b)stored in stack and poped out after fn. returns
c)stored in data area
d)stored in disk
22) relation between x-application and x-server (x-win)
23)uil(user interface language) (x-win)
24)which is right in ms-windows
a)application has single qvalue system has multiple qvalue
b) " multiple " " single "
c)" " " multiple "
d)none
25)widget in x-windows is
26)gadget in x_windows is
27)variable destdir in make program is accessed as
a)$(destdir)
b)${destdir}
c)destdir
d)destdir
28)the keystroke mouse entrie are interpreted in ms windows as
a)interrupt
b)message
c)event
d)none of the above
29)link between program and out side world (ms -win)
a)device driver and hardware disk
b)application and device driver
c)application and hardware device
d)none
30)ms -windows is
a)multitasking
b) c) d)
31)dynimic scoping is
32) after logout the process still runs in the background by giving
the command
a)nohop
b)
33)process dies out but still waita
a)exit
b)wakeup
c)zombie
d)steep
34)in dynamic memory allocation we use
a)doubly linked list
b)circularly linked
c)b trees
d)l trees
e)none
35)to find the key of search the data structure is
a)hask key
b)trees
c)linked lists
d)records
36)data base
--------------------------
--------------------------------
employ_code salary employ_code leave
---------------------------
----------------------------------
from to

--------------------------------------
1236 1500 1238 --- ---
1237 2000 1238 --- ---
1238 2500 1237 ---
-----
1237 --- ---
1237 --- ---
1237 --- ---
-------------------------- --------------------------------------
select employ_code,employ_data ,leave
the number of rows in the o/p
a)18
b)6
c)7
d)3
37)dbms
38)read about sql,db
39)which is true
a)bridge connects dissimiler lanand protocol insensitive
b)router " " " " "
c)gateway " " " " "
d)none of the above
40)read types of tree traversals.
41)42)43) simple programs on pointers in c

_____________________________________________________________________

*********************************************
novel network paper
*********************************************

the paper consists os three sections.


1. aptitude 15 questions 20 min.
2. system concepts 20 questions 20 min.
3. 'c' 15 questions 20 min.

note:::::

i'm sendin the questions, somany have no answers. u do check the


answers for all the given questions also.

all questions are multiple choices only.

u do try to cover the related topics to get some understanding.


in interview they are asking c c++ operating system concepts
and networks.
they will ask about ur interesting subjects, u try to have good
grip on those intresting subjects.

they selected 11 out of 55 students.


they asked about c c++ project work and os,networks.
but they selected only 2 , one from cs and another is btech ece.

section 1 is main factor selection to the interview

--------------------------------------------------------------
in the interveiw you should be very active .and should be frank to say
no also.one cs student said that he don't know network and he hasn't
taken networks subject.but he has selected.
ok best of luck.

section 1::::;

*. gre book test paper 3 section 5


question no. 8 to 12 (ships wxyz starting on dec. 23rd..........)
* 5 programs are sheduled from monday to saterday, monday is not holiday,
pqrst are the programs. the day before p is holiday, and some other clues are
given, we have to find the sequence (4 questions)

ans: tuesday is the holiday


p comes on wednesday ,
u can do the remaining very easily.
* suppose u r the presoner, there are two guards who will tell truth or
one will tell truth. there is a gate for liberty and another foe hell.

some questions are given, like


i. are u tell truth?
ii. another tell truth or not??
iii. both of u r telling truth??
iv. both of u lieing??
v. --------------

they gave a. b. c. d. and asking about which sequencing is sufficient to


find the gate for liberty??
( i can't give exact thing)

* there are wife and husband at one side of river. they have one child,
they want to cross the river. the child can't be lonly. the boat boy won't
permit morethan one to cross the river, what is the correct way to
cross the river??

* there are 7 targets, a b and c has to shoot them. all should be shooted
consicutively.
1. the no. of chances for a and b are not less thn 2,
2. for c there is only one chance.
3. a can't shot 3 times consicutively.
4. b is permited to shoot in even chances only.

they have given some 3or 4 questions on this

-----------------------------------------------------------------------------
------------------------------------------------------------------------------
section 3::

1.max value of signed int


a. b. c. d.
2.one questin is given, long one, to find the answer u should be
femiliar
with the operation as follows

int *num={10,1,5,22,90};
main()
{
int *p,*q;
int i;
p=num;
q=num+2;
i=*p++;
print the value of i, and q-p, and some other operations are there.
}
how the values will change??
3. one pointer diff is given like this:
int *(*p[10])(char *, char*)
asked to find the meaning.
4. char *a[4]={"jaya","mahe","chandra","buchi"};
what is the value of sizeof(a)/sizeof(char *)
a. 4 b.bytes for char c-- d.--
( we don't know the answer)

5. void fn(int *a, int *b)


{
int *t;
t=a;
a=b;
b=t;
}
main()
{
int a=2;
int b=3;
fn(&a,&b);
print the values os a and b;
}
what is the output--- out put won't swap, the same values remain.

a. error at runtime
b. compilation error
c.2 3
d. 3 2
6.
#define scanf "%s is a string"
main()
{
printf(scanf,scanf);
}
what is the output.

ans : %s is string is string

7. i=2+3,4>3,1;
printf("%d"i);

ans is 5 only.
8. char *p="abc";
char *q="abc123";

while(*p=*q)
{
print("%c %c",*p,*q);
}

a. aabbcc
b. aabbcc123
c. abcabc123
d. infinate loop ( this may be correct)
9. printf("%u",-1)
what is the value?
a. -1 b. 1 c. 65336 d. --
(maxint value-1 i think, check for the answer)

10. #define void int


int i=300;
void main(void)
{
int i=200;
{
int i=100;
print the value of i;
}
print the value of i
}
what is the output?

may be 100 200


11.

int x=2;
x=x<<2;
printf("%d ",x);

ans=8;
12.

int a[]={0,0x4,4,9}; /*some values are given*/

int i=2;

printf("%d %d",a[i],i[a]);

what is the value??? (may be error)

13.
some other program is given , i can't remember it
u can get it afterwads,

the answer is 3 3, so u can check this in the exam. itself.

i'll send the remaining two afterwars whenever i get them. ok !

-------------------------------------------------------------------------

*******************************************
mascot
*******************************************

this test contains 2 papers


paper1=aptitude-30min-44questions
contains 3 sections
paper2=computer skill-30min-60questions-6sections-10questions each
-------
paper1
-------
section1
---------
i do not remember the q's.simple quantitative apt. q's.
but takes more time ,do fast.
section2
--------
1-8 q's on bus route. a b c d e
a x 8 15 20 7
b 6 x 9 13 21
c 10 12 x 3 11
d 9 1 18 x 5
e 3 4 17 14 x
where x is starting point.a&e are first and last stations.and b,c,d
are intermediate stations. fig's are no. of passengers.cost of ticket
is 0.7rs /pass.
between any successive stations. based on this few q's were given.
the fig's
are not correct. q's like total no.of pass.in onward journey.
rest of q's are
2 statements were given. u have to answer they are correct or not
.littlebit
easy.
section3
--------
simple q's from r.s agarwal_quantitative apt.
1.1/10power18 - 1/10power20 .....value?
2.pipes-leaking-cisterns.
paper2
------------
1.general awareness.2.
1.father of computers
2.expand html,dma,fat,lan,wan,fddetc
3.intel's first micropro...a.pentium b.pentiumproetc
4.1024(dec)convert to hexa&octal
5.first micro.pro.a)8085b)8088etc
6.motorola's processor name?
7.windows_nt expand
8.simple programs on pascal&c
9.diff between 8087,8086 (which is latest vers.)
10.some basic q's on gui.
11.q's on ibmpc
12.one program on finding factorial
--------------------------------
i remember only these q's. i will ask my pals about rest of q's.i
want to know
when mascot is coming.they have taken 14 from here. most of them are
mech.&ime.
---------------------------------------------------

********************************************
tisl
********************************************

i know some 25 questions.

the technical comprises of 50 questions on c,unix and windows.


1.const char *
char * const
what is the differnce between the above tow?.
2.in unix inter process communication take place using?.
3.what are the files in /etc directory?.
4.about i-node numbers
5.max relaxable permisssion value with out giving write permission
to others?.
6.about ln(linking)
7.a question on until
until (who |grep mary)
do
sleep(60)
done
8.linking across directories?.

9.process id for kernell process


10.very first process created by kernell
11.function to repaint a window immediately?.
12.function entry for dll in win3.1
13.win 3.1 is a
14.win 3.1 supports which type of multi tasking?.
15.message displayed when a window is destroyed
16.about fork()?
17.about send message and post message
18.message to limit the size of window
19.system call executable binary file intoa process
20.about gdi object?.
21.api used to hide window
22.initialize contents of a dialog?.
_____________________________________________________________________

************************************
verifone
************************************

there will be 45 qstns. 15+5+5+5+5+5+5.


section: electronics:
1.a logic ckt is given and asked to identify the configuration.
ans: xor.
2.multi vibrator with nor gates is given
ans: astable multi sqr wave opt.
3.4 stage ripple counter with delay(f/f) 10msec. how much time it takes
for a state to change. 4*10=40.
4.impedence of a p'lel resonant circiut at resonance:r.
5.serial to parellel conversion is done by ans:shift register.
5.if the address bus id 20bits.then the memory space is 1mb.
6.filtering can be done with:capacitor,iductor,both,none.
7.the config that is worst effected by low cmmr
ans:non inverting amplifier.
8.two progs are given. one satrts counting frm 0 to max and the other
stars frm max to 0. which one executes fast.may be max to 0.think of.it
should be.

another 6 qstns are there. simple.

networks:
1.
the fctn of datalink layer is:bit stuffing.
2.which of the following is not fctn of datalink layer: encription.
3.voltage levels of rs232x:+12,-12.
4.which of the following is not used for client/server.
rpc,tcp/ip,messageqs none ans:may be none.
5.

database:
1. which of the following is true.
ans:the primary key in database design is very important.
2. sql is a non procedural query langauge.
3.
4
5.
compiler/algo/ds:
1.data structure used to impliment a menu: doubly linked circular linked
list.
2. some regular expression is given:waw'.may be it is context free grammar.
3.,4,5.
os:
1.the feature of real time os is: fast context swithing.
2.os impliments protection with the help of hardware(like virtual
addressing in 386/286 etc).

c:
some small c progs are given asked to tell the function/errors etc.
one of the qstn(last in the paper) is an invalid statement.
finish of all these very fast and think about those others. they are
simple only.

interview:
depends on the member.(mr.deep if he comes asks archirect of 586/486/386/286
etc. otherwise ds,os,c,tcp/ip . they hv taken 21(18btechs+3mtechs).
ok if u hv any queries ask immediately.
ok
all the best.
------------------ complete --------------------

part 1

1). a beggr collects cigarette stubs and makes one ful cigarette
with every 7 stubs. once he gets 49 stubs . how many cigarettes
can he smoke totally.
ans. 8
2). a soldiar looses his way in a thick jungle at random walks
from his camp but mathematically in an interestingg fashion.
first he walks one mile east then half mile to north. then 1/4
mile to west, then 1/8 mile to south and so on making a loop.
finally hoe far he is from his camp and in which direction.
ans: in north and south directions
1/2 - 1/8 + 1/32 - 1/128 + 1/512 - and so on
= 1/2/((1-(-1/4))
similarly in east and west directions
1- 1/4 + 1/16 - 1/64 + 1/256 - and so on
= 1/(( 1- ( - 1/4))
add both the answers
3). hoe 1000000000 can be written as a product of two factors
neither of them containing zeros
ans 2 power 9 x 5 ppower 9 ( check the answer )
4). conversation between two mathematcians:
first : i have three childern. thew pproduct of their ages is 36
. if you sum their ages . it is exactly same as my neighbour's
door number on my left. the sacond mathematiciaan verfies the
door number and says that the not sufficient . then the first
says " o.k one more clue is that my youngest is the youngest"
immmediately the second mathematician answers . can you aanswer
the questoion asked by the first mathematician?
what are the childeren ages? ans 2 and 3 and 6
5). light glows for every 13 seconds . how many times did it
between 1:57:58 and 3:20:47 am
ans : 383 + 1 = 384

6). 500 men are arranged in an array of 10 rows and 50 columns .


all tallest among each row aare asked to fall out . and the
shortest among them is a. similarly after resuming that to their
originaal podsitions that the shorteest among each column are
asked to fall out. and the longest among them is b . now who is
taller among a and b ?
ans a
7). a person spending out 1/3 for cloths , 1/5 of the remsaining
for food and 1/4 of the remaining for travelles is left with
rs 100/- . how he had in the begining ?
ans rs 250/-
8). there are six boxes containing 5 , 7 , 14 , 16 , 18 , 29
balls of either red or blue in colour. some boxes contain only
red balls and others contain only blue . one sales man sold one
box out of them and then he says " i have the same number of red
balls left out as that of blue ". which box is the one he solds
out ?
ans : total no of balls = 89 and (89-29 /2 = 60/2 = 30
and also 14 + 16 = 5 + 7 + 18 = 30
9). a chain is broken into three pieces of equal lenths
conttaining 3 links each. it is taken to a backsmith to join into
a single continuous one . how many links are to tobe opened to
make it ?
ans : 2.
10). grass in lawn grows equally thickand in a uniform rate. it
takes 24 days for 70 cows and 60 for 30 cows . how many cows can
eat away the same in 96 days.?
ans : 18 or 19
11). there is a certain four digit number whose fourth digit is
twise the first digit.
third digit is three more than second digit.
sum of the first and fourth digits twise the third number.
what was that number ?
ans : 2034 and 4368

if you qualify in the first part then you have to appear for
the second i.e the following part.

part 2.

1. from a vessel on the first day, 1/3rd of the liquid


evaporates. on the second day 3/4th of the remaining liquid
evaporates. what fraction of the volume is present at the end of
the ii day.
2. an orange galss has orange juice. and white glass has apple
juice. bothe equal volume 50ml of the orange juice is taken and
poured into the apple juice. 50ml from the white glass is poured
into the orange glass. of the two quantities, the amount of
apple juice in the orange glass and the amount of orange juice in
the white glass, which one is greater and by how much?
3. there is a 4 inch cube painted on all sides. this is cut
into no of 1 inch cubes. what is the no of cubes which have no
pointed sides.
4. sam and mala have a conversation. sam says i am vertainly not
over 40. mala says i am 38 and you are atleast 5 years older
than me. now sam says you are atleast 39. all the sattements by
the two are false. how hold are they realy.
5. ram singh goes to his office in the city, every day from his
suburbun house. his driver mangaram drops him at the railway
station in the morning and picks him up in the evening. every
evening ram singh reaches the station at 5 o'clock. mangaram
also reaches at the same time. one day ramsingh started early
from his office and came to the station at 4 o'clock. not
wanting to wait for the car he starts walking home. mangaram
starts at normal time, picks him up on the way and takes him back
house, half an hour early. how much time did ram singh walk.
6. in a railway station, there are tow trains going. one in the
harbour line and one in the main line, each having a frequency of
10 minutes. the main line service starts at 5 o'clock. the
harbour line starts at 5.02a.m. a man goes to the station every
day to catch the first train. what is the probability of man
catchinhg the first train
7. some people went for vaction. unfortunately it rained for 13
days when they were there. but whenever it rained in the
morning, they had clean afternood and vice versa. in all they
enjoyed 11 morning and 12 afternoons. how many days did they
stay there totally
8. exalator problem repeat
9. a survey was taken among 100 people to firn their preference
of watching t.v. programmes. there are 3 channels. given no of

people who watch


at least channel 1
" " 2
" " 3
no channels at all
atleast channels 1and 3
" " 1 and 2
" " 2 and 3
find the no of people who watched all three.
10. albert and fernandes they have two leg swimming race. both
start from opposite and of the pool. on the first leg, the boys
pass each other at 18 mt from the deep end of the pool. during
the ii leg they pass at 10 mt from the shallow end of the pool.
both go at const speed. but one of them is faster. each boy
rests for 4 sec to see at the end of the i leg. what is the
length of the pool.
11. t h i s each alphabet stands for one
i s digit, what is the maximum value t
-------------- can take
x f x x
x x u x
--------------
x x n x x
--------------

1. an escalator is descending at constant speed. a walks down


and takes 50 steps to reach the bottom. b runs down and takes 90
steps in the same time as a takes 10 steps. how many steps are
visible when the escalator is not operating.
2. evvery day a cyclist meets a train at a particular crossing.
the road is straignt before the crossing and both are travelling
in the same direction. cyclist travels with a speed of 10 kmph.
one day the cyclist comes late by 25 min. and meets the train 5km
before the crossing. what is the seppd of the train.
3. five persons muckerjee, misra, iyer, patil and sharma, all
take then first or middle names in the full names. there are 4
persons having i or middle name of kumar, 3 persons with mohan, 2
persons withdev and 1 anil.
--either mukherjee and patil have a i or middle name of dev or
misra and iyer have their i or middle name of dev
--of mukherkjee and misre, either both of them have a first or
middle name of mohan or neither have a first or middle name of
mohan
--either iyer of sharma has a i or middle name of kumar hut not
both.
who has the i or middle name of anil
4. reading conprehension
5. a bird keeper has got ppigeon, m mynas and s sparrows. the
keeper goes for lunch leaving his assistant to watch the birds.
a. suppose p=10, m=5, s=8 when the bird keeper comes back, the
assistant informs the x birds have escaped. the bird keeper
exclaims oh no! all my sparrows are gone. how many birds flew
away.
b. when the bird keeper come back, the assistand told him that
x birds have escaped. the keeper realised that atleast2 sparrows
have escaped. what is minimum no of birds that can escape.
6. select from the five alternatives a,b,c,d,e
at the end of each question ,two conditions will be given.
the choices are to filled at follows.
a. if a definete conclusion can be drawn from condition 1
b. if a definete conclusion can be drawn from condition 2
c. if a definete conclusion can be drawn from condition 1 and 2
d. if a definete conclusion can be drawn from condition 1 or 2
e. no conclusion can be drawn using both conditions
1. person 1 says n<5
person says n>5
person 3 says 3n>20
person 4 says 3n>10
person 5 says n<8
what is value of n
a) 1. no of persons who speak false being less than no of persons
who tells the truth.
2. person 2 is telling the truth.
b) 1. no of persong telling the truth is greater than no of
persons telling lies
2. person 5 is telling the truth.
7. there are n coins on a table. there are two players a&b.
you can take 1 or 2 coins at a time. the person who takes the
last coin is the loser. a always starts first
--1. if n=7
a) a can always win by taking two coins in his first chanse
b) b can win only if a takes two coins in his first chance.
c) b can always win by proper play
d) none of the above
--2. a can win by proper play if n is equal to
a) 13 b) 37 c) 22 d) 34 e) 48 ans. e.
--3. b can win by proper play if n is equal to
a) 25 b)26 c) 32 d) 41 e) none
--4. if n<4, can a win by proper play always

8. two turns have vertain pecular charcteristics. one of them


always lies on monday, wednesday, friday. \the other always lies
on tuesdays, thursdays and saturdays. on the other days they tel
the truth. you are given a conversation.
person a-- today is sunday my name is anil
person b-- today is tuesday, my name is bill

answers for selected questions


2. equal 1. 150
3. 8 2. 60 kmph
4. 37(m),41(s) 3. mukherjee
5. 45 min. 8. today is tuesday
6. 0.8
7. 18
11. t max value = 4

3. anil kumar mukherjee


kumar misra dev
mohan iyer dev
kumar patil mohan
mohan sharma kumar
infosys
--------

1) there are two balls touching each other circumferencically.


the radius of the big ball is 4 times the diameter of the small
ball.the outer small ball rotates in anticlockwise direction
circumferencically over the bigger one at the rate of 16 rev/sec.
the bigger wheel also rotates anticlockwise at nrev/sec. what is
'n' for the horizontal line from the centre of small wheel always
is horizontal.

2) 1 2 3 4
+ 3 4 5 5
----------
4 6 8 9
- 2 3 4 5
----------
2 3 4 4
+ 1 2 5 4
------------
3 6 9 8

q) strike off any digit from each number in seven rows (need not
be at same place) and combine the same operations with 3 digit numbers
to get the same addition. after this strike off another digit from all
and add all the no.s to get the same 2 digit no. perform the same
process again with 1 digit no.s. give the ' no.s in 7 rows at
each stage.

3) there is a safe with a 5 digit no. the 4th digit is 4 greater than
second digit, while 3rd digit is 3 less than 2nd digit. the 1st digit
is thrice the last digit. there are 3 pairs whose sum is 11. find
the number. ans) 65292.

4) there are 2 guards bal and pal walking on the side of a wall of a
wearhouse(12m x 11m) in opposite directions. they meet at a point and
bal says to pal " see you again in the other side". after a few moments
of walking bal decides to go back for a smoke but he changes his
direction again to his previous one after 10 minutes of walking in
the other(opposite) direction remembering that pal will be waiting
for to meet.if bal and pal walk 8 and 11 feet respectively, how
much distance they would have travelled before meeting again.

5) xxx)xxxxx(xxx
3xx
-------
xxx
x3x
------
xxx
3xx
------
q) find the 5 digit no.
hint: 5 is used atleast once in the calculation.
6) afly is there 1 feet below the ceiling right across a wall length
is 30m at equal distance from both the ends. there is a spider 1 feet
above floor right across the long wall eqidistant from both the ends.
if the width of the room is 12m and 12m, what distance is to be
travelled by the spider to catch the fly? if it takes the shortest
path.

7) ramesh sit around a round table with some other men. he has one
rupee more than his right person and this person in turn has 1 rupee
more than the person to his right and so on, ramesh decided to give
1 rupee to his right & he in turn 2 rupees to his right and 3 rupees
to his right & so on. this process went on till a person has
'no money' to give to his right. at this time he has 4 times the
money to his right person. how many men are there along with ramesh
and what is the money with poorest fellow.

8)question related to probabilities of removing the red ball from a


basket,given that two balls are removed from the basket and the other
ball is red. the basket contains blue,red,yellow balls.

9)venkat has 1boy&2daughters.the product of these children age is 72.


the sum of their ages give the door numberof venkat.boy is elder of
three.can you tell the ages of all the three.

analytical
----------
1)l:says all of my other 4 friends have money
m:says that p said that exact one has money
n:says that l said that precisely two have money
o:says that m said that 3 of others have money.
p:land n said that they have money.
all are liers.who has money&who doesn't have?

2)a hotel has two,the east wing and the west wing.some east wing rooms
but not all have an ocean view(ov).all ww have a harbour view(hv).the
charge for all rooms is identical, except as follows
* extra charge for all hv rooms on or above the 3rd floor
* extra charge for all ov rooms except those without balcony
* extra charge for some hv rooms on the first two floor&some ew rooms
without ov but having kitchen facilities. (gre modrl test 3-question
1j-22)
3)post man has a data of name surname door no.pet name of 4 families.
but only one is correct for each family.there are a set of statements
&questions.
4)4 couples have a party.depending on the set of statements,find who
insulted whom and who is the host of the party.
5)5 women given some of their heights(tall,medium,short)hair( long,
plainted),stards(black or brown), sari,2 medium,2-short.tall->no
sari.plainted->medium.answer the combinations.

1) a person has to go both northwards&southwards in search of a job.


he decides to go by the first train he encounters.there are trains for
every 15 min both southwards and northwards.first train towards south
is at 6:00 a.m. and that towards north is at 6:10 .if the person arrives
at any random time,what is the probability that he gets into a train
towards north.
2) a person has his own coach&whenever he goes to railway station he
takes his coach.one day he was supposed to reach the railway station
at 5 o'clock.but he finished his work early and reached at 3 o'clock.
then he rung up his residence and asked to send the coach immediately.
he came to know that the coach has left just now to tje railway station.
he thought that the coach has left just now to the railway station.he
thought that he should not waste his time and started moving towards
his residence at the speed of 3mi/hr.on the way,he gets the coach and
reaches home at 6 o'clock.how far is his residence from railway
station.

3)radha,geeta&revathi went for a picnic.after a few days they forgot the


date,day and month on which they went to picnic.radha said that it was
onthursday,may 8 and geeta said that it was thursday may 10.revathi
said friday jun 8.now one of them told all things wrongly,others one
thing wrong and the last two things wrongly.if april 1st is tuesday
what is the right day,date and month?

> dear shankar, mani,


> here are the infosys questions which appeared here., they had
> five sets of que. papers a,b,c,d,e. this is set a. others
> are also of similar type with few modifications.
> love-kalya.
>
>
> infosys
>
>
>
> > there are 5 q papers with them. this is only one of those.
> > all otheres were more or less of the same model but different
> > qs. this is just to give u an idea.
> >
> > the questions are not in order.
> >
> > 1)a,b,c,d,e related.4 of them made these statements each.
> > i)c is my son in law's brother.
> > ii)b is my father's brother.
> > iii)e is my mother in law.
> > iv)a is my brother's wife.
> > who made these statements?(person mentioned is one of a,b,c,d,e)(10 mar
ks).
> > 2)e means belong.
> > all members of e e d.
> > all members of d e a.
> > not all members of d e e.
> > not all members of a e d.
> > all members of c e both a and b.some questions are asked about relatio
n.use venn diagram.(5 marks).
> >
> > 3)complete the table.
> >
> > played won lost draw goals goals
> > for against
> > a 2 2 1
> > b 2 1 2 4
> > c 2 3 7
> >
> > a,b,c are 3 hockey teams.(2 marks).
> >
> > 4) a says party was held on :thursday ,may 8th.
> > b says party was held on :tuesday,may 10th.
> > c says party was held on :friday ,june 8th.
> > given april 1 st was tuesday.one of a,b,c says 1 correct.one says 1
> > wrong.and one was completely wrong of date,month and day. find the
> > day the party held. (5marks).
> >
> > 5) a ship is away from the shore by 180 miles.a plane is travelling at
> > 10 times speed of the ship.how long from the shore will they meet?
> > (2marks)
> >
> > 6) every station in n railroad issue everyother station's ticket.
> > some stations are added.now they have to issue 46 more tickets.
> > say the no.of stations after and before added.(5 marks).
> >
> > 7) 3 persons say these statements.
> > a says either democratic or liberal wins the elections.
> > b says democratic wins.c says neither democratic nor liberal wins
> > the election.of these only one is wrong.who wins the election?
> > (5 marks).
> >
> > 8) a clock showing 6 o'clock takes 30 secs to strike 6 times.how long
> > will it take to strike 12 at midnight?ans.66 secs.(2marks)
> >
> > 9) only boys aged > 16 wear coats.
> > boys aged > 15 go to watch football.some more statements are given.
> > what can be said about those who are watching football ? (age and
> > costume)
> > (5 marks).
> >
> > 10) there are 3 societies a,b,c having some tractors each.
> > a gives b and c as many tractors as they already have.
> > after some days b gives a and c as many tractors as they have.
> > after some days c gives a and b as many tractors as they have.
> > finally each has 24 tractors.what is the original no.of
> > tractors each had in the beginning?
> > ans.a -39.
> > b- 21.
> > c- 12.(7 marks).
> > 11) 4,5 statements.from that find the answer.(7 marks).
> >
> >
> > reference books
> > 1.) puzzles and teasers by summer's
> > 2.) shakuntala devi. (puzzles).
> >
>
>
> --

from: oepg9627 (pullabhotla srinivas)


message-id: <9709141152.aa53207@vanavil>
to: mepg9618
status: r

ramco 'c' question paper

1).
--------------------------------------------------------------

main()
{
char *p1="name";
char *p2;
p2=(char *)malloc(20);
while(*p2++=*p1++);
printf("%s\n",p2);
}

ans : an empty string

2).
--------------------------------------------------------------

main()
{
int x=20,y=35;
x = y++ + x++;
y = ++y + ++x;
printf("%d %d\n",x,y);
}

ans 57 94

3).
--------------------------------------------------------------

main()
{
int x=5;
printf("%d %d %d\n",x,x<<2,x>>2);
}

ans 5 20 1
4).
-----------------------------------------------------------------
#define swap1(a,b) a=a+b;b=a-b;a=a-b;
main()
{
int x=5,y=10;
swap1(x,y);
printf("%d %d\n",x,y);
swap2(x,y);
printf("%d %d\n",x,y);
}

int swap2(int a,int b)


{
int temp;
temp=a;
b=a;
a=temp;
return;
}

ans 10 5
10 5

5).
--------------------------------------------------------------

main()
{
char *ptr = "ramco systems";
(*ptr)++;
printf("%s\n",ptr);
ptr++;
printf("%s\n",ptr);
}
ans samco systems
amco systems

6).
--------------------------------------------------------------

#include<stdio.h>
main()
{
char s1[]="ramco";
char s2[]="systems";
s1=s2;
printf("%s",s1);
}

ans compilation error giving it cannot be an modifible 'lvalue'

7).
--------------------------------------------------------------

#include<stdio.h>
main()
{
char *p1;
char *p2;
p1=(char *) malloc(25);
p2=(char *) malloc(25);
strcpy(p1,"ramco");
strcpy(p2,"systems");
strcat(p1,p2);
printf("%s",p1);
}

ans : ramcosystems

8).
--------------------------------------------------------------

[1]. the following variable is available in file1.c

static int average_float;

ans all the functions in the file1.c can access the variable

9).
--------------------------------------------------------------

ans : [2]. extern int x;

check the answer

10).
--------------------------------------------------------------
[3]. another problem with

# define true 0

some code

while(true)
{
some code

}
this won't go into the loop as true is defined as 0
ans none of the above i.e d

11).
--------------------------------------------------------------
ans : [4]. a question in structures where the memebers are dd,mm,yy.

mm:dd:yy
09:07:97

12).
--------------------------------------------------------------
ans : [5]. another structure question

1 rajiv system analyst

13).
--------------------------------------------------------------

answer

infile.dat is copied to outfile.dat

14).
--------------------------------------------------------------

a question with argc and argv .

input will be

c:\temp.exe ramco systems india

output will be

india: i n d i a
systems: s y s t e m s
ramco: r a m c o

answer is choice d

15).
--------------------------------------------------------------

structure swap

ramco india
ramco systems corporation
ramco ... limited .

after swapping the result will be

first two will be swapped.

ramco systems corporation


ramco india
ramco ... limited .

16).
--------------------------------------------------------------

int x;
main()
{
int x=0;
{
int x=10;
x++;
change_value(x);
x++;
modify_value();
printf("first output: %d\n",x);
}
x++;
change_value(x);
printf("second output : %d\n",x);
modify_value();
printf("third output : %d\n",x);
}

modify_value()
{
return (x+=10);
}

change_value()
{
return(x+=1);
}

ans : 12 1 1

17).
--------------------------------------------------------------
main()
{
int x=10,y=15;
x=x++;
y=++y;
printf("%d %d\n",x,y);
}

ans : 11 16

18).
--------------------------------------------------------------

main()
{
int a=0;
if(a=0) printf("ramco systems\n");
printf("ramco systems\n");
}

ans : ony one time


"ramco systems"
will be printed

------------------ over ------------------

/* ------------------- ramqp -------------------------*/


/*

1) a - g are 7 consecutive +ve integers not necessarily in the same order

1) b is the middle number


2) d is 3 less than c
3) the difference between f & a is equal in magnitude and sign
to the difference between e & c
4) neither f nor c lie between e & g

a) what is the value of b-f

1 2 -1 -2 cannot be determined

b) which is greatest

f c a e cannot be determined

c) given both a & b are primes what is the lowest value of e

8 6 9 12 cannot
2) given that a,b,c,d,e each represent one of the digits between
1-9 and that the following multiplication holds

a b c d e
4
----------
e d c b a

what digit does e represent

a) 4
b) 6
c) 7
d) 8
e) none

1. how many butes does an array a(1:8,-2:2,1:5) require for storage if


each element of the array is 24 bits long.

200 480 600 800 none

2. begin

i:=0;
j:=0; | block d

loop:

if(i != 0)
i := i-1;
else
i := i+1;

i := i+1; | block a
j := j+1; | block b

if (j <= 25)
goto loop;

end | block c

a) what is the value of i at [c]


2 ?
b) how many times is the goto executed
25 ?
c) how many times is the loop executed if i is initialized to 1
in [d] 26
d) how many times is the loop entered if the block [b] is changed
to j=j+1 ?

e) what is the value of i at [c] interchanging blocks [a] and [b] ?


2 ?

follow the instructions given below [ from 1 to 8 ]


1. a cause b or c but not both

2. f occurs only if b occurs

3. d occurs if b or c occurs

4. e occurs if only c occurs

5. j occurs only if e or f occurs

6. h occurs if e occurs

7. d causes g, h or both.

8. g occurs if f occurs.

questions
---------

1. if a occurs which of the following may occur

1. f & g (ii) e & h (iii) d

ans
---
(a) 1 only (b) 2 only (c) 3 only (d) 1,2,3 or 2 & 3 but not 1

(e) 1,2 & 3

2. if b occurs which must occur

ans
--- (a) f & g (b) d & g (c) d (d) g & h (e) j

3. if j occurs which must occur

ans
---
(a) e (b) both e & f (c) either b or c (d) b (e) both b & c

4. which may occur as a result by a cause not mentioned.

(i) d (ii) a (iii) f

ans
---
(a) i only (b) ii (c) i & ii (d) ii & iii (e) i,ii,iii

5. if e occurs which cannot occur.

(a) f (b) a (c) d (d) c (e) j


#include<stdio.h>
int sumelement(int *,int);
void main(void)
{
int x[10];
int i=10;
for(;i;)
{
i--;
*(x+i)=i;
}
printf("%d",sumelement(x,10));
}
int sumelement(int array[],int size)
{
int i=0;
float sum=0;
for(;i<size;i++)
sum+=array[i];
return sum;
}

#include<stdio.h>
void main(void);
int printf(const char*,...);
void main(void)
{
int i=100,j=10,k=20;
int sum;
float ave;
char myformat[]="ave=%.2f";
sum=i+j+k;
ave=sum/3.0;
printf(myformat,ave);
}

#include<stdio.h>
void main(void);
void main(void)
{
int a[10];
printf("%d",((a+9) + (a+1)));
}

#include<stdio.h>
void main(void);
void main(void)
{
struct s{
int x;
float y;
}s1={25,45.00};
union u{
int x;
float y;
} u1;
u1=(union u)s1;
printf("%d and %f",u1.x,u1.y);
}

#include<stdio.h>
void main(void)
{
unsigned int c;
unsigned x=0x3;
scanf("%u",&c);
switch(c&x)
{
case 3: printf("hello!\t");
case 2: printf("welcome\t");
case 1: printf("to all\t");
default:printf("\n");
}
}

#include<stdio.h>
int fn(void);
void print(int,int(*)());
int i=10;
void main(void)
{
int i=20;
print(i,fn);
}
void print(int i,int (*fn1)())
{
printf("%d\n",(*fn1)());
}

int fn(void)
{
return(i-=5);
}

#include<stdio.h>
void main(void);
void main(void)
{
char numbers[5][6]={"zero","one","two","three","four"};
printf("%s is %c",&numbers[4][0],numbers[0][0]);
}

int bags[5]={20,5,20,3,20};
void main(void)
{
int pos=5,*next();
*next()=pos;
printf("%d %d %d",pos,*next(),bags[0]);
}
int *next()
{
int i;
for(i=0;i<5;i++)
if (bags[i]==20)
return(bags+i);
printf("error!");
exit(0);
}

#include<stdio.h>
void main(void)
{
int y,z;
int x=y=z=10;
int f=x;
float ans=0.0;
f *=x*y;
ans=x/3.0+y/3;
printf("%d %.2f",f,ans);
}

#include<stdio.h>
void main(void);
double dbl=20.4530,d=4.5710,dblvar3;
void main(void)
{
double dbln(void);
dblvar3=dbln();
printf("%.2f\t%.2f\t%.2f\n",dbl,d,dblvar3);
}
double dbln(void)
{
double dblvar3;
dbl=dblvar3=4.5;
return(dbl+d+dblvar3);
}

#include<stdio.h>
static int i=5;
void main(void)
{
int sum=0;
do
{
sum+=(1/i);
}while(0<i--);
}

#include<stdio.h>
void main(void)
{
int oldvar=25,newvar=-25;
int swap(int,int);
swap(oldvar,newvar);
printf("numbers are %d\t%d",newvar,oldvar);
}
int swap(int oldval,int newval)
{
int tempval=oldval;
oldval=newval;
newval=tempval;
}

#include<stdio.h>
void main(void);
void main(void)
{
int i=100,j=20;
i++=j;
i*=j;
printf("%d\t%d\n",i,j);
}

#include<stdio.h>
void main(void);
int newval(int);
void main(void)
{
int ia[]={12,24,45,0};
int i;
int sum=0;
for(i=0;ia[i];i++)
{
sum+=newval(ia[i]);
}
printf("sum= %d",sum);
}
int newval(int x)
{
static int div=1;
return(x/div++);
}

#include<stdio.h>
void main(void);
void main(void)
{
int var1,var2,var3,minmax;
var1=5;
var2=5;
var3=6;
minmax=(var1>var2)?(var1>var3)?var1:var3:(var2>var3)?var2:var3;
printf("%d\n",minmax);
}

#include<stdio.h>
void main(void);
void main(void)
{
void pa(int *a,int n);
int arr[5]={5,4,3,2,1};
pa(arr,5);
}

void pa(int *a,int n)


{
int i;
for(i=0;i<n;i++)
printf("%d\n",*(a++)+i);
}

#include<stdio.h>
void main(void);
void print(void);
void main(void)
{
print();
}
void f1(void)
{
printf("\nf1():");
}

#include "6.c"
void print(void)
{
extern void f1(void);
f1();
}
static void f1(void)
{
printf("\n static f1().");
}

#include<stdio.h>
void main(void);
static int i=50;
int print(int i);
void main(void)
{
static int i=100;
while(print(i))
{
printf("%d\n",i);
i--;
}
}
int print(int x)
{
static int i=2;
return(i--);
}

#include<stdio.h>
void main(void);
typedef struct ntype
{
int i;
char c;
long x;
} newtype;
void main(void)
{
newtype *c;
c=(newtype *)malloc(sizeof(newtype));
c->i=100;
c->c='c';
(*c).x=100l;
printf("(%d,%c,%4ld)",c->i,c->c,c->x);
}

#include<stdio.h>
void main(void);
const int k=100;
void main(void)
{
int a[100];
int sum=0;
for(k=0;k<100;k++)
*(a+k)=k;
sum+=a[--k];
printf("%d",sum);
}

/* baan

paper 1 completely from r.s.agarwal


1 ---> analogy 1c(1-20)[pg 15-17]
2----> directions sense test 7b[1-10]
3---->logic deduction 17b[30-50]
4----> mathematical modelling 15a[1-20]

technical paper is same for all papers

peper 2
analogy from gre book
logic model test 1

page q.no new gre


406 1-4 (motorist prob)
407 17-22 (letters a,b,c)
508 1-4 (all g's are h's)
381 40 -41 (president prob)
377 7-11 (office staff)

part 2
problems on ages
part 3
blood relations

series problems

6,9,14,21,(30)
5,2,(2.5),8
2,10,(30),68,130,(222)
9,15,23,33,(45)
6,25,62,123,314,241
5,11,19,29,(41),55
2,12,30,56,90,(132)
4,18,52,(17)
39,21,(45),93
1,3,7,(15)31
3,9,21,(45),93
35,24,15,8,(3)
2,12,30,56,90,(132)
3,11,19,29,(39),51

analogy

fans:bleachers:: gre389
archipilago:islands::gre393
crow:boastful 393
bracket:shelf 394
taxonomy:classification 394
moderator:debate 413
glossary:words 413
lumber: bear 414
celerity:snail 414
wood:sand 454

carpenter :saw 87
horns:bull 87
gullible: 87
marathon :: (13th gre 87p)
skin : man ::
kick : football :: betal :chew (pno.45, rs 46)
bamboo: shoot :: bean : sprout
deflect : missile : distract : attraction
editor : magazine :: director : film
pine : clock :: calander :date
volcano : lava :: fault : earthquate (eyes : tears)
hero : accuhade :: hanging stork : ridicle
agitator : firebrand :: renegade ; turncoat
burst : sound :: tinder : fire
star : cluster :: tree : clump
piston : cylinger :: elevator : shaft
mitigate : punishment :: commute : sentence
erudite : scholar :: illeterate : ignorant
fire : ashes :: explosion : debris (11 rs 11th)
mason : wall :: author : book (87p gre )
fire : ashes :: event : memories

section ii coding

load == mpbe drive = esjwf


how will you code laddler -> mbeemfs

2) start =walka budpi = xzfmr

what would be stupid = wazmrf

from 3 to 7

a=z, b=y, c=x, .......... z=a

3) limit = ornrg
4) sour = hlfi
5) pocket = klxpug
6) group = tilfk
7) zerd = avil

from 8 to 9
here each letter is coded a s

a = (bc)d , b=(cd ) e, c=(de)f

8) shoot ---- vkrrw


9) vwduw ------ yzgxz
10) if dbmdvuub = calcutta
how will be bombay = ? (anlazx)
11) delhi =
cd********************************************************************************
**********************************************************************************
**********************************************************************************
**********************************************************************************
**********************************************************************************
**********************************************************************************
**********************13, o = 15 then deaf = ? (16)

--------------------------

technical :

1. binary equivalent of 52 ---- 110100


2. hexadecimal equivalent of 3452 ------ 72a
3.consider the boolean table
ans : a) 1

4. consider the circuit

ans a) (a+b).(c+b)
5. just in time concept address ----
ans : elimination of waste by purchasing manufacturing exactly when needed
6. a better way of unit testing s/w program is
ans : user test
7.a lowest level of security by most rdbms are
ans : a) field low
8. oot uses
ans : encapsulated of detect methods
9.edi useful in
ans : electronic transmission
10. mrpii different from mrp
ans : modular version of man redundant initials
11. hard disk time for r/w head to move to correct
and : a) lactency
12. percentage of times a page number bound in associate register
ans : bit ratio
13. modem --------modulation and demodulation
14. rdbms file system
ans : interrelated
15. super key is ----- primary key+ attribute
16. transmission is said to be ----committed
17. windows 95 supports
a) multiuser b) n tasks c) both
ans : c
18. difference between printf and fprintf ----
19.to change permission r&w to owner group to no permission to others
a) chmod 614 b) chmod 604

c) chmod 640 d) chmod 310


20. in batch process ------------better job

1) one rectangular plate with length 8inches,breadth 11 inches and


2 inches thickness is there.what is the length of the circular rod
with diameter 8 inches and equal to volume of rectangular plate?
ans: 3.5inches
2) what is the number of zeros at the end of the product of the numbers
from 1 to 100
3) in some game 139 members have participated every time one fellow will
get bye what is the number of matches to choose the champion to be
held?
ans: 138
4) one fast typist type some matter in 2hr and
another slow typist type the
same matter in 3hr. if both do combinely in how much time they
will finish.
ans: 1hr 12min
5) in 8*8 chess board what is the total number of squares
refer odel
ans:204
6) falling height is proportional to square of the time.
one object falls 64cm in 2sec than in 6sec from how much
height the object will fall.
7) gavaskar average in first 50 innings was 50 . after the 51st
innings his average was 51 how many runs he made in the 51st
innings
8)2 oranges,3 bananas and 4 apples cost rs.15 . 3 ornages 2 bananas
1 apple costs rs 10. what is the cost of 3 oranges, 3 bananas and
3 apples ans rs 15.
9)in 80 coins one coin is counterfiet what is minimum number of
weighings to find out counterfiet coin
10)in a company 30% are supervisors and 40% employees are male
if 60% of supervisors are male. what is the probability
that a randomly choosen employee is a male or female?
11)statement: all green are blue are blue, all blue are white
conclusion:
i) some blue are green ii) some white are green
iii)some green are not white iv) all white are blue
a) he has given four choices like gre type
12)all teachers are students. some students are girls.
this type of questions are there.
we cant able to reproduce them.:wq

1.while((*p++=*q++)!=0){}
is equal to
expl: while((*p++=*q++)!='\0'){}
a) b) c) d)

2.the function strcmp(str1,str2) returns


ans: int

3. int *x[](); means


expl: elments of an array can't be functions.

4.#define print(int) printf("int=%d",int);


main()
{int x,y,z;
x=03;y=-1;z=01;
print(x^x);
z<<=3;print(x); ----> i think here may be (z). for this ans=8.
if (x) ans ix =3.
y>>=3;print(y);
}
expl: 0,8,-1 (if second is z)

5. struct list{
int x;
struct list *next;
}*head;
the struct head.x =100
above is correct / wrong

expl: before using the ptr type struct variable we have to give memory
to that .
and also when ever the struct variable is ptr then we access the members
by "->" operator.

6. '-'=45 '/'=47
printfr(%d/n,'-','-','-','-','/','/','/');
o/p =?
ans: 45 ( i.e it takes first argument in
printf function.

12.o/p=?
int i;
i=1;
i=i+2*i++;
printf(%d,i);
************************************************************************
**********************************************************************************
**********************************************************************************
**********************************************************************************
**********************************************************************************
**********************************************************************************
*******************************pointer;
b=c;
printf("a=%d b=%d",a,b);
a. a=1 b=3
b a=3 b=3
c 3 2
d. error
ans: d . because ";" indicates
competion
of that statement. so it give
error.

* imp: and in above program " a=c/*pointer " statement


it considering as starting of comment statement.
so , it also causing for syntax error.

11.#include<malloc.h>
char *f()
{char *s=malloc(8);
strcpy(s,"goodbye")}
main()
{
char *f();
printf("%c",*f()='a');
o/p=?
ans: prints " a "

13. int sum(n)


int n;
if(n<1)return n;
else return(n+sum(n-1))
a 10 b 16 c 14 d 15
ans: if we take n=5 then ans is 15.

14. when a function is recursively called all ,


automatic variables are a. stored in stack b . c. d

ans: (a)

15) #define man(x,y) (x)>(y)?(x):(y)


{ int i=10;j=5;k=0;
k= max(i++,++j)
printf(%d %d %d %d,i,j,k)}
16) a=10;b=5; c=3;d=3;
if(a<b)&&(c=d++)
printf(%d %d %d %d a,b,c,d)
else printf("%d %d %d %d a,b,c,d);

: .............................................
19. what is o/p
#include<stdarg.h>
show(int t,va_list ptr1)
{
int a,x,i;
a=va_arg(ptr1,int)
printf("\n %d",a)
}
display(char)
{int x;
listptr;
va_star(otr,s);
n=va_arg(ptr,int);
show(x,ptr);
}
main()
{
display("hello",4,12,13,14,44);
}
a) 13 b) 12 c) 44 d) 14
.............................................

17. if the following program (my prog)


main(int size of ,char *arg[])
{ while(size of arg) printf("%s",arg[--size of arg)
}
is run from the command line as myprog jan feb mar apr
what would be the o/p
a)myprog jan,feb,mar,apr
b)rev
c)jan,feb,mar,apr
d)error
.............................................

18.what is o/p
main()
{int i=3;
while(i--)
{
int i=100
i--;
printf("%d..",i);
}
}
a) infinite loop
b) error
c) 99..99..99..99
d) 3..22..1..
.............................................
20)what is the o/p of the program
main()
{
int rows=3,colums=4;
int a[rows][colums]={1,2,3,4,5,6,7,8,9,10,11,12};
i=j=k=99;
for(i=0;i<rows;i++)
for(j=0;j<colums;j++)
if(a[k][j]<k) k=a[i][j];

printf("%d\n",k);
........................................................
~

wish you good luck


yours venu

to
dear venku, iitb.

1) one rectangular plate with length 8inches,breadth 11 inches and


2 inches thickness is there.what is the length of the circular rod
with diameter 8 inches and equal to volume of rectangular plate?
ans: 3.5inches
2) what is the number of zeros at the end of the product of the numbers
from 1 to 100
3) in some game 139 members have participated every time one fellow will
get bye what is the number of matches to choose the champion to be
held?
ans: 138
4) one fast typist type some matter in 2hr and
another slow typist type the
same matter in 3hr. if both do combinely in how much time they
will finish.
ans: 1hr 12min
5) in 8*8 chess board what is the total number of squares
refer odel
ans:204
6) falling height is proportional to square of the time.
one object falls 64cm in 2sec than in 6sec from how much
height the object will fall.
7) gavaskar average in first 50 innings was 50 . after the 51st
innings his average was 51 how many runs he made in the 51st
innings
8)2 oranges,3 bananas and 4 apples cost rs.15 . 3 ornages 2 bananas
1 apple costs rs 10. what is the cost of 3 oranges, 3 bananas and
3 apples ans rs 15.
9)in 80 coins one coin is counterfiet what is minimum number of
weighings to find out counterfiet coin
10)in a company 30% are supervisors and 40% employees are male
if 60% of supervisors are male. what is the probability
that a randomly choosen employee is a male or female?
11)statement: all green are blue are blue, all blue are white
conclusion:
i) some blue are green ii) some white are green
iii)some green are not white iv) all white are blue
a) he has given four choices like gre type
12)all teachers are students. some students are girls.
this type of questions are there.
we cant able to reproduce them.:wq

1.while((*p++=*q++)!=0){}
is equal to
expl: while((*p++=*q++)!='\0'){}
a) b) c) d)

2.the function strcmp(str1,str2) returns


ans: int

3. int *x[](); means


expl: elments of an array can't be functions.

4.#define print(int) printf("int=%d",int);


main()
{int x,y,z;
x=03;y=-1;z=01;
print(x^x);
z<<=3;print(x); ----> i think here may be (z). for this ans=8.
if (x) ans ix =3.
y>>=3;print(y);
}
expl: 0,8,-1 (if second is z)
5. struct list{
int x;
struct list *next;
}*head;
the struct head.x =100
above is correct / wrong

expl: before using the ptr type struct variable we have to give memory
to that .
and also when ever the struct variable is ptr then we access the members
by "->" operator.

6. '-'=45 '/'=47
printfr(%d/n,'-','-','-','-','/','/','/');
o/p =?
ans: 45 ( i.e it takes first argument in
printf function.

12.o/p=?
int i;
i=1;
i=i+2*i++;
printf(%d,i);
************************************************************************
**********************************************************************************
**********************************************************************************
**********************************************************************************
**********************************************************************************
**********************************************************************************
*******************************pointer;
b=c;
printf("a=%d b=%d",a,b);
a. a=1 b=3
b a=3 b=3
c 3 2
d. error
ans: d . because ";" indicates
competion
of that statement. so it give
error.

* imp: and in above program " a=c/*pointer " statement


it considering as starting of comment statement.
so , it also causing for syntax error.

11.#include<malloc.h>
char *f()
{char *s=malloc(8);
strcpy(s,"goodbye")}
main()
{
char *f();
printf("%c",*f()='a');
o/p=?
ans: prints " a "

13. int sum(n)


int n;
if(n<1)return n;
else return(n+sum(n-1))
a 10 b 16 c 14 d 15
ans: if we take n=5 then ans is 15.

14. when a function is recursively called all ,


automatic variables are a. stored in stack b . c. d

ans: (a)

15) #define man(x,y) (x)>(y)?(x):(y)


{ int i=10;j=5;k=0;
k= max(i++,++j)
printf(%d %d %d %d,i,j,k)}
16) a=10;b=5; c=3;d=3;
if(a<b)&&(c=d++)
printf(%d %d %d %d a,b,c,d)
else printf("%d %d %d %d a,b,c,d);

: .............................................
19. what is o/p
#include<stdarg.h>
show(int t,va_list ptr1)
{
int a,x,i;
a=va_arg(ptr1,int)
printf("\n %d",a)
}
display(char)
{int x;
listptr;
va_star(otr,s);
n=va_arg(ptr,int);
show(x,ptr);
}
main()
{
display("hello",4,12,13,14,44);
}
a) 13 b) 12 c) 44 d) 14
.............................................

17. if the following program (my prog)


main(int size of ,char *arg[])
{ while(size of arg) printf("%s",arg[--size of arg)
}
is run from the command line as myprog jan feb mar apr
what would be the o/p
a)myprog jan,feb,mar,apr
b)rev
c)jan,feb,mar,apr
d)error
.............................................

18.what is o/p
main()
{int i=3;
while(i--)
{
int i=100
i--;
printf("%d..",i);
}
}
a) infinite loop
b) error
c) 99..99..99..99
d) 3..22..1..
.............................................
20)what is the o/p of the program
main()
{
int rows=3,colums=4;
int a[rows][colums]={1,2,3,4,5,6,7,8,9,10,11,12};
i=j=k=99;
for(i=0;i<rows;i++)
for(j=0;j<colums;j++)
if(a[k][j]<k) k=a[i][j];

printf("%d\n",k);
........................................................
~

wish you good luck


yours venu

to
dear venku, iitb.

/* baan

paper 1 completely from r.s.agarwal


1 ---> analogy 1c(1-20)[pg 15-17]
2----> directions sense test 7b[1-10]
3---->logic deduction 17b[30-50]
4----> mathematical modelling 15a[1-20]
technical paper is same for all papers

peper 2
analogy from gre book

logic model test 1

page q.no new gre


406 1-4 (motorist prob)
407 17-22 (letters a,b,c)
508 1-4 (all g's are h's)
381 40 -41 (president prob)
377 7-11 (office staff)

part 2
problems on ages
part 3
blood relations

series problems

6,9,14,21,(30)
5,2,(2.5),8
2,10,(30),68,130,(222)
9,15,23,33,(45)
6,25,62,123,314,241
5,11,19,29,(41),55
2,12,30,56,90,(132)
4,18,52,(17)
39,21,(45),93
1,3,7,(15)31
3,9,21,(45),93
35,24,15,8,(3)
2,12,30,56,90,(132)
3,11,19,29,(39),51

analogy

fans:bleachers:: gre389
archipilago:islands::gre393
crow:boastful 393
bracket:shelf 394
taxonomy:classification 394
moderator:debate 413
glossary:words 413
lumber: bear 414
celerity:snail 414
wood:sand 454

carpenter :saw 87
horns:bull 87
gullible: 87
marathon :: (13th gre 87p)
skin : man ::
kick : football :: betal :chew (pno.45, rs 46)
bamboo: shoot :: bean : sprout
deflect : missile : distract : attraction
editor : magazine :: director : film
pine : clock :: calander :date
volcano : lava :: fault : earthquate (eyes : tears)
hero : accuhade :: hanging stork : ridicle
agitator : firebrand :: renegade ; turncoat
burst : sound :: tinder : fire
star : cluster :: tree : clump
piston : cylinger :: elevator : shaft
mitigate : punishment :: commute : sentence
erudite : scholar :: illeterate : ignorant
fire : ashes :: explosion : debris (11 rs 11th)
mason : wall :: author : book (87p gre )
fire : ashes :: event : memories

section ii coding

load == mpbe drive = esjwf


how will you code laddler -> mbeemfs

2) start =walka budpi = xzfmr

what would be stupid = wazmrf

from 3 to 7

a=z, b=y, c=x, .......... z=a

3) limit = ornrg
4) sour = hlfi
5) pocket = klxpug
6) group = tilfk
7) zerd = avil

from 8 to 9
here each letter is coded a s

a = (bc)d , b=(cd ) e, c=(de)f

8) shoot ---- vkrrw


9) vwduw ------ yzgxz
10) if dbmdvuub = calcutta
how will be bombay = ? (anlazx)
11) delhi =
cd????????????????????????????????????????????????????????????????????????????????
????????????????????????????????????????????????book on microprocessor)
> > 24. qn on pipeline architecture
> > 25 qn on lapb protocol
> >
> >
>

###############???????????????????????????????????????????????????????????????????
??????????????????????????????????????????????????????????????????????????????????
??????????4???4??????????????????4??????4????????????????????????????????????????�
?????????????????????????13, o = 15 then deaf = ? (16)

--------------------------
technical :

1. binary equivalent of 52 ---- 110100


2. hexadecimal equivalent of 3452 ------ 72a
3.consider the boolean table

ans : a) 1

4. consider the circuit

ans a) (a+b).(c+b)
5. just in time concept address ----
ans : elimination of waste by purchasing manufacturing exactly when needed
6. a better way of unit testing s/w program is
ans : user test
7.a lowest level of security by most rdbms are
ans : a) field low
8. oot uses
ans : encapsulated of detect methods
9.edi useful in
ans : electronic transmission
10. mrpii different from mrp
ans : modular version of man redundant initials
11. hard disk time for r/w head to move to correct
and : a) lactency
12. percentage of times a page number bound in associate register
ans : bit ratio
13. modem --------modulation and demodulation
14. rdbms file system
ans : interrelated
15. super key is ----- primary key+ attribute
16. transmission is said to be ----committed
17. windows 95 supports
a) multiuser b) n tasks c) both
ans : c
18. difference between printf and fprintf ----
19.to change permission r&w to owner group to no permission to others
a) chmod 614 b) chmod 604

c) chmod 640 d) chmod 310


20. in batch process ------------better job

##################################################################################
##################################################################################
##################################################################################
##########************************************************************************
**********************************************************************************
**********************************************************************************
**********************************************************************************
*******************************
**********************************************************************************
**********************************************************************************
**********************************************************************************
**********************************************************************************
**********************************************************************************
**********************************************************************************
**********************************************************************************
******************4***************************************************************
******5***************************************************************************
***********5**
*************************************************************7********************
***********************************************8**********************************
**********************************************************************************
**********************************************************************************
**********************************************************************************
**********************************************************************************
**********************************************************************************
**********************************************************************************
**********************************************************************************
**********************************************************************************
**********************************************************************************
**********************************************************************************
**********************************************************************************
***********************************************************
**********************************************************************************
**********************************************************************************
**********************************************************************************
**********************************************************************************
**********************************************************************************
**********************************************************************************
**********************************************************************************
**********************************************************************************
**********************************************************************************
**********************************************************************************
**********************************************************************************
**********************************************************************************
**********************************************************************************
********************************************************************
**********************************************************************************
**********************************************************************************
**********************************************************************************
**********************************************************************************
**********************************************************************************
**********************************************************************************
**********************************************************************************
**********************************************************************************
**********************************************************************************
**********************************************************************************
**********************************************************************************
**********************************************************************************
**********************************************************************************
**********************************************************************************
**********************************************************************************
**********************************************************************************
**********************************************************************************
**********************************************************************************
**********************************************************************************
**********************************************************************************
**********************************************************************************
**********************************************************************************
**********************************************************************************
**********************************************************************************
**********************************************************************************
**********************************************************************************
**********************************************************************************
**********************************************************************************
**********************************************************************************
**********************************************************************************
**********************************************************************************
**********************************************************************************
**********************************************************************************
*************************************************************************
**********************************************************************************
**********************************************************************************
**********************************************************************************
**********************************************************************************
***
>>from reddy@chemical.iitd.ernet.in wed sep 3 08:59:58 1997
received: from [144.16.224.3] by vanavil (aix 3.2/ucb 5.64/4.03)
id aa33370; wed, 3 sep 1997 08:59:58 gmt
received: from violet by brahma.iitm.ernet.in; (5.65/1.1.8.2/07feb96-0917am)
id aa06799; wed, 3 sep 1997 09:01:59 +0530
received: from [144.16.224.5] by violet.iitm.ernet.in (aix 3.2/ucb 5.64/4.03)
id aa06123; wed, 3 sep 1997 09:01:42 gmt
received: from milan.doe.ernet.in by spark; (4.1/1.1.8.2/13aug95-0211pm)
id aa26135; tue, 2 sep 97 21:31:52+060
received: from henna.iitd.ernet.in ([202.141.64.30]) by milan.doe.ernet.in
(4.1/smi-4.1)
id aa05863; sun, 31 aug 97 22:57:38+050
received: from chemical.iitd.ernet.in (netearth) by henna.iitd.ernet.in (4.1/smi-
4.1-mhs-7.0
)
id aa20218; fri, 29 aug 97 17:00:11 ist
x-organisation: indian institute of technology, new delhi.
return-path: <reddy@chemical.iitd.ernet.in>
received: by chemical.iitd.ernet.in; id aa05623; fri, 29 aug 1997 17:09:15 +0530
received: by chemical; fri, 29 aug 1997 17:09:15 +0530
id aa05623; fri, 29 aug 1997 17:09:15 +0530
date: fri, 29 aug 1997 17:09:14 +0530 (ist)
from: "mr.mv. surendra reddy" <reddy@chemical.iitd.ernet.in>
sender: "mr.mv. surendra reddy" <reddy@chemical.iitd.ernet.in>
reply-to: "mr.mv. surendra reddy" <reddy@chemical.iitd.ernet.in>
subject: from ravi
to: mepg9618@violet.iitm.ernet.in
message-id: <pine.3.89.9708291610.a32648-0100000@chemical.iitd.ernet.in>
mime-version: 1.0
content-type: text/plain; charset=us-ascii
status: r

dear srihari,
i received your mail & that hot mail too. how you got that mail.
earlier i sent a mail with full details of my interview.i think you did't
receive that.ok .i got in satyam secunderabad.till now dbss have not come.
it was a formal g.d followed by personel interview.in g.d the toppic was
"internet effect on indian culture."the lucky thing for me was no good
competitor in our batch. first he divided 50 people into 5 batches
and for each batch he given seperate topic.those are 1.computers in
communications...2.g.d is essential for mode of selection.etc
they took 4fellows from each batch.then in my interview heasked the
the first question was what is the differance between cad/cam.that
question i expected i answered very confidently.then i told about
concurrent engg. &reengineering. then they asked about personnel questions.
do interested in further studies etc..again they asked that have you done
any course in computers.actually i have't done any course. i told him
that i operated some packages.that is about auto cad & uni graphics&
pattern. on which platform you used auto cad. on ms-dos.then they asked
what is icon.then about unigraphics i did a minor project on that.
i told some more good on that.so that was my interview.is satyam sec bad
is coming to your campus or only dbss.our joining date is on 1st januvary of
98.what about our potti usha. she got job in which company.convey my best
wishes to vijay, srikanth,g.v.s.ok . waiting for your reply.

_ _ _ _
| | | | __ ___ _____ __ _ _ __ (_) ___ ___ __| | __ _ _ _
| |_| |/ _` \ \ / / _ \ / _` | | '_ \| |/ __/ _ \ / _` |/ _` | | | |
| _ | (_| |\ v / __/ | (_| | | | | | | (_| __/ | (_| | (_| | |_| |
|_| |_|\__,_| \_/ \___| \__,_| |_| |_|_|\___\___| \__,_|\__,_|\__, |
|___/
-----------------------------------------------------------------------------

yours

#[7m #[m
#[7m #[m#[4m #[7m #[m
#[7m #[m#[4m #[7m#[5m m.v. surendra reddy #[0m#[7m #[m
#[7m #[m#[4m nc-5, shivalik hostel #[7m #[m
#[7m #[m#[4m iit delhi, hauz khas #[7m #[m
#[7m #[m#[4m new delhi - 110 016 #[7m #[m
#[7m #[m#[7m #[7m #[m

>>from d@ren.ren.nic.in tue sep 30 18:34:09 1997


received: from [144.16.224.5] by violet.iitm.ernet.in (aix 3.2/ucb 5.64/4.03)
id aa18759; tue, 30 sep 1997 18:34:08 gmt
received: from cello.cs.iitm.ernet.in by spark; (4.1/1.1.8.2/13aug95-0211pm)
id aa09457; tue, 30 sep 97 18:35:27+060
received: from ren.ren.nic.in (ren.nic.in [164.100.10.18]) by
cello.cs.iitm.ernet.in (8.7.5/8.6.9) with smtp id saa16642 for
<mepg9618@cc.iitm.ernet.in>; tue, 30 sep 1997 18:31:49 +0530
received: by ren.ren.nic.in (smi-8.6/smi-svr4)
id saa06203; tue, 30 sep 1997 18:27:12 +0530
>received: (from u9626305@localhost) by iitkgp.ren.nic.in (8.6.11/8.6.9) id
oaa07313; tue, 30 sep 1997 14:41:23 +0500
date: tue, 30 sep 1997 14:41:22 +0500 (gmt+0500)
from: b v l n murthy <u9626305@iitkgp.ren.nic.in>
to: mepg9618@violet.iitm.ernet.in
cc: sekhar@ece.iitk.ernet.in
in-reply-to: <pine.lnx.3.91.970930143220.6535d-100000@iitkgp.ren.nic.in>
message-id: <pine.lnx.3.91.970930143906.6864b-100000@iitkgp.ren.nic.in>
mime-version: 1.0
content-type: text/plain; charset=us-ascii
status: ro

>
>
> siemens info
> this paper consists 6 parts. all are multiple choice q's
> 1)general
> 2)c/unix
> 3)c++/motif
> 4)database
> 5)x-windows
> 6)ms-windows
> we have written q's not acc. to each part.total 50. q's. time is
> sufficient.
> if u have basic idea about all of the u can easily answer the paper.
> paper
> ------
> 1)which of following operator can't be overloaded.
> a)== b)++ c)?! d)<=
> 2)#include<iostream.h>
> main()
> {
> printf("hello world");
> }
> the program prints hello world without changing main() the o/p should
> be
> intialisation
> hello world
> desruct
> the changes should be
> a)iostream operator<<(iostream os, char*s)
> os<<'intialisation'<<(hello world)<<destruct
> b) c) d)none of the above
> 3)cdpath shell variable is in(c-shell)
> a) b) c) d)
> 4) term stickily bit is related to a)kernel
> b)undeletable file
> c) d)none
> 5)semaphore variable is different from ordinary variable by
> 6)swap(int x,y)
> {
> int temp;
> temp=x;
> x=y;
> y=temp;
> }
> main()
> {
> int x=2;y=3;
> swap(x,y);
> }
> after calling swap ,what are yhe values x&y?
> 7) static variable will be visible in
> a)fn. in which they are defined
> b)module " " " "
> c)all the program
> d)none
> 8)unix system is
> a)multi processing
> b)multi processing ,multiuser
> c)multi processing ,multiuser,multitasking
> d)multiuser,multitasking
> 9)x.25 protocol encapsulates the follwing layers
> a)network
> b)datalink
> c)physical
> d)all of the above
> e)none of the above
> 10)tcp/ip can work on
> a)ethernet
> b)tokenring
> c)a&b
> d)none
> 11)a node has the ip address 138.50.10.7 and 138.50.10.9.but it is
> transmitting data from node1 to node2only. the reason may be
> a)a node cannot have more than one address
> b)class a should have second octet different
> c)classb " " " " "
> d)a,b,c
> 12) the osi layer from bottom to top
> 13)for an application which exceeds 64k the memory model should be
> a)medium
> b)huge
> c)large
> d)none
> 14)the condition required for dead lock in unix sustem is
> 15)set-user-id is related to (in unix)
> 16) bourne shell has
> a)history record
> b)
> c)
> d)
> 17)wrong statement about c++
> a)code removably
> b)encapsulation of data and code
> c)program easy maintenance
> d)program runs faster
> 18)struct base {int a,b;
> base();
> int virtual function1();
> }
> struct derv1:base{
> int b,c,d;
> derv1()
> int virtual function1();
> }
> struct derv2 : base
> {int a,e;
> }
> base::base()
> {
> a=2;b=3;
> }
> derv1::derv1(){
> b=5;
> c=10;d=11;}
> base::function1()
> {return(100);
> }
> derv1::function1()
> {
> return(200);
> }
> main()
> base ba;
> derv1 d1,d2;
> printf("%d %d",d1.a,d1.b)
> o/p is
> a)a=2;b=3;
> b)a=3; b=2;
> c)a=5; b=10;
> d)none
> 19) for the above program answer the following q's
> main()
> base da;
> derv1 d1;
> derv2 d2;
> printf("%d %d %d",da.function1(),d1.function1(),d2.function1());
> o/p is
> a)100,200,200;
> b)200,100,200;
> c)200,200,100;
> d)none
> 20)struct {
> int x;
> int y;
> }abc;
> you can not access x by the following
> 1)abc-->x;
> 2)abc[0]-->x;
> abc.x;
> (abc)-->x;
> a)1,2,3
> b)2&3
> c)1&2
> d)1,3,4
> 21) automatic variables are destroyed after fn. ends because
> a)stored in swap
> b)stored in stack and poped out after fn. returns
> c)stored in data area
> d)stored in disk
> 22) relation between x-application and x-server (x-win)
> 23)uil(user interface language) (x-win)
> 24)which is right in ms-windows
> a)application has single qvalue system has multiple qvalue
> b) " multiple " " single "
> c)" " " multiple "
> d)none
> 25)widget in x-windows is
> 26)gadget in x_windows is
> 27)variable destdir in make program is accessed as
> a)$(destdir)
> b)${destdir}
> c)destdir
> d)destdir
> 28)the keystroke mouse entrie are interpreted in ms windows as
> a)interrupt
> b)message
> c)event
> d)none of the above
> 29)link between program and out side world (ms -win)
> a)device driver and hardware disk
> b)application and device driver
> c)application and hardware device
> d)none
> 30)ms -windows is
> a)multitasking
> b) c) d)
> 31)dynimic scoping is
> 32) after logout the process still runs in the background by giving
> the command
> a)nohop
> b)
> 33)process dies out but still waita
> a)exit
> b)wakeup
> c)zombie
> d)steep
> 34)in dynamic memory allocation we use
> a)doubly linked list
> b)circularly linked
> c)b trees
> d)l trees
> e)none
> 35)to find the key of search the data structure is
> a)hask key
> b)trees
> c)linked lists
> d)records
> 36)data base
> --------------------------
> --------------------------------
> employ_code salary employ_code leave
> ---------------------------
> ----------------------------------
> from to
>
> --------------------------------------
> 1236 1500 1238 --- ---
> 1237 2000 1238 --- ---
> 1238 2500 1237 ---
> -----
> 1237 --- ---
> 1237 --- ---
> 1237 --- ---
> -------------------------- --------------------------------------
> select employ_code,employ_data ,leave
> the number of rows in the o/p
> a)18
> b)6
> c)7
> d)3
> 37)dbms
> 38)read about sql,db
> 39)which is true
> a)bridge connects dissimiler lanand protocol insensitive
> b)router " " " " "
> c)gateway " " " " "
> d)none of the above
> 40)read types of tree traversals.
> 41)42)43) simple programs on pointers in c
>
>

------------------------------------------------------
b v l n murthy
m.tech.
dept. of mechanical engg.
room# d/221
azad hall
iit, kharagpur
721 302
india
------------------------------------------------------

date: 28th aug 97

dear sekar,

tcs complete peper ( except psychometry ) i am sendig to you


with solutions.

psychomery is very very easy. but you should answer all quetions.

ok!

please give me reply immediatly.

paper-97
___________________

--------------------------------------------------------------------------------
calicut tcs paper --1997
--------------------------------------------------------------------------------

section 1 inturn consists of three parts.

part 1
verbal---
it consists of 40 words which has 5 multiple answers
for them.the main words which i remember are as follows for which
you should learn the synonyms of these---

merry= gay
alienate= estrange
solicit= beechat
heap= to pile
cargo= freight
momentary= transient
volume= quantity
veer= diverge
dispel= dissipate
admonish= cautious
meager= scanty
latitude= scope
latent= potential
covet= crave
discretion= prudence
latent= potential
emanicipate= liberate
lethargy= stupor
concur= acquiesce
confiscate= appropriate
baffle= frustrate
subside= wane
misery= distress
pretentious= ostentatious
tranquil= serene
efface= obliterate
obstinate= stubborn
hover= linger
caprice= whim
belate= too late
renounce= reject
brim= border
divulge= reveal
lament= wail
bileaf= conviction
adhesive= tenacious
furtive= stealthy
hamper= obstruct
to merit= to deserve

section 1 part b 20min 30 marks 15 quantitative.


~~~~~~~~~~~~~~~~~~~~~~~~~~~~~~~~~~~~~~~~~~~~~~~~~~~~~~~~~~~~~~~~~
these are some problems :

1.) two pencils cost 8 cents. then 5 pencils cost?


(20 cents)

2. a work is done by the people in 24 minutes. one of them can do


this workalonely in 40 minutes. how much time to do the same work
for the second person?
(60 minutes)

3. a car is filled with four and half gallons of fuel for a round trip.
fuel is taken 1/4 more in going then coming. what is the fuel
consumed in coming up? (2 gallons)
4.low temperature at the night in a city is 1/3 more than 1/2 high as
higher temperature in a day. sum of the low tem. and highest temp.
is 100 degrees. then what is the low temp? (40 deg.)

5. a person, who decided to go to weekened trip should not


exceed 8 hours driving in a day. average speed of forward journey
is 40 m/h. due to traffic in sundays, the return journey average speed
is 30 m/h. how far he can select a picnic spot?
a) 120 miles
b) between 120 and 140 miles
c) 160 miles
ans: 120 miles

6. a salesperson multiplied a number and get the answer 3,


instead of that number devided by 3.
what is the answer he actually has to get?
1 x 3 = 3
so number = 1
devided by 3, the ans. is 1/3.

7. a ship started from port and moving with i miles per hour and another
ship started from l and moving with h miles per hour.
at which place these two ships meet?

|----|----|----|----|----|----|
port g h i j k l

8. a building with height d shadow upto g. a neighbour building with


what height shadows c feet.

|----|----|----|----|----|----|----|
a b c d e f g h

9. a person was fined for exceeding the speed limit by 10 mph.


another person was also fined for exceeding the same speed limit
by twice the same. if the second person was travelling at a
speed of 35 mph, find the speed limit. (15 mph)

10.a bus started from bustand at 8.00am, and after 30 minutes staying
at destination, it returned back to the busstand. the destination
is 27 miles from the busstand. the speed of the bus is 18mph. in
return journey bus travels with 50% fast speed.
at what time it returns to the busstand? (11.00am).

11.in a mixture, r is 2 parts, s is 1 part. in aoder to make s to


25% of the mixture, howmuch r is to be added?

12. wind flows 160 miles in 330 min, for 80 miles how much time required.

13. with 4/5 full tank vehicle travels 12 miles, with 1/3 full tank
how much distance travels ( 5 miles )

14. two trees are there. one grows at 3/5 of the other in 4 years,
total growth of trees is 8 ft. what growth will smaller tree will have
in 2 years ( < 2 ft. )

15. a storm will move with a velocity of towards the centre in hours,
at the same rate how much far will it move in hrs.
( but the answer is 8/3 or 2 2/3 )

section-1 part-3 marks-50 questions-50 30 min

critical reasoning

1. my father has no brothers. he has three sisters who has two childs each.
my grandfather has sons.

a) my grandfather has two sons


b) three of my aunts have two sons
c) my father is only child to his father
d) i have six cousins from jmy mother side
e) i have one uncle

2. ether injected into gallablader to dissolve galstones. this type oneday


treatment is enough for gallastones not for calcium stones. this method is
alternative to surgery for millions of people who are suffering from this
disease.

a) calcium stones can be cured in oneday


b) hundreds of people contains calcium stones
c) surgery is the only treatment to calcium stones

3. hacking is illegal entry into other computer. this is done mostly because
of lack of knowledge of computer networking with networks one machine can
access to another machine. hacking go about without knowing that each network
is accredited to use network facility.

a) hacking people never break the code of the company which they work for
b) hacking is the only vulnerability of the computers for the usage
of the data.
c) hacking is done mostly due to the lack of computer knowledge.
(there will be some more questions in this one)

4. alphine tunnels are closed tunnels. in the past 30 years not even a single
accident has been recorded for there is one accident in the rail road system.
even in case of a fire accident it is possible to shift the passengers into
adjacent wagons and even the living fire can be detected and extinguished
with in the duration of 30 min.

a) no accident can occur in the closed tunnels


b) fire is allowed to live for 30 min.
c) all the care that travel in the tunnels will be carried by rail
shutters.
d)

5. in the past helicopters are forced to ground or crash because of the


formation of the ice on the rotors and engines. a new electronic device
has been developed which can detect the watercontent in the atmosphere and
warns the pilot if the temp. is below freezing temp. about the formation
of the ice on the rotors and wings.

a) the electronic device can avoid formation of the ice on the wings
b) there will be the malfunction of rotor & engine because of formation
of ice
c) the helicopters are to be crashed or down
d) there is only one device that warn about the formation of ice.

6. in the survey conducted in mumbai out of 63 newly married house wives


not a single house wife felt that the husbands should take equal part in the
household work as they felt they loose their power over their husbands.
inspite of their careers they opt to do the kitchen work themselves after
coming back to home. the wives get half as much leisure time as the husbands
get at the week ends.

a) housewives want the husbands to take part equally in the household


b) wives have half as much leisure time as the husbands have
c) 39% of the men will work equally in the house in cleaning and washing
d)

7. in confucius days the technology development was less and it took weeks
to communicate a message. wherein we can send it through satellite with in
no time........ even with this fast developments it has become difficult to
understand each other.

a) people were not intelligent during confucius days


b) transport facilities are very much improved in now-a-days
c) even with the fast developments of the technology we cannot live
happily.
d) we can understand the people very much with the development of
communication.

8. senior manager in a big company said that new japanies company invades
in india for transfering the cars from industrial and warned that jobs were
under threat from japanies company. they stated that increasing competence
would be coupled with an inevitable down term in car market and recent rise
in interest rate which has already hit demand.

a) manager issue their warning after a rise in interest rate


b) manager told workers that japanies workers are taking jobs away from
indian workers
c) manager said that more people want to buy new cars in future
d) increasing rate of interest mean that japanies firm will create into
operate in the country

9. human existence is suspecious of arbitrary divide between concise and


unconcise. the concise world invades shape activity of the unconcise, while
many of great activity of humanity waking as whole or partially improved
by dreams. even it could be ignored that dreams precede exceptional such a
dichotomy could not be drawn as the influence of dream on waking state would
reamin unclear. but as yet no company rebuilt exists to record the substitute
of prendtl dreaming.

a) sleepy can be creative state


b) it is difficult to tell whether a sleeper is dream or not
c) if we know what babies would dream about before they are born we could
show that the concise and unconcise mind influence on one another
d) it is untrue claim that concise and unconcise world never impinge
one another
10. any one who has has systematic exam phases will have perceived a
profound although not a prolif of asymmetry whether or not the exception
is volitions and self control of spontoneous appeal to predict facial as
symmetry as does the type of emotion potrayed. position can not displace
symmetric at left side regret of a negative emotion is more common posed
expression negation emotions are likely to be symmetric representation
and where as symmetric occurs relative left sided expression is more common.

a) any angry person is more likely to have left sided expression than
some one who has smiling
b) an actor is likely to smile symmetric when acting
c) delecious facial expression will always be as symmetrical

11. in the totalitariturian days, the words have very much devalued. in the
present day, they are becoming domestic that is the words will be much more
devalued. in that days, the words will be very much effected in political
area. but at present, the words came very cheap, we can say they come free
at cost.

a) totalitarian society words are devalued


b) totalitarian will have to come much about words
c) the art totalitarian society the words are used for the political
speeches
d)

12. there should be copyright for all arts. the rule has come that all the
arts has come under one copy right society, they were use the money that
come from the arts for the developments. there may be a lot of money will
come from the tagore works. we have to ask the benifiters from tagore work
to help for the development of his works.

a) tagore works are come under this copy right rule


b) people gives to theater and collect the money for development
c) people are free to go to the because of the copy right rule
d) we have ask the tagore residents to help for the developments of art.

if any doughts please use talk command.


seeu.

tcs
-----------------------------------------------

source : university of roorkee


dated : 31/07/97

1. vocabulary.(synonyms) 40 bits. 20 marks. 20min.


2. quantitative aptitude 15 bits 30 marks 15 min.
3. critical reasioning.(comprehension) 50 bits 50 marks 25 min.
4. psyhometric test. 150 bits 150 marks 30 min
-------- ------
250 marks 90 min
-------- -------

the psychometric test is simple and there is no


need of of preparation. it simply test your psychology. give most
positive answers to this section. i am sending some questions to
you. the vocab. is same. but there is some changes in passages.but
the idea is same.

take following precautions while answering test.


-------------------------------------------------------------
1. don't write full answers to vocab. section. do only 36 to 37
qwue's.because they disqualified some candidates here.

2. we are sending answers to this section. so you simply mug up


answers.

3. for critical reasoning section we are sending answers.but


some of these answers may change. so you please check in
examination. the answers is simply (true) or (false) or
(can't say) type. for this section you have read as fast
as you can. it simply test your comprehension ability.

4. eventhough we are sending answers to mathematical ability.


section. you do " rough " work back of your page. it is also
one of importnant caution.

5. in psychology section the qwe's may repeate at least four


times. by simply changing sentence formation. so becareful
give same answers to those qwe's.

there will be no negative marking. time 90 min., appeared for exam =255 students
selected for interview =122, finally short listed = 55 students.

section i vocabulary (synonyms) time :15 min. marks: 20.


direct answers :
-----------------------------------------------------------------
admonish : usurp meager :scanty alienate : estrange
merry : gay brim : boarder obstinate : stubborn
pretention:pretentioius tranquil:serene solicit : urge
subside : wane furtive :stealthy misery : disstress
volume :quantity veer : diverge stiffle :sniths
adhesive : --- hamper : obstruct belief : conviction
lament : wail to merit :to deserve incentive : ----
inert: passive baffle : frustrate confiscate : appropriat
covet : crave caprice : whim concur :acquiesce
cargo :freight dispel : scatter divulge : -----
discretion: prudence emancipate : liberate efface : obliterate
hover : linger heap : to pile instigate : incite
latitude : scope latent : potential lethergy : stupor
momentary : transient
----- means , for these words we are not able to reproduce answers.

part ii quantitative aptitude ,time 20 min. marks :30.


-----------------------------------------------------------------
1. two pencils costs 8 cents, then 5 pencils cost how much
(ans:20 cents).
2. a work is done by the people in 24 min. one of them can do
this work a lonely in 40 min. how much time required to do the same
work for the second person.
(ans:60 min.)
3. a car is filled with four and half gallons of oil for full round
trip. fuel is taken 1/4 gallons mor3 in going than coming. what is
the fiel consumed in coming up? (2 gallons)
4. low temperature at the night in a city is 1/3 more than 1/2
hinge as higher temperature in a day. sum of the low temp and
higherst temp is 100c. then what is the low temperature (40 c)
5. a person who decided to go weekend trip should not exceed 8 hours
driving in a day average speed of forward journy is 40 mph. due to
traffic in sundays, the return journey average speed is 30 mph.
how far he can select a picnic spot (120 miles).

6. a sales person multiplied a number and get the answer is 3,


instead of that number divided by 3. what is th answer he actually
has to get ? (1/3).
7. a ship started from port and moving with i mph and another ship
started from l and moving with h mph. at which place these two ships
meet ? ( ans is between i and j and close to j)

!_____!_____!_____!_____!_____!_____!
port g h i j k l

8. a building with hight d ft shadow upto g a neighbour building with


what height shadow c ft is (b ft.)

!_____!_____!_____!_____!_____!_____!_____!
a b c d e f g h

9. a person was fined for exceeding the speed limit by 10 mph.another


person was also fined for exceeding the same speed limit by twice
the same. if the second person was travelling at a speed of 35 mph.
find the speed limit (15 mph)

10. a bus started from bustand at 8.00a m and after 30 min staying at
destination, it returned back to the bustand. the destination is 27
miles from the bustand. the speed of the bus 50 percent fast speed.
at what time it retur4ns to the bustand (11.00)

11.in a mixture, r is 2 parts, s is 1 part. in order to make s to 25%


of the mixture, howmuch r is to be added ( one part).

12. wind flows 160 miles in 330 min, for 80 miles how much time
required.
13. with 4/5 full tank vehicle travels 12 miles, with 1/3 full tank
how much distance travels ( 5 miles).

14. two trees are there. one grows at 3/5 of the other. in 4 years,
total growth of trees is 8 ft. what growth will smaller tree will
have in 2 years. (<2ft)

15. a storm will move with a velocity of towords the center in


hours. at the same rate how much far will it move in hrs.
(but ans is 8/3 or 2 2/3).

part iii: time 25 min, marks :50.


-----------------------------------------------
critical reasoning : there will be 13 passages with 50 questions time 30 min.
here i am sending only some of the passages (these will give only rough idea)
(answers will be as yes/no/can't say we are giving our answers, please check.)

1. my father has no brothers. he has three sisters who has two childs each.

1> my grandfather has two sons (f)


2> three of my aunts have two sons(can't say)
3> my father is only child to his father(f)
4> i have six cousins from my mother side(f)
5> i have one uncle(f)

2. ether injected into gallablader to dissolve galstones. this type oneday


treatment is enough for gallstones not for calcium stones. this method is
alternative to surgery for millions of people who are suffering from this
disease.
1> calcium stones can be cured in oneday (f)
2> hundreds of people contains calcium stones(can't say)
3> surgery is the only treatment to calcium stones(t)
4> eather will be injected into the gallbleder to cure the cholestrol
based gall stones(t).

3. hacking is illigal entry into other computer. this is done mostly


because of lack of knowledge of computer networking with networks one
machine can access to another machine. hacking go about without knowing
that each network is accredited to use network facility.
1> hacking people never break the code of the company which they
work for (can't say).
2> hacking is the only vulnerability of the computers for the usage
of the data.(f)
3> hacking is done mostly due to the lack of computer knowledge (f).
(there will be some more questions in this one )

4. alphine tunnels are closed tunnels. in the past 30 yrs not even a single
accident has been recorded for there is one accident in the rail road
system. even in case of a fire accident it is possible to shift the passengers
into adjacent wagons and even the living fire can be detected and extinguished
with in the duration of 30 min.
1> no accident can occur in the closed tunnels (true)
2> fire is allowed to live for 30 min. (false)
3> all the care that travel in the tunnels will be carried by rail
shutters.(t)
4>

5. in the past helicopters are forced to ground or crash because of the


formation of the ice on the rotors and engines. a new electronic device has
been developed which can detect the watercontent in the atmosphere and warns
the pilot if the temp.is below freezing temp. about the formation of the ice
on the rotors and wings.
1> the electronic device can avoid formation of the ice on the wings
(false).
2> there will be the malfunction of rotor & engine because of formation of
ice (t)
3> the helicopters are to be crashed or down (t)
4> there is only one device that warn about the formation of ice(t).

6.in the survey conducted in mumbai out of 63 newly married house wives not a
single house
wife felt that the husbands should take equal part in the household work as
they felt they loose their power over their husbands. inspite of their careers
they opt to do the kitchen work themselves after coming back to home. the
wives get half as much leisure time as the husbands get at the week ends.
1> housewives want the husbands to take part equally in the household(f)
2> wives have half as much leisure time as the husbands have(f)
3> 39% of the men will work equally in the house in cleaning and washing
3>

7. copernicus is the intelligent. in the days of copernicus the transport and


technology
development was less & it took place weeks to comunicate a message at that
time.wherein we can send it through satellite with in no time ----------.
even with this fast developments it has become difficult to understand each other.
1> people were not intelligent during copernicus days (f).
2> transport facilities are very much improved in noe a days (can' say)
3> even with the fast developments of the techonology we can't live
happily.(can't say)
4> we can understand the people very much with the development of
communication(f).

q8) senior managers warned the workers that because of the intfoductors
of japanese industry in the car market. there is the threat to the workers.
they also said that there will be the reduction in the purchase of the sales
of
car in public.the interest rates of the car will be increased with the loss
in demand.
1> japanese workers are taking over the jobs of indian industry.(false)
2> managers said car interests will go down after seeing the raise in interest
rates.(true)
3> japanese investments are ceasing to end in the car industry.(false)
4> people are very much interested to buy the cars.(false)

q9) in the totalitariturican days,the words have very much devalued.in


the present day,they are becoming domestic that is the words will be much more
devalued.
in that days, the words will be very much effected in political area.but
at present,the words came very cheap .we can say they come free at cost.
1> totalitarian society words are devalued.(false)
2> totalitarian will have to come much about words(t)
3> the art totalitatian society the words are used for the political
speeches.
4>

q10) there should be copyright for all arts. the reele has came that all the
arts has come under one copy right society,they were use the money that come
from
the arts for the developments . there may be a lot of money will come from
the tagore works. we have to ask the benifiters from tagore work to help for
the development of his works.
1> tagore works are came under this copy right rule.(f)
2> people are free to go to the because of the copy right rule.(can't
say)
3> people gives to theater and collect the money for development.(can't say)
4> we have ask the tagore resedents to help for the developments of
art.(can't say)

note : do 1,2,3,4,5 passages which are easy. last but one also. do that
passages carefully. time will be insufficient. passages are not as exactly
as above. there is highlevel english in all the passages, we are giving
in our own words , u cannot expect the same type of english there. while
answering u should be very fast, do not waste time, it is insufficient,try to
answer
as many as possible.

section 4. psychometric test.


----------------------------------------
do not bothere about much about this test. be optimistic while answere.
there will be 150 questions in 30 min. the questions in this section may
repeated with slight variations answer should be same in both the cases.
(ans will be as yes/no/can't say)

for example
1> you will be interested in social activities.
2> while going upstairs you will move two steps at a time.
3> you will be making friendship with same sex or with opposite sex also.
4> your friends will consider u as a leader in your group
5> people think that your'e serious minded.
6> some times you feel dull without any reason.
7> your'e host or hostes for several parties.
8> relatives come to your house you will entertain them.
9> you will do work for longtime without tireness.
10> in your company you want lead the organasition.
etc.. the qwestions may repeate several times so becareful and give same ans's.

*************** all the best **********************


================================================================================

*******************************************************************************

-. .-
_..-'( )`-.._
./'. '||\\. (\_/) .//||` .`\.
./'.|'.'||||\\|.. (o o) ..|//||||`.`|.`\.
./'..|'.|| |||||\`````` '`"'` ''''''/||||| ||.`|..`\.
./'.||'.|||| || b.v.v.s.n.murthy ||||||.`||.`\.
/'|||' email- 96pg072@karthika.reccal.ernet.in |||`\
'.|||'.||||||| |||| ||||||||||||.`|||.`
'.||| ||||||||| |/' ``\||`` ''||/'' `\| ||||||||| |||.`
|/' \./' `\./ \!|\ /|!/ \./' `\./ `\|
v v v }' `\ /' `{ v v v
` ` ` v ' ' '

b.v.v.s.n.murthy(m.tech) p.g hostel #320


rec.calicut
kerala-673601
***************************************************************************

>>from chandra@cupid.che.iitb.ernet.in thu sep 4 14:39:52 1997


dear srihari,
telco was over here.gd and interview will be tomorrow.
for s/w persons test is as follows.
there are two sections each containg 40 questions.one arthematic
and another is reasoning. in arthematic 11 are data sufficiency and 8 are
anology.in reasoning all are puzzle type. probably he rejects very
few from test , since after that gd follows. any i am sending some q's as for
as my rememberance.
arthematic
1) hypothesis :problem (below 4 he will give)
2) mirror:image
3)money:misapprobation
4)nackles:adoration
5)construction:building
6)file:pile
7)ours:we
8)1/3,1 1/3,3,5 1/3, next
9)selling price of 4 articles=cost price of 3 articles then %loss(ans. 25%)
10)|x-3|=3-x then x=
11)data sufficiency p>q?
1)p,q positive
2)q-1=q*2+p
this type you can see in gmat book.
reasoning
1) abcdef attended for an interview, in which 3 were selected
a is worst of the lot
c got equal marks as f(c=f)
d<f and g
e is not selected
b>c
there are 5 questions below this.
2)there are 7 fellows sat in the following way
c and f always sit as a apart as follows
no. of fellows sat b/w c and d and d and f are equal like that he has given
ans: order is as follows c e b d a g f by using this you can answer all
about 4 to 5 are there
3)abcde are five brothers. there are twins(only one pair of equal age)
both are neither younger nor older. d is younger to 3 brothers.
b is older than e and c.
q's like who is youngest? eldest ? about 4 are there.
4)a person has meet a king for that ha has to cross 7 gates. at each gate he
has to pay half the amount he is carrying. finally he gave rs 3/- to the
king. then the amount he carried at the beginning and some questions
(about 5)like that.
5) heros tell truth and cowards lie.there pqr three persons. p tells q "i
may be hero or i may be coward". q tells r "p was telling that he was
coward. then r tells q "p was not a coward but he was a hero". you better
study the question it may not be entirely correct.
in thus 3 q"s are there

if want to go for hardcore there wont"be any test direct gd and interview

best of luck
sekhar
tisl -----------> paper model
two parts.
part a: aptitude (100 que) 100 marks
1 to 40 leter series
41 to 65 problems
66 to 100 figures (very very easy)
part b: comp knowledge test
50 que 50 marks
in this mail i am sending part a 1to 40 que in order
and part b 9to36 in order
+ten more que not in order
i will mail remaining questions in part a tomorrow.
sorry to day i am suffering with fever.
oooookkkkkkkkk!

tisl(tata-ibm)paper
-------------------
q9). what will be the result of executing following program
main
{
char *x="new";
char *y="dictonary";
char *t;
void swap (char * , char *);
swap (x,y);
printf("(%s, %s)",x,y);

char *t;
t=x;
x=y;
y=t;
printf("-(%s, %s)",x,y);
}
void swap (char *x,char *y)
{
char *t;
y=x;
x=y;
y=t;
}

a).(new,dictionary)-(new,dictionary)
b).(dictionary,new)-(new,dictionary)
c).(new,dictionary)-(dictionary,new)
d).(dictionary,new)-(dictionary,new)
e).none of the above
(ans will be b or e) check

q10).if a directory contains public files (can be valied and used


by any one ) which should not be altered ,the most liberal
permissions that can be given to the directory is
a)755
b)777
c)757
d)775
e)none of the above
(ans a)
11) what would the following program results in
main()
{
char p[]="string";
char t;
int i,j;
for(i=0,j=strlen(p);i<j;i++)
{
t=p[i];
p[i]=p[j-i];
p[j-i]=t;
}
printf("%s",p);
}
a)will print:string
b)will not print anything since p will be pointing to a null string
c)will print:gnirts
d)will result in a complication error
e)will print invallid characters(junk)
(ans will be b ) check
12) after the following command is executed
$ ln old new
a listing is performed with the following output
$ ls -li

total 3
15768 -rw-rw-rw- 2 you 29 sep 27 12:07 old
15768 " " " " " " " " new
15274 " " 1 " 40 " " 09:34 veryold

which of the following is true


a)old and new have same i-node number,2
b) " " " " " " " , 15768
c)old and new have nothing yo do with each other
d)very old and new are linked
e)very old and old are linked
(ans is b)
13) what will be the result of executing the following statement
int i=10;
printf("%d %d %d",i,++i,i++);
a).10 11 12
b).12 11 10
c).10 11 11
d).result is os dependent
e).result is compiler dependent
(ans is e)
14) what does extern means in a function declaration
a)the funct has global scope
b)the funct need not be defined\
c)nothing really
d)the funct has local scope only to the file it is defined in
e)none of the above
(ans will be c)
15) what will be result of the following program
main()
{
void f(int,int);
int i=10;
f(i,i++);
}
void f(int i,int j)
{
if(i>50)
return;
i+=j;
f(i,j);
printf("%d,",i);
}
a).85,53,32,21
b)10,11,21,32,53
c)21,32,53,85
d)32,21,11,10
e)none of the above
(ans is e)
16). ms windows 3.1 is a
a)operating system
b)application
c)programing language
d)database
e)shell
(ans will be b)
17).ms windows 3.1 supports which tyoe of multi-tasking?
a)cycle
b)executive
c)preemptive
d)non-preemptive
e)manual
(ans )
18)the command ......ln/bin/mail /usr/you/bin/m
a)will not be executed because you are linking files
across different file systems
b)results ln /bin/main being the same file as /usr/you/bin/m
c)results in 2 links to the file mail
d) " " " " m
e)none
(ans will be b)
19)in a standerd directory lay out ,/etc is the directory where
a) basic programs such as who and ed reside
b) - - - - - - -
c)various administrative files such as password file reside
d) - - - - - - -
e) - - - - - -
(ans is c)
20) the command echo *
a) echoes all files in the current directory
b) - - - - -
c)
d)
e)
(ans is a)
21)what will be the result of the following segment of the program
main()
{
char *s="hello world";
int i=7;
printf("%.*%s",s);
}
a)syntax error
b)hello w
c)
d)
e)
(ans is b)
22) what will be the result of the following program
main()
{
int a,b;
printf("enter two numbers :");
scanf("%d%d",a,b);
printf("%d+%d=%d",a,b,a+b);
}
a)- - - - -
b) - --
c) will generate run time error /core dump
d)
e)
(ans is c)
23) what is the size of 'q'in the following program?
union{
int x;
char y;
struct {
char x;
char y;
int xy;}p;
}q;
a)11
b)6
c)4
d)5
e)none
(ans is b why because no of bytes for int =4 given in instructions)
24) which message is displayed when a window is destroyed
a)wm_close
b)wm_destroy
c)wm_ncdestroy
d)
e)
(ans is b)
25)send message and postmessage are
a)send message puts the message in the message queue and results,
postmessage processes the message immediately
b)sendmessage processes the message immediately,postmessage puts
the message in the queue and returns
c) both put the message in the message queue and returns
d) both process the message immediately
e) none of the above
(ans will be b check)
26) which of the following message is used to limit the size
of teh window
a)wm_size
b)wm_pain
c)- - - -
d)- -- - -
(ans is a)
27)until who|grep mary
do
sleep 60
done
a) is syntactically incorrect
b) waits 60 seconds irrespective of mary being logged in or not
c) waits until marry is logged in
d)waits till mary exited
e)none
(ans is c)
28)the unix system call that transforms an executable binary file into
a process is
a)execl()
b)execv()
c)execle()
d)execve()
e)all of the above
(ans will be d check)
29)which of the following is true about fork()
a)- - - - -
b)causes the creation of a new process ,the child process
with a new process id
c)
d)
e)
(ans is b)
30) what do the following variable names represents?
sort register
volatile default
a)- - - -
b ) - - - -
c)all the above are keywords
(ans is c)
31)what will be the result of the following program
main()
{
char *x="string";
char y[] = "add";
char *z;
z=(char *) malloc(sizeof(x)+sizeof(y)=1);
strcpy(z,y);
strcat(z,y);
printf("%s+%s=%s",y,x,z);
}
a)add+string=add string
b)syntax error during compilation
c)run time error/core dump
d)add+string=
e)none
(ans will be e consider cap&small leters)
32)what does the following expression means
a)
b)
c)
d)an arrey of n pointers to function returning pointers to
functions returning pointers to characters
(ans is d)
33)which of the following is not a ddl object
a)hbrush
b)hpen
c)hbitmap
d)hrgn
e)hwnd
(ans ic e)
34) which of the following message is used to initialize the
contents of a dialog
a)wm_create
b)wm_size
c)wm_command
d)wm_initdialog
e)none
(ans will be d)
35)interprocess communication in unix can be achieved using
a)pipe
b)message
c)semaphores
d)shared memory
e)all of the above
(ans is e)
36) which of the following is true
a)unix is a time sharing multi-user os
b)unix has a device independent file system
c)unix is full duplex
d)unix has command interpreter
e)all of the above
(ans is e)

q). ps1 pwd


export ps1 results in
a). your primary prompt being your current directory
b). " " and secondary prompts being the current dir
c). " " prompt being your home dir
d). " " and secondary prompts being the home dir
e). none of the above.
q). if you type in the command
nohup sort employees > list 2 > error out &
and log off ,the next time you log in . the output
will be
a). in a file called list and the error will de typed in
a file error out
b). there will be no file called list or error out
c). error will be logged in a file called list and o/p
will be in error out
d). you will not be allowed to log in
e). none of the above
q). in unix a files i-node
a)is a data structure that defines all specifications
of a file like the file size ,number of lines to a
file ,permissions etc.
b).----
c). - - - --
d). _ _ _
( ans is ---------(a) )
q). the unix shell is....
a).does not come with the rest of the system
b).forms the interface between the user and the kernal
c)-- -- ---
d) - - - -
e) none
(ans is (b) )
q).enum number { a=-1, b= 4,c,d,e}
what is the value of e ?
7,4,5,15,3
(ans is 7 ) check again
q).the very first process created by the kernal that runs
till the kernal process is haltes is
a)init
b)getty
c)
d)
e)none
(ans is a)
q) result of the following program is
main()
{
int i=0;
for(i=0;i<20;i++)
{
switch(i)
case 0:i+=5;
case 1:i+=2;
case 5:i+=5;
default i+=4;
break;}
printf("%d,",i);
}
}
a)0,5,9,13,17
b)5,9,13,17
c)12,17,22
d)16,21
e)syntax error
(ans is d )
q) what is the result
main()
{
char c=-64;
int i=-32
unsigned int u =-16;
if(c>i){
printf("pass1,");
if(c<u)
printf("pass2");
else
printf("fail2");}
else
printf("fail1);
if(i<u)
printf("pass2");
else
printf("fail2")
}
a)pass1,pass2
b)pass1,fail2
c)fail1,pass2
d)fail1,fail2
e)none
(ans is c)

aptitude test
*****************************************************

missing leter
1).eefgghii- (ans j)
2)
3)defdefghi- (ans g)
4)cdexyzfghxyz-(ans i)
5)defdegde- (h)
6)abczabcyabc- (x)
7)fgbhibjkb- (l)
8)- - - - - (ans is r)
9)aarbsctarb- (s)
10)bccdeefg- (g)
11)efhikl- (n)
12)abccdeffg- (h)
13amnbopc- (q)
14)tttssrqqqp- (p)
15)ddffhhjj- (l)
16)mnmnklopopkl- (q)
17)cddeeefff- (f)
18)gfed- (c)
19)dfhjl- (n)
20)abcijdefij (g)
21)efgefghefghi- (e)
22)bcbdedfgfhi- (h)
23)aababccdc- (d)
24)aibcidef- (i)
25)cehl- (q)
26)abdehimn- (s) check again
27)becfdge- (h)
28)agbhc- (i)
29)adhko- (r)
30)efghjklno- (q)
31)aedhg- (k)
32aeibf- (j)
33)zdwgt- (j)
34)zeyijxg- ()find
35)cqreuvg- (y)
36)ksjtiuh- (v)
37)rsjtuhvw- (f)
38)ieajfbk- (g)
39)hebifej- (g)
40)hjlmiel- (h)
--------------------------------

total 60 que's 25-problems & 35 figures(figure series,we


have to find next coming figure)
here 13 prob's are thers.remaining also almost like this .

1) a boy multiplied a number with 10 and got 100,insted of


dividing it . if he devided it what would be the answer?
(ans is 1)
2)if 12 shell cup board requires 18ft of wall space then
30 " " " " how much wall space?
(ans is 45)
3)the average salary of three employee is 95rs.per week.
if one employee earns 115 and other earns 65 rupees.
how much will third be earn?
(ans is 105rs)
4)a company instaled 36 punching machines at the begining of
the year.in the spring they instaled 9 additional m/c's
and then discontinued 18 in the fall.how many were still
installed at the end of the year?
(ans is 9)check
5)during a given week a programmer spend 1/4 of his time
preparing charts,3/8 of his time for coding,rest of his time
for debugging the programs.if he had 48 hrs during the week
how many hours did he spend debugging the program.
(ans is 18hrs)
6)a 16 story building has 12000ft on each floor. company a
rents 7 floors and company b rents 4 floors. what is the
number of square feet of unrented floor space?
(ans is 60000 sqft)
7)a man owns 2/3 of a computer service buroue business and
sells 3/4 of his share for $75000.what is the value of the
business.
(ans is 150,000)
8)a computer printer produces 176400 lines in a given day.
if the printer was in operation for 7hrs during the day
how many lines did it print per minute?
(ans is 420)
9)from its total income a company spent $20000 for advertising
half of the remainder on salaries and had $6000 left.what
was the total income?
(ans is $32000)
10)in a certain company 20% of the men and 40% of the women
attended the annual company picnic.if 35% of all the emplo-
yees are men .what % of all the employee went to the picnic?
(ans is 33%)
11)the dimensions of certain ibm m/c are 48"*30". if the
size of the m/c is increased proportionally until the
sum of its dimensions equals to 156".what will be the
increase in the shortest side?
(ans is 30)
12)if a card punch operator can process 80 cards in half an hour
how many cards can this process in 7hr30min?
(ans is 1200)
13)in a computer tape library there are two racks with 40 tapes
per rack.in a given day 30 tapes are in use . what fraction
remains in the rack?
(ans is 5/8)
1.

name of the company: tcs


date of test/interview: 4,5,6 sept
people attended from m.tech.,(cs):
ch.ravikumar,a.rajamohan,j.jayakumar
about the company: junta company
3 year bond
largest s/w firm in india
revenue of 560 crores
they claim to work in all fields.
they are emphasising that they are not just cobol
company they do other good things also.
place of posting can be any where.
three to four months training .the training
is in one of the following places .
delhi,madras,bombay.
pay is basic:6500
hra:30% of the basic
medical :31000rs
pf:10%
etc.
people screened for interview: ch.ravi kumar,j.jaya kumar,a.raja mohan
people selected from m.tech., (cs):
j.jayakumar
2.

name of the company: infosys technologies ltd. bangalore


date of test/interview: 10,11 sept
people attended from m.tech.,(cs): 9
ch.ravikumar,a.rajamohan,m.raveendra babu,m.kedar prabhakar,
sandeep singh,naveen k. thyagi,n.d.p.rao,arvind kumar,
m.ravi kumar.
about the company: turnover 900+ million rupees
1165 personnel
main office in kreonics city, ba.
they work in
distribution
banking & finance
manufacture
communications
insurance
85% of their turnover is in s/w exports.apart from that
they have some packages like bancs2000
they market some products also.
they have iso9000 certification
they have excellent facilities in their bangalore office.
they are planning to expand .
avg. age is 24 yrs in the company.
1 year bond and no money involved.
pay is :rs10,577 p.m. (gross)
people screened for interview: naveen kumar thyagi,m.ravi kumar
people selected: none from cs dept.

the question paper what ever we remember:


1.my neighbour has seven children.every brother has equal no. of
brothers and siters ,but each sister has twice as many brothers
as sisters?
ans: 4,3
2.there are 11 more animals than birds in a pet shop.if there are
as many birds as animals and if there are as many animals as birds
the no. of legs is 4/5 of the original.howmany birds and animals
are there?
ans: 22,11
3.one soap can be made out of the scraps obtained while preparing
11 soaps.how many soaps can be prepared out of the scrap obtained
out of 251 soaps.(take all possibilities)
ans: 25
4.find out the five digit number whose first digit is three times
that of it's fifth digit,the fourth digit is four more than the
second digit,the third digit is three less than the second digit,
and there are 3 pairs of digits such that each sum is 11?
ans: 65292
5. 2xx
3xx
---
5xx
x4x
xx3
-------
xxxxx
-------
find out the x's and seven is not there in the calculation.

ans: 281x332
6.there are 5 burglars and once went to a bakery to rob it obviously
the first guy ate 1/2 of the total bread and 1/2 of the bread.
the second guy ate 1/2 of the remaining and 1/2 of the bread.
the third guy ,fourth guy and fifth guy did the same.after fifth
guy there is no bread left out.how many bread are there?
ans:31
7.the main line train starts at 5.00am and the harbour line train
starts at 5.02am.each train has the frequency of 10 minutes.if a
guy goes in the morning at a random time what is the probability
of he getting main line train?
ans: 0.8
8.there is 66x33m rectangular area .ram is 11/8 times faster than
krishna.both of them started walking at opposite ends and they
met at some point then,ram said "see you in the other end"then
they continued walking.after some time ram thought he will have tea
so he turned back walked back 15 meters then he changed his mind
again and continued walking .how much krishna has travelled by
the time they meet?
9.a farmer has c chickens.a sack of feed comes for 9 days.as the
feed cost is increasing the farmer sells some chickens and retains
12 chicken.if he reduces the feed quantity by 10% .then he observes
that the feed comes for 30 days.what is c?
ans:36
10.escalator problem.
a person walking takes 26 steps to come down on a escalator and it
takes 30 seconds for him for walking.the same person while running
takes 18 second and 34 steps.how many steps are there in the escalator?
ans:46
anal paper:
1. there are nine cards arranged in three rows and three columns.
each row contains atleast one green card.there are three yellow
corners.red is either in the first row or second row.there are
2 greens in the 3rd column and 2 blues in the second row.what is
the arrange ment of the cards
ans: y r g
b b g
y g y
2. there are eight cards.each card has colour on both sides.there
are 2 blue 2 red 2 green 2 yellow.
name of the company: ramco systems ltd.
date of interview/test: 12/9/96 ,13/9/96.
people attended from mtech: naveen kumar thyagi, ch.ravikumar, n.durga prasada
rao, ravindran, rajamohan.
people attended from ms: salome jacob and radha krishnan.

about the company:


it is a company started in 1989 by ramco group of companies
it has some 950 personnel working. it is a madras based company. their's is the
only product based company in india. their product is called ramco marshall
it is based on client-server architecture and the front end for that is ms-
windows.
their workareas are as follows:
client-server computing
gui
rdbms
distributed computing(this is jargon)
work flow
business objects
edi(electronic data interchange)

they have an allience with ms and monitoring technologies,usa.


they believe in localising operations and global marketing(official way of saying
we won't send you abroad)
pay package:rs.1.44 lac per annum.
take home is 9,400 rs.
people screened for interview: salome jacob,naveen kumar thyagi
people selected from mtech: none
people selected from ms: salome jacob.

4.
name of the company: computer vision india ltd.
date of the interview/test:14/9/96
people who attended for test: m.raveendra babu,m.kedar prabhakar,arvind kumar,
a.rajamohan,ch.ravikumar,m.ravikumar,n.durgaprasada rao,j.gopala krishna
about the company:

it is started in 1994 in india. it is a subsidary of computer vision,us.


they are mainly known as cad/cam company.they have 220 people right now.
they have some 250 sun sparc workstations in pune.it is a flat organisation.
flexi working hours.among 220 personnel 17 are ph.d's and 110 are mtechs
and the rest of them are mca's.
it is a product based company.it's products include cadds5,medusa,dim3
pelorus(scalable cad architecture),discover(oodevelopment tool),discover,
purify,optegra.they are interested in the following
math tech.
design
soft.stage(mainly maintainance)
developing tools(mainly case tools)
electronic product development(epd)
pay package: rs1.65lac+free lunch+transportation

people screened for interview: m.raveendra babu,m.kedar prabhakar,m.ravi kumar,


arvind kumar,j.gopala krishna,n.durga prasada rao

people selected: m.raveendra babu,m.kedar prabhakar,m.ravi kumar,arvind


kumar,j.gopala krishna

5.

name of the company: wipro infotech,(global r&d)


date of the test/interview:15/9/96
people attended for test from mtech: anurag sharma,m.ranga rao choudary,
a.phani bhushan
n.v.shaji,m.shilpa,ashish k. hanwadikar,k.venkatesh,v.surya narayana,
a. siva shankar,k.venu madhav,sandeep singh,ch. ravi kumar,a.raja mohan
people attended for test from ms: anantha raman,shankara raman,rajesh kumar,
santosh kumar,gayathri,renuka,sudha kiran,n.jagdish babu,radha krishnan,
dheena,vargheese
about the company:
they are working in
fault tolerant technology,
h/w & asic
object oriented technology,
communications&n/w
os & device drivers

database internals
client-server
ibm mainframes
telecom software
financial applications

their clients include at&t,ub networks,tandem etc.


their turnover is around rs.50cr and it is expected to be around
rs.80 cr. this time.expected growth rate is around 30% for 1996-97
pay package: rs.1.57lac per annum
take home rs.12,000 p.m.
people scrrened for first interview:
from mtech: mrrc,anurag,phani,akh,venky,shaji,surya,sans,sm
from ms: shankr,araman,krishv,rajesh,skumar,gayatri,renuka,kiran,
people screened for second interview:
from mtech: mrrc,anurag,phani,akh,venky,shaji
from ms:shankr,araman,rajesh,skumar,kiran,gayatri
people selected from mtech: m.ranga rao choudary,anurag sharma,
a.phani bhushan,ashish k. hanwadikar,k.venkatesh
people selected from ms: k.n.anantha raman,shankar raman,b.rajesh kumar
i.santosh kumar,sudha kiran,t.r.gayatri

6.
name of the company: hcl_hp
date of test/interview:17/9/96 & 18/9/96
people attended for test:
from mtech: naveen kumar thyagi,sandeep singh,t.c.vengatesan
n.durga prasada rao,ch.ravi kumar,a.raja mohan,ravindran,n.v.shaji,
a.siva sankar,k.venu madhav
from ms: n.jagdish babu,
about the company:
it started in 1976.it is located in two places madras and noida.it
has two seperate tests for s/w and h/w.
totally informal and one guy who came here from company was wearing
a t-shirt and jeans.
flexible working hours.
niit is also a componant of it.
pay package: rs.1.56lac per annum
people screened for interview:
from mtech: naveen kumar thyagi
from ms: n.jagdish babu
people selected :
from mtech: naveen kumar thyagi
from ms: n.jagdish babu

7.
name of the company: oracle
date of the interview/test: 19/9/96
people who attended test:
from mtech: v.surya narayana,sandeep singh,naveen kumar thyagi,
n.v.shaji,ch.ravi kumar,j.gopala krishna,m.shilpa,
a.raja mohan,a.siva sankar,k.venu madhav
from ms: n.jagdish babu,kalyan,ganesh, ........
about the company:
it is the second largest s/w firm.hazzaar perks.lot of parties.
screwed junta in the written by asking 70% rdbms,which they never
quoted as their requirement(they quoted c,unix,c++) in their
advertisements.they will give atz grindlays card,somany loans,
weekly booze parties etc etc
pay package: rs.1.77lac per annum
people screened for interview:
from mtech: none
from ms: n.jagdish babu

8.
name of the company: intergraph
date of the interview/test: 18/9/96
people who attended test:
from mtech: a.raja mohan
from ms: none
(there is a cg cut off for writing the test)
about the company:
initially they said they require only civil and mech guys
they called cs guys with reluctance and they didn't screened
anyone for interview.

people screened for interview:


from mtech: none
from ms: none

9.
name of the company: motorola india electronics ltd.
date of the interview/test: 20/9/96 and 21/9/96
people who attended the interview:
from mtech: anurag sharma,k.n.guruprasad,a.phani bhushan,
k.venkatesh,n.v.shaji,sandeep singh,
v.surya narayana,k.venu madhav,m.shilpa,
ashish k. hanwadikar,t.c.vengatesan
from ms: dheena,sudhakiran,ganesh
about the company:(the info here is the opinion expressed by the class members who
attended the interview immedieately after the interview. so take note of this and
it might have been affected by one of senior who is working comments or what ever
it is.take this into consideration.)
this is the right royal screwing company you can get in indian
scenario.it seems they put video cameras to monitor the moments
of the peopl inside the company.their e-mails will be filtered
and you will be doing only testing.you might have to work minimum
12 hours a day to 14hours a day.no perks or facilities.work environment
is good,salary is less compared to avg.industry standard.company
stamp is considered to be best.they won't give the details about
their work .everything is confidential.in the world
everybody is suspect until proved otherwise
for them.they screwed guys in the interviews saying that you people
will be leaving the company.how will you assure us to the contrary?
is their paining repeated question.it seems among 15 persons
in a team in one year itself 10 persons left.that's why i think
they are getting pained.anyway,if you wanna enjoy u'r self this is
not the company for you.if you want the stamp then yes it is the
company.
phani adds
(as on 10/10/96)
that there are no cameras for watching and it seems that he got it confirmed from
one of senior working there called bagi
pay package: rs.1.42lac per annum
people selected finally:
from mtech: k.venkatesh,a.phani bhushan
from ms: dheena

10

name of the company: cmc, hyderabad


date of test/interview: 23/9/96
people who attended test:
from mtech: k.venumadhav,ch.ravikumar,a.rajamohan
from ms: none
about the company:
it works in banking s/w,automation(ports,offices etc etc)
it is one of the good public sector company.the experience in this
company is considered to be good. it is sole company which does all
sorts of job for the govt.of.india.it is responsible for the railway
reservations and all.so basically it is a good company.

pay package: rs.1.26lac per annum(take home is around 8000 rs.p.m)


people screened for interview:
from mtech: a.rajamohan
from ms: none
people selected finally:
from mtech:
from ms: none

12
name of the company: honeywell india ltd.
date of test/interview:25/9/96
people who attended test:
from mtech:sandeep singh,ch.ravikumar,n.durgaprasadarao,k.venumadhav,
v.suryanarayana,a.sivasankar,m.shilpa,

from ms:
about the company:
it is an mnc and is mainly in the development of inflight s/w and has
very good customers.first one or two years like in any other mnc one
will be put in testing then only any development work will be given.

pay package;
people screened for interview:
from mtech: sandeep singh,a.siva sankar,a.raja mohan
from ms: renuka,ramakrishnan,santosh
people selected finally:
from mtech:
from ms:

13
name of the company: hughes software systems , gurgaon
b
b
b
b
b
b
b
b
b
b
b
b
b
b
b
b
b
b
b
b
b
b
people who attended test:
from mtech:n.v.shaji,arvindkumar,t.c.vengatesan,n.durga prasada rao,
a.siva sankar,v.surya narayana,k.venu madhav,a.raja mohan,
sandeep singh,ch. ravi kumar,r.ravindran
from ms: varghese.........
about the company:
pay package;rs.1.85lac per annum
people screened for interview:
from mtech:n.v.shaji,arvindkumar,t.c.vengatesan,n.durga prasada rao,
a.siva sankar,v.surya narayana,k.venu madhav,sandeep singh
from ms:varghese....................
people selected finally:
from mtech:n.v.shaji,arvindkumar,t.c.vengatesan,a.siva sankar,v.surya
narayana
from ms: varghese
14
name of the company: future s/w,madras
date of test/interview:28/9/96
people who attended test:
from mtech:sandeep singh, m.ranga rao choudary,r.ravindran,n.durga
prasada rao,k.venu madhav
from ms: radha krishnan, renuka
about the company:
pay package; rs. 1.7lac per annum
people screened for interview:
from mtech: sandeep singh,m.ranga rao choudary,r.ravindran,n.durga
prasada rao
from ms: none
people selected finally:
from mtech:sandeep singh, m.ranga rao choudary
from ms: none
15
name of the company:
date of test/interview:
people who attended test:
from mtech:
from ms:
about the company:
pay package;
people screened for interview:
from mtech:
from ms:
people selected finally:
from mtech:
from ms:
16
name of the company:
date of test/interview:
people who attended test:
from mtech:
from ms:
about the company:
pay package;
people screened for interview:
from mtech:
from ms:
people selected finally:
from mtech:
from ms:
17
name of the company:
date of test/interview:
people who attended test:
from mtech:
from ms:
about the company:
pay package;
people screened for interview:
from mtech:
from ms:
people selected finally:
from mtech:
from ms:
>>from krishna@bronto.iitm.ernet.in thu aug 28 18:33 ist 1997
return-path: <krishna@bronto.iitm.ernet.in>
received: from deer.iitm.ernet.in by bronto.iitm.ernet.in (5.0/smi-4.0)
id aa00499; thu, 28 aug 1997 18:32:26 --5-30
received: (from krishna@localhost) by deer.iitm.ernet.in (8.7.5/8.7.3) id
saa08527; thu, 28 aug 1997 18:34:48 +0530
from: krishna@bronto.iitm.ernet.in
message-id: <199708281304.saa08527@deer.iitm.ernet.in>
subject: verifone paper!
to: alugu@bronto.iitm.ernet.in, umamr@bronto.iitm.ernet.in,
mani@bronto.iitm.ernet.in, prasad@bronto.iitm.ernet.in,
sreej@bronto.iitm.ernet.in, badri@bronto.iitm.ernet.in,
rafi@bronto.iitm.ernet.in, amit@bronto.iitm.ernet.in,
adir@bronto.iitm.ernet.in, chigu@bronto.iitm.ernet.in,
dalton@bronto.iitm.ernet.in, hanu@bronto.iitm.ernet.in,
ksunil@bronto.iitm.ernet.in, nethi@bronto.iitm.ernet.in,
raella@bronto.iitm.ernet.in, ravik@bronto.iitm.ernet.in,
sasik@bronto.iitm.ernet.in, sreeni@bronto.iitm.ernet.in,
sriram@bronto.iitm.ernet.in
date: thu, 28 aug 1997 18:34:48 +0530 (ist)
x-mailer: elm [version 2.4 pl25]
mime-version: 1.0
content-transfer-encoding: 7bit
content-type: text/plain; charset=us-ascii
content-length: 5675
status: or

hai friends,

here is the verifone paper. enjoy!!

bye
mohan.

----------------------------------------------------------------------------
verifone

verifone test questions :

there are two parts :

1. aptitute test : 15 minutes, 20 questions

some questions are:


(not in order)
1. a question (first one)on addition of fraction of inches
a. was the answer

2. there were 36 chairs. how many ways can they be placed such
that all rows have equal no. of chairs and atleast three chairs
are there in each row and there are atleast three rows.
5 ways.

3. there are 27 balls, of which 1 is heavier. given a balance


how many times you need to weigh to find out the odd ball.
3 weighs.
4. product of three consecutive nos. 210. what is the sum
of two least numbers
11.

5. if the area of the sqaure is increased by 69 % how


much the length of the side will increase?
30%
6. if the sum of five consecutive nos. 35? how many prime nos
are there :
2 primes.

7. if the length of the rectangle is reduced by 20% and breath


is increased by 20 % what is the net change ?
4 % decrease
8. a question on sets.
there are some 20 basketball players & 30 football players,
and 25 cricket players. 1 of them plays all the three games.
8 of them plays atleast two games. they are 50 altogether.
how many of them plays none of the games.
9. a question on directions.
b is 20 miles east of a. d is 30 miles east of c. e is 10 miles
north of d. c is 20 miles north of b. how far e is from a?
some 3 questions on reasoning like,
10. if you say that giving stock options to employees increases the
productivity of the company, which of the following sentences
support it.
a) giving stock options increases the morale of the employees
..
..
etc.,
11. gamblers comes to the amusement parks. there are some amusement
parks in each city. there are some gamblers in each city. so
what can you infer.
a) amusement park always have gamblers.
..
..
etc.,

2. technical questions.

i. electrical & electronics : 15 questions


1.a circuit with nand gates. (ans. may be xor)

2.cmrr. relates to (options not in order)


voltage follower
non invering amplifier
inverting amplifier
integrator

3. given a circuit , give the ouput.


(ans. may be triangular wave.)

4. o/p of an assembly code.


mulitply by 11.

5. how to handle asynchronous events.


a) polling
b) interrrupt
etc.

ii)data structures, algo., & complexity theory : 5 questions


iii) os : 5 questions
iv) networks and hardware: 5 questions
v) databases and misc.: 5 questions
vi) c pgm. : 5 questions
some more 10 questions.

1. if w is a sequence of strings without a and w' is its reversal


then waw' is accepted by:

context free grammmars

2. whether all recusive pgm can be writtten iteratively?


yes.
3. what data structes you will use if you want to go to first record from
the last and vice versa?
doubly linked circular list

4. given 10000 nos. and 48mb memory. what is the complexity of the
efficient sorting algo.? (the algo. is not mentioned)
o(n)

5. given a c code and ask what it does?


code was something similar to bubble sort and that
particular code does the sorting in desending order and the complexity
is o(n^2)(which is the next question).

6.
a code like this is given.
a. for(i=0;i<num;i++)
b. for(i=num;i>0;i--)
assuming no code optimization and assume that the microprocessor
has flags etc. which one is correct.

b will execute faster.

7. if there are too many page faults what is the problem?

8. to ensure one pgm. doesnt corrupt other pgm. in a multi-pgm enviornment


what you should do?

9. which one you will use to implement critical section?


binary semaphore

10. which one is not needed for multi-processing. enviornment?


options are: virtual memory,security,time sharing,none of the
above.

11. which one is not done by data link layer ?


bit stuffing, lrc,crc,parity check

12. which one is not related to data link layer?


13. which one is not suitable for client-server application?
tcp/ip,message passing,rpc,none of the above.

14. what is sql.


procedural relational db query language.

15. indexing in databases give you


options were like 1.efficient deleting and inserting
2.efficient deleting.
etc.
16. int a=1,b=2,c=3;
printf("%d,%d",a,b,c);
what is the output?
17. scope of static variable ............
in a file.

18. for(i=0; i<=10;i++,printf("%d",i)); +- (+- is there in the questions)

20. real time os should have


a)fast context switch
b)virtual memory etc.

21. messages are transferred in some e71 code, where after 7 bits of data,
1 bit of stopping data is to be transferred. what should be done.
options were like
a) send directly
b) send after encoding
etc.
22. there are three processes a, b, c. a sends data to b. b removes the
header stores it and sends the data to c. c returns it to b. b receives the
message, identifies the message and adds the header that was stored and
sends to a.b receives the messages from c such that atmost 'm' messages
b
are pending.
identify the best data structure.

23. a question in compiler about the heap and stack allocation of memory.

24. struct
{
char a[3];
int b;
}x;
char *cp;

a) size of x is 7.
b
b)
c)
d) cp takes the size of a pointer.
(d ) is the ans.
wipro paper(system software)
july-1997
------------

part --a
------------------------------------------------------
1) abcd+abcd+abcd+abcd
then the simplified function is
( capital letters are copliments of corresponding letters
a=compliment of a)

[a] a [b] ab [c] abc [d] a(bc)* [e] mone


(bc)*=compliment of bc

ans: e

-------------------------------------
2) a 12 address lines maps to the memory of

[a] 1k bytes [b] 0.5k bytes [c] 2k bytes [d] none

ans: b

----------------------------------------
3) in a processor these are 120 instructions . bits needed to impliment
this instructions
[a] 6 [b] 7 [c] 10 [d] none

ans: b

-----------------------------------------
4) in 8085 microprocessor ready signal does.which of the following
is incorrect statements
[a]it is input to the microprocessor
[b] it sequences the instructions

ans : b
----------------------------------------

5) return address will be returned by function to


[a] pushes to the stack by call
ans : a
------------------------------------------
6)
n=7623
{
temp=n/10;
result=temp*10+ result;
n=n/10
}

ans : 3267
----------------------------------------------
7) if a>b then
f=f(g);
else b>c then
f=g(g);
in this , for 75% times a>b and 25% times b>c then,is 10000 instructions
are there ,then the ratio of f to g
[a] 7500:2500 [b] 7500:625 [c] 7500:625 if a=b=c else
7500:2500
--------------------------------------------------
8) in a compiler there is 36 bit for a word and to store a character 8bits are
needed. in this to store
a character two words are appended .then for storing a k characters string,
how many words are needed.
[a] 2k/9 [b] (2k+8)/9 [c] (k+8)/9 [d] 2*(k+8)/9 [e] none

ans: a
---------------------------------------------------------
9) c program code

int zap(int n)
{
if(n<=1)then zap=1;
else zap=zap(n-3)+zap(n-1);
}
then the call zap(6) gives the values of zap
[a] 8 [b] 9 [c] 6 [d] 12 [e] 15

ans: b
---------------------------------------------------------------

part-b
-------
1) virtual memory size depends on
[a] address lines [b] data bus
[c] disc space [d] a & c [e] none

ans : a
-----------------------------------------------
2) critical section is
[a]
[b] statements which are accessing shared resourses
ans : b
-------------------------------------------------

3) load a
mul a
store t1
load b
mul b
store t2
mul t2
add t1

then the content in accumulator is

ans : a**2+b**4
---------------------------------------------------
4) question (3) in old paper
5) q(4) in old paper
6) question (7) in old paper
7) q(9) in old paper
------------------------------

1]. the following variable is available in file1.c

static int average_float;

all the functions in the file1.c can access the variable

[2]. extern int x;

check the answer

[3]. another problem with

# define true 0

some code

while(true)
{
some code

this won't go into the loop as true is defined as 0

[4]. a question in structures where the memebers are dd,mm,yy.

mm:dd:yy
09:07:97

[5]. another structure question

1 rajiv system analyst

[6]. infile.dat is copied to outfile.dat

[7]. a question with argc and argv .

input will be

c:\temp.exe ramco systems india

-----------------------------------------------------------------------
main()
{
int x=10,y=15;
x=x++;
y=++y;
printf("%d %d\n",x,y);
}

----------------------------------------------------------------------

int x;
main()
{
int x=0;
{
int x=10;
x++;
change_value(x);
x++;
modify_value();
printf("first output: %d\n",x);
}
x++;
change_value(x);
printf("second output : %d\n",x);
modify_value();
printf("third output : %d\n",x);
}

modify_value()
{
return (x+=10);
}

change_value()
{
return(x+=1);
}

----------------------------------------------------------------------------

main()
{
int x=20,y=35;
x = y++ + x++;
y = ++y + ++x;
printf("%d %d\n",x,y);
}

-----------------------------------------------------------------------

main()
{
char *p1="name";
char *p2;
p2=(char *)malloc(20);
while(*p2++=*p1++);
printf("%s\n",p2);
}
----------------------------------------------------------------------

main()
{
int x=5;
printf("%d %d %d\n",x,x<<2,x>>2);
}

--------------------------------------------------------------------

#define swap1(a,b) a=a+b;b=a-b;a=a-b;


main()
{
int x=5,y=10;
swap1(x,y);
printf("%d %d\n",x,y);
swap2(x,y);
printf("%d %d\n",x,y);
}

int swap2(int a,int b)


{
int temp;
temp=a;
b=a;
a=temp;
return;
}
----------------------------------------------------------------------

main()
{
char *ptr = "ramco systems";
(*ptr)++;
printf("%s\n",ptr);
ptr++;
printf("%s\n",ptr);
}

---------------------------------------------------------------------

#include<stdio.h>
main()
{
char s1[]="ramco";
char s2[]="systems";
s1=s2;
printf("%s",s1);
}
-----------------------------------------------------------------

#include<stdio.h>
main()
{
char *p1;
char *p2;
p1=(char *) malloc(25);
p2=(char *) malloc(25);
strcpy(p1,"ramco");
strcpy(p2,"systems");
strcat(p1,p2);
printf("%s",p1);
}

hcl

here hcl came yesterday. they are taking students in 3 streams at


present.
one:
application software
- open to computer science and non-computer science
students
- written test of 1 hour duration consisting of 4 sections

system software:
- open to all
written test of 90 minutes duration

* computer awareness section


-questions from computer science (network, windows, ole,
internet, osi)
* c prgramming fundamentals (c/c++/java) (comparision of c and
java)
* analysis of program code (c/c++/java) (only one question from
java)
* general aptitude (quantitative aptitude, analytical ability)

*********************************************************************
questions from systems:
section 1:
1. piggy backing refers to....
2. osi emulation layer ?
3. parity error checking is used for.........?
4. in a 20 mhz computer, the time taken for a 3clock cycle
instruction is:........
ans: 120 ns (check)
5. convert 0xffe into radix 7
6. internet ip address is unique for..
ans: domain of the node (check)
section 2:
1. comparision of java and c
the feature present in c but not in java
(a) recursion (b)passing variable arguments (c).... (d)...
programs
1.
main()
{
int y = 10;
if(y++ == 10 && y++ != 10 && y++ > 11)
printf("%d",y);
else
printf("2y = %d",2*y);
}
2.
void funca(p)
int *p;
{
*p += 20;
}

void funcb(m)
int *m;
{
int n;
m = &n;
*m += 10;
}

main()
{
int var = 25;
int *varp;
varp = &var;
funca(varp);
*varp += 10;
funcb(varp);
printf("%d...%d",var,*varp);
}
3.
main()
{
static int *p = 100;
char *q = p;
p++;
q++;
printf("%d...%d",p,q);
}
/* don't run it. its a java program */
/* find the error if any */
class a
{
static final int var = 10;
private :
int obj1;
public :
char obj2[]="some string";
func()
{
var = 15;
system.out.obj1;
system.out.obj2;
}
}
choices:
1) there is no semicolon at the end of the class
2) the variable var cann't be changed after declaration
3) class a has not been declared as of public type
4) string can not be printed directly
ans: ?
5.
/* declare static int i; globally
1) the variable is available for all the functions after its
declaration
2) the variable is not available for other than the current program
3) .....
4) .....
*/

static int i;
main()
{
void fun1();
printf("%d\n",i);
fun1();
}

void fun1()
{
printf("%d\n",i);
}

aptitude:
gre book, page no. 300 range, anlaytical ability question
a causes b or c but not both....
*********************************************************************

all the best and good luck.

reply to me soon.

****************************************************************
v. subramanian,
69, sapphire hostel,
regional engineering college,
tiruchirappalli - 620 015.
****************************************************************
today is made for you.

>>from sushant@me.iitb.ernet.in tue nov 4 22:08:31 1997


date: tue, 4 nov 1997 22:06:23 +0530 (ist)
from: "k.s .sushant kumar" <sushant@me.iitb.ernet.in>
to: susant@urja.me.iitb.ernet.in

this is the tcs paper held at iitm.

tcs is as follows.

hai guys

107 members were selected out of 400+ attentd for written and 200+ were short
listed for intv's. 40+ from m.tech 30+ from b.tech and remaining from ms.
intv's for phds are yet to be started.

there is a rumour that, if age is above 24 tcs will not take.


it is absolutely wrong. here they took fellows who having less cgpa and who
does't know minimum basics about compuer knowledge.

so what i am saying is we should have some fate at that time.

paper-97
___________________

--------------------------------------------------------------------------------
calicut tcs paper --1997
--------------------------------------------------------------------------------

section 1 inturn consists of three parts.

part 1
verbal---
it consists of 40 words which has 5 multiple answers
for them.the main words which i remember are as follows for which
you should learn the synonyms of these---
merry = gay

alienate = isolate, antagonize, separate, estranged

solicit = request, ask

heep = gather

cargo = consignment, goods (wool)

momentary = short time, temporary

volume =

veer = change of direction

disperse = scatter
admonish = rebuke , scolding

meager = small amount

latitude = parallel, scope

latent = hidden

covet = desire

discretion = tact, judicially

potential = capacity

emanicipate = liberate

lethargy = dull

concur = agree

confiscate = seize, impound, bannish

dispel = away

baffle = puzzle

subside = reduce the value

misery = sorrow

pretentious = pompous, showy

tranquil = calm

efface = erase, rub out

effigee = image, carving statue

abstinate = refraining, to resist

obstinate = self willed, stubborn

hover = hang over

caprice = vagry, impulse, changes mind without any reason, whim

belate = late

renaissance = interest on particular topic, revival of interest

abstriate = forming abstract

brim = edge

divulge = disclose

lament = express sadness


belief =

embrace = hug

adhesive = tenatitous

miserable = grief

furtue = secrete

section 1 part b 20min 30 marks 15 quantitative.


~~~~~~~~~~~~~~~~~~~~~~~~~~~~~~~~~~~~~~~~~~~~~~~~~~~~~~~~~~~~~~~~~~~
these are some problems :
1.) two pencils cost 8 cents. then 5 pencils cost?
(20 cents)

2. a work is done by the people in 24 minutes. one of them can do


this work alonely in 40 minutes. how much time to do the same work
for the second person?
(60 minutes) 1/24-1/40=1/60

3. a car is filled with four and half gallons of fuel for a round trip.
fuel is taken 1/4 more in going then coming. what is the fuel
consumed in coming up? (2 gallons)
(1+1.25)x=4.5
x = 2 gallons

4.low temperature at the night in a city is 1/3 more than 1/2 high as
higher temperature in a day. sum of the low tem. and highest temp.
is 100 degrees. then what is the low temp? (40 deg.)

ans.
(x/2)+x/3+x=100
x=600/11
5x/6=500/11=45.45

5. a person, who decided to go to weekened trip should not


exceed 8 hours driving in a day. average speed of forward journey
is 40 m/h. due to traffic in sundays, the return journey average speed
is 30 m/h. how far he can select a picnic spot?
a) 120 miles
b) between 120 and 140 miles
c) 160 miles
x/40+x/30=8
x=120*8/7=137.14

6. a salesperson multiplied a number and get the answer 3,


instead of that number devided by 3.
what is the answer he actually has to get?
1 x 3 = 3
so number = 1
devided by 3, the ans. is 1/3.

7. a ship started from port and moving with i miles per hour and another
ship started from l and moving with h miles per hour.
at which place these two ships meet?
|----|----|----|----|----|----|
port g h i j k l

ans.
ix+hx=l
x=l/(i+h)
xi=li/(i+h)

8. a building with height d shadow upto g. a neighbour building with


what height shadows c feet.

|----|----|----|----|----|----|----|
a b c d e f g h

cd/g

9. a person was fined for exceeding the speed limit by 10 mph.


another person was also fined for exceeding the same speed limit
by twice the same. if the second person was travelling at a
speed of 35 mph, find the speed limit. (15 mph)

10*2=20
35-20=15 mph

10.a bus started from bustand at 8.00am, and after 30 minutes staying
at destination, it returned back to the busstand. the destination
is 27 miles from the busstand. the speed of the bus is 18mph. in
return journey bus travels with 50% fast speed.
at what time it returns to the busstand? (11.00am).

11.in a mixture, r is 2 parts, s is 1 part. in aoder to make s to


25% of the mixture, howmuch r is to be added?

section-1 part-3 marks-50 questions-50 30 min

critical reasoning

it is very difficult to reproduce them and so we are giving here the


first question of the section so that you may have a general idea of the
section.be careful time management is important.
you have to mark yes, no or we can't say..
my father is only child to his father.my father has three sisters.
all are married and have two children each.

1) my grandfather has two sons.


2) i am having six cousins.
3) i have three uncles.

section-2 psychometric test 150 questions 50 marks.

in this section there will be questions related to your psychology


you will have to say yes ,no or i cant say. you mark them in the
most positive way .but we are not certain weather the paper is
corrected or not .

--------------------------------
--------------------------------

note:

in addition the above calicut paper we are adding some passages below ,
whichever not covered by them.

vocab
some corrections are there in your paper please make a note

admonish = to caution, to warn (not usurp)


adhesive = tenatious
incentive = -------------

quant

critical reasoning

madhu here in this section time is very less that you just can't even read
the total passages. that's why you should have an idea about the passage
before going to the test. and put the ans just studying the q's.
8 passages which you have sent are ok i am sending the remaining ones
with this info you can answer the q's.i am sending only that portion of the
passage which is useful in answering the q's.

*)passage related to concious and un concious world


----------------------------------------------------
there is an influence of dreams on the waking state(sleeping can be creative)
the other q's given in under this passage can not be answered just with
the info given in it so don't struggle to read it.

q's
1) sleeping can be creative
2) it's difficult to tell whether a person is dreaming.
3) if we could record the drems of prenatal >.........some conclusion.

*)passage related to pests


--------------------------

funda is that though pests offer threat to flora and fauna two methods are
there to prevent. one is natureal and the other is some engg.. pest controller.
both the methods should be regarded as potential ones.

just two q's under this


1)engg pest controller should be regarded as potential method
2)this q can be answered with the first two lines of the passage.

*)passage related to words


--------------------------
words have unhealthy importance in totalitarian days. people have to chose the
words cautiously. the words have political importance in those days.
words are devalued in democratic rule people are free to chose them and we
can say that they come free of cost.

q's
1)words have no importance in totalitarian days.
2)poets ----------
3)

*)passage related to tagore works


---------------------------------
every form of lit.. comes under copy write act. people having tagore works
are earning more,there should be a trust for that.

q's
1)tagore works comes under the copy write act.
2)regarding the trust
3)people going for movies are paying for the devolopment of arts .......

*)passage related to facial feelings


------------------------------------

natural expressions have symmetric facial expressions. negetive expressions


have assymetric facial expressions. left sided expression dominates for a
personwith negetive feelings (some thin is given about acters and this passage is
very simple you can do it.)

q's
1)
2)an actor when smiling poses symmetrical face feelings
3)an angry man poses more lop sided face feelings
(madhu you better go through this passage it's very easy).

psycho.. test

don't worry about this section. you should make sure that you are choosing the
same alternative when ever you encounter the same q.. in different forms.
the most repeating ones i am sending . you must answer all the 150 q's and
eliminate can't say as far as possible.

*)you can work for a long time with out rest.


the other forms are like this
*)you want to relax freq..

*)you feel difficult to introduce to new friends.


*)you can easily make new friends
*)you want to limit your friends circle
*)you feel good at talking with new people
-----
----
the importaant thing is that you should fill all the 150 pshycometry quetions.
1]. the following variable is available in file1.c

static int average_float;

all the functions in the file1.c can access the variable

[2]. extern int x;

check the answer

[3]. another problem with

# define true 0

some code

while(true)
{
some code

this won't go into the loop as true is defined as 0

[4]. a question in structures where the memebers are dd,mm,yy.

mm:dd:yy
09:07:97

[5]. another structure question

1 rajiv system analyst

[6]. infile.dat is copied to outfile.dat

[7]. a question with argc and argv .

input will be

c:\temp.exe ramco systems india

-----------------------------------------------------------------------

main()
{
int x=10,y=15;
x=x++;
y=++y;
printf("%d %d\n",x,y);
}

----------------------------------------------------------------------

int x;
main()
{
int x=0;
{
int x=10;
x++;
change_value(x);
x++;
modify_value();
printf("first output: %d\n",x);
}
x++;
change_value(x);
printf("second output : %d\n",x);
modify_value();
printf("third output : %d\n",x);
}

modify_value()
{
return (x+=10);
}

change_value()
{
return(x+=1);
}

----------------------------------------------------------------------------

main()
{
int x=20,y=35;
x = y++ + x++;
y = ++y + ++x;
printf("%d %d\n",x,y);
}

-----------------------------------------------------------------------

main()
{
char *p1="name";
char *p2;
p2=(char *)malloc(20);
while(*p2++=*p1++);
printf("%s\n",p2);
}
----------------------------------------------------------------------

main()
{
int x=5;
printf("%d %d %d\n",x,x<<2,x>>2);
}

--------------------------------------------------------------------

#define swap1(a,b) a=a+b;b=a-b;a=a-b;


main()
{
int x=5,y=10;
swap1(x,y);
printf("%d %d\n",x,y);
swap2(x,y);
printf("%d %d\n",x,y);
}

int swap2(int a,int b)


{
int temp;
temp=a;
b=a;
a=temp;
return;
}
----------------------------------------------------------------------

main()
{
char *ptr = "ramco systems";
(*ptr)++;
printf("%s\n",ptr);
ptr++;
printf("%s\n",ptr);
}

---------------------------------------------------------------------

#include<stdio.h>
main()
{
char s1[]="ramco";
char s2[]="systems";
s1=s2;
printf("%s",s1);
}

-----------------------------------------------------------------

#include<stdio.h>
main()
{
char *p1;
char *p2;
p1=(char *) malloc(25);
p2=(char *) malloc(25);
strcpy(p1,"ramco");
strcpy(p2,"systems");
strcat(p1,p2);
printf("%s",p1);
}

- tcs-
> section 1 inturn consists of three parts.
>
> part 1
> verbal---
> it consists of 40 words which has 5 multiple answers
> for them.the main words which i remember are as follows for which
> you should learn the synonyms of these---
>
> merry
> alienate
> solicit
> heap
> cargo
> momentary
> volume
> veer
> dispel
> admonish
> meager
> latitude
> latent
> covet
> discretion
> latent
> emanicipate
> lethargy
> concur
> confiscate
> baffle
> subside
> misery
> pretentious
> tranquil
> efface
> obstinate
> hover
> caprice
> belate
> renounce
> brim
> divulge
> lament
> bileaf
> adhesive
> hamper
> to merit
>
>
> section 1 part b 20min 30 marks 15 quantitative.
> ~~~~~~~~~~~~~~~~~~~~~~~~~~~~~~~~~~~~~~~~~~~~~~~~~~~~~~~~~~~~~~~~~
> these are some problems :
>
> 1.) two pencils cost 8 cents. then 5 pencils cost?
> (20 cents)
>
> 2. a work is done by the people in 24 minutes. one of them can do
> this workalonely in 40 minutes. how much time to do the same work
> for the second person?
> (60 minutes)
>
> 3. a car is filled with four and half gallons of fuel for a round trip.
> fuel is taken 1/4 more in going then coming. what is the fuel
> consumed in coming up? (2 gallons)
>
> 4.low temperature at the night in a city is 1/3 more than 1/2 high as
> higher temperature in a day. sum of the low tem. and highest temp.
> is 100 degrees. then what is the low temp? (40 deg.)
>
> 5. a person, who decided to go to weekened trip should not
> exceed 8 hours driving in a day. average speed of forward journey
> is 40 m/h. due to traffic in sundays, the return journey average speed
> is 30 m/h. how far he can select a picnic spot?
> a) 120 miles
> b) between 120 and 140 miles
> c) 160 miles
> ans: 120 miles
>
> 6. a salesperson multiplied a number and get the answer 3,
> instead of that number devided by 3.
> what is the answer he actually has to get?
> 1 x 3 = 3
> so number = 1
> devided by 3, the ans. is 1/3.
>
> 7. a ship started from port and moving with i miles per hour and another
> ship started from l and moving with h miles per hour.
> at which place these two ships meet?
>
> |----|----|----|----|----|----|
> port g h i j k l
>
> 8. a building with height d shadow upto g. a neighbour building with
> what height shadows c feet.
>
> |----|----|----|----|----|----|----|
> a b c d e f g h
>
> 9. a person was fined for exceeding the speed limit by 10 mph.
> another person was also fined for exceeding the same speed limit
> by twice the same. if the second person was travelling at a
> speed of 35 mph, find the speed limit. (15 mph)
>
> 10.a bus started from bustand at 8.00am, and after 30 minutes staying
> at destination, it returned back to the busstand. the destination
> is 27 miles from the busstand. the speed of the bus is 18mph. in
> return journey bus travels with 50% fast speed.
> at what time it returns to the busstand? (11.00am).
>
> 11.in a mixture, r is 2 parts, s is 1 part. in aoder to make s to
> 25% of the mixture, howmuch r is to be added?
>
>
> 12. with 4/5 full tank vehicle travels 12 miles, with 1/3 full tank
> how much distance travels ( 5 miles )
>
> 13. two trees are there. one grows at 3/5 of the other in 4 years,
> total growth of trees is 8 ft. what growth will smaller tree will have
> in 2 years ( < 2 ft. )
>
>
> section-1 part-3 marks-50 questions-50 30 min
>
>
> critical reasoning
>
>
>
> 1. my father has no brothers. he has three sisters who has two childs each.
> my grandfather has sons.
>
> a) my grandfather has two sons
> b) three of my aunts have two sons
> c) my father is only child to his father
> d) i have six cousins from jmy mother side
> e) i have one uncle
>
> 2. ether injected into gallablader to dissolve galstones. this type oneday
> treatment is enough for gallastones not for calcium stones. this method is
> alternative to surgery for millions of people who are suffering from this
> disease.
>
> a) calcium stones can be cured in oneday
> b) hundreds of people contains calcium stones
> c) surgery is the only treatment to calcium stones
>
> 3. hacking is illegal entry into other computer. this is done mostly because
> of lack of knowledge of computer networking with networks one machine can
> access to another machine. hacking go about without knowing that each network
> is accredited to use network facility.
>
> a) hacking people never break the code of the company which they work for
> b) hacking is the only vulnerability of the computers for the usage
> of the data.
> c) hacking is done mostly due to the lack of computer knowledge.
> (there will be some more questions in this one)
>
>
> 4. in the past helicopters are forced to ground or crash because of
the
> formation of the ice on the rotors and engines. a new electronic device
> has been developed which can detect the watercontent in the atmosphere and
> warns the pilot if the temp. is below freezing temp. about the formation
> of the ice on the rotors and wings.
>
> a) the electronic device can avoid formation of the ice on the wings
> b) there will be the malfunction of rotor & engine because of formation
> of ice
> c) the helicopters are to be crashed or down
> d) there is only one device that warn about the formation of ice.
>
>
>
> 6. in confucius days the technology development was less and it took
weeks
> to communicate a message. wherein we can send it through satellite with in
> no time........ even with this fast developments it has become difficult to
> understand each other.
>
> a) people were not intelligent during confucius days
> b) transport facilities are very much improved in now-a-days
> c) even with the fast developments of the technology we cannot live
> happily.
> d) we can understand the people very much with the development of
> communication.
>
> 7. senior manager in a big company said that new japanies company
invades
> in india for transfering the cars from industrial and warned that jobs were
> under threat from japanies company. they stated that increasing competence
> would be coupled with an inevitable down term in car market and recent rise
> in interest rate which has already hit demand.
>
> a) manager issue their warning after a rise in interest rate
> b) manager told workers that japanies workers are taking jobs away from
> indian workers
> c) manager said that more people want to buy new cars in future
> d) increasing rate of interest mean that japanies firm will create into
> operate in the country
>
> 8. human existence is suspecious of arbitrary divide between concise
and
> unconcise. the concise world invades shape activity of the unconcise, while
> many of great activity of humanity waking as whole or partially improved
> by dreams. even it could be ignored that dreams precede exceptional such a
> dichotomy could not be drawn as the influence of dream on waking state would
> reamin unclear. but as yet no company rebuilt exists to record the substitute
> of prendtl dreaming.
>
> a) sleepy can be creative state
> b) it is difficult to tell whether a sleeper is dream or not
> c) if we know what babies would dream about before they are born we could
> show that the concise and unconcise mind influence on one another
> d) it is untrue claim that concise and unconcise world never impinge
> one another
>
> 9. any one who has has systematic exam phases will have perceived a
> profound although not a prolif of asymmetry whether or not the exception
> is volitions and self control of spontoneous appeal to predict facial as
> symmetry as does the type of emotion potrayed. position can not displace
> symmetric at left side regret of a negative emotion is more common posed
> expression negation emotions are likely to be symmetric representation
> and where as symmetric occurs relative left sided expression is more common.
>
> a) any angry person is more likely to have left sided expression than
> some one who has smiling
> b) an actor is likely to smile symmetric when acting
> c) delecious facial expression will always be as symmetrical
>
>
> 10. there should be copyright for all arts. the rule has come that
all the
> arts has come under one copy right society, they were use the money that
> come from the arts for the developments. there may be a lot of money will
> come from the tagore works. we have to ask the benifiters from tagore work
> to help for the development of his works.
>
> a) tagore works are come under this copy right rule
> b) people gives to theater and collect the money for development
> c) people are free to go to the because of the copy right rule
> d) we have ask the tagore residents to help for the developments of art.
>

Vous aimerez peut-être aussi